Facebology 5th year exam notes

Page 1


Facebology Fifth Year Exam Notes

Name: ………………………..

2


‫ميحرلا نمحرلا هللا مسب‬ ‫يقدم لكم فريق ‪ Facebology‬مذكر ة أسئلة أمتحانات ألسنوأت ألسابقة للفرقة ألخامسة في‬ ‫مادتي‪:‬‬ ‫‪Internal Medicine & Pediatrics‬‬ ‫تحتوي ألمذكرة على طريقتين لعرض ألسئلة ألولى ألمتحان كامل وألثانية على شكل تجميعات‬ ‫للسئلة أللتي تكررت في كل أمتحان لكل فرع من ألمادة ‪.‬‬ ‫نتمنى أن ينال هذأ ألعمل رضاكم فقد أمضى عليه ألفريق من ألوقت وألجهد حتى يسهل على‬ ‫ألطالب ألوصول للمعلومة وعرض ألمتكرر من ألسئلة باك ثر من طريقة لتعم ألفائدة للجميع ‪.‬‬ ‫زملؤنا ألعزأء ‪ ...‬إن دأئما ما يشغل تفكيرنا هو أنتم وألوصول بكم لتحقيق أعلى درجات ألنجاح‬ ‫وتحقيق ألمهارة وألك فاءة ألعلمية لكافة أفرأد دفعتنا ومن ياتوأ بعدنا فادرس وأجتهد وأعمل ودع‬ ‫ألنتائج على هللا ‪.‬‬ ‫ّ‬ ‫لقد ّ‬ ‫شرفنا هللا وكلفنا باسمى ألمهن فل تدرس للمتحان فقط وضع دأئما في ذهنك أنك ستتخرج‬ ‫وتصبح طبيبا تقع بين يديه حياة ألبشر ‪.‬‬

‫‪‬‬ ‫تفضلوأ بزيارة صفحتنا على ‪: Facebook‬‬ ‫‪https://www.facebook.com/Facebology‬‬ ‫‪Facebology Team‬‬ ‫‪2‬‬

‫‪3‬‬


Content

Page

6

69

97

138

4



Tanta University Faculty of Medicine Time allowed three hours

M.B.B.Ch. Internal Medicine Exam. Fifth Year Exam. Paper I August 2015

All questions must be answered (5 questions) (150 marks)/the exam is in two pages: 1. A) A 42 years-old male presents to the emergency room with one day history of vomiting of fresh blood, he also reported passing black tarry stool for the last 3 days and feeling generally unwell for the last 6 weeks. He noticed increase of the abdominal girth. He was diagnosed as having HCV hepatitis 3 years ago. On examination: he was disoriented with flapping tremors, jaundice and pale. Pulse 100 beat/m. Bp: 80/60mmhg. Abdominal examination, revealed pal- pable liver 2 cm below right costal margin, firm in consistancy, moderate spleenomegaly and moderate ascites. 1. Which of the following is true regarding loop diuretics therapy for this patient? (2 marks) and explain. (3 marks) a- Intravenous intake in high dose. b- Oral intake in low dose. c- Intravenous intake in small dose. d- Not recommended for this patient. 2. What are the manifestations of portal hypertension in this case? (3 marks) 3. Enumerate causes of portal hypertension. (7 marks) B) Describe causes (5 marks), clinical manifestations (6 marks) and complications of acute pancreatitis. (4 marks) C) Describe pathology (2 marks), clinical picture (5 marks) and investigations of crohn’s disease (3 marks). 2. A) A 42 years-old woman presented with renal colic secondary to renal stone. She also com- plains of fatigue and weakness in all extremities. Serum calcium level was 13mg/dl, referance range (8-10.5 mg/dl) and phosphate level was 1.9 mg/dl, referance range (2.3-5 mg/dl). Serum creatinine 0.9 mg/dl. Parathyroid hormone level is high. 1. Which of the following is your provisional diagnosis (2 marks) and explain. (3 marks) aHypvitaminosis D. b- Secondary hyperparathyroidism. c- Primary hyperparathyroidism. d- Hypothyroidism. 2. Enumerate 5 causes of hypercalcemia. (3 marks) 3. Describe treatment of hypercalcemia. (7 marks) B) Enumerate: 1. 5 chronic complications of diabetes mellitus. (5 marks) 2. 5 diagnostic criteria of metabolic syndrome. (5 marks) 3. 5 groups of oral hypoglycemic drugs. (5 marks) C) Describe clinical picture (5 marks) and enumerate 5 investigations of acromegaly (5 marks).

6


3. A) A 25 years- old man presented to the clinic complaining of several days of pain upon Micturation and Left hip, left knee and right ankls pain. He states that all of these symproms were preceded two weeks ago by diarrhea and gastroenteritis. His examination is significant for severe conjunctivitis of his right eye. 1. Which of the followings is the most likely diagnosis (2 marks) and explain. (3 marks) a- Rheuatic arthritis. b- Osteoarthritis. c- Reiter’s syndrome. d- Gout. 2. Enumerate 5 causes of seronegative spondylarthritis. 3. Enumerate 5 extra-articular manifestations of rheumatoid arthritis. B) Describe pathogenesis (3 marks) and 5 clinical presentations of scleroderma. C) Describe 5 diagnostic createria of SLE.

(5 marks) (5 marks) (7 marks) (5 marks)

4. A) Describe complications (5 marks) and management (5 marks) of acute mycardial infarction. B) Describe causes (3 marks), diagnosis (2 marks) and management of atrial fibrillation. (5 marks) 5. A) Describe the transmission (2 marks), clinical picture (4 marks) abd complications of brucellosis. (4 marks) B) Give an account on: 1. Causes of fever with spleenomegaly. (5 marks) 2. Amebic liver abscess management. (5 marks)

7


Tanta University Faculty of Medicine Time allowed three hours

M.B.B.Ch. Internal Medicine Exam. Fifth Year Exam. Paper II August 2015

All questions must be answered (5 questions) (150 marks)/The exam is in two pages. 1. A) A 13 year old-male was noticed by his parents to have periorbital oedema. Ten days earlier he complained of a sore throat and had recieved on week course of antibiotic treatment. He also complains of scanty urine, which is slightly smoky during the previous 48 hours. On examination the jagular venous pressure was raised and blood pressure was 150/100mmHg, Hb 10.5gm/dl, WBCs 9*103/dl, serum albumin 4gm/dl. Dipstick testing of his urine is positive for blood and protein. No history of loin pain, no skin or mucous membrane affection, no arthritis. 1. Which of the following is your provisional diagnosis? (2 marks) and explain. (3 marks) a- Chronic kidney disease. b- Lupus nephritis. c- Nephrotic syndrome. d- Acute glomerulonephritis. 2. What is the pathogenesis of periorbital oedema (3 marks) and hypertension (2 marks) in this case? 3. What is the treatment of this condition? (7 marks) B) Define micro-albuminuria (3 marks), describe its significance (3 marks) and methods of its detection. (4 marks) C) Describe 5 of: 1. Urinary bladder causes of hematuria. (5 marks) 2. Complications of nephrotic syndrome. (5 marks) 3. Investigations of chronic pyelonephritis. (5 marks) 2. Enumerate: 1. 5 causes of acute abdominal pain. (5 marks) 2. 5 complications of chemotherapy. (5 marks) 3. 5 causes of short stature. (5 marks) 4. 5 causes of syncope. (5 marks) 3. A) A 22 year- old man came to the clinic with general fatigue abd a bleeding tendency from gums since 2 months. He developed fever in the last week. On examination, temprature 38C, purpuric eruption all over the body. No organomegaly, no lymphadenopathy. Haemoglobin was 5.5 gm/dl, RBCs 1.9*106 /dl, WBCs count 2*103 /dl and platelets 25*103 /dl. The reticulocyte is 0.2%. He started treatment with folic acid, vit. B12 and ferrous sulphate with no improvement. 1. Which of the following is your provisional diagnosis? (2 marks) and explain. (3 marks) a- Chronic myeloid leukemia. b- Hyperslenism. c- Iron deficiency anemia. d- Aplastic anemia. 2. How can you prove your diagnosis? (3 marks) 3. How do you treat this case? (7 marks)

8


B) Describe: 1. 5 lines of treatment of idiopathic thrombocytopenic purpura. 2. 5 diagnostic investigations for multiple myeloma. 3. 2 investigations and 3 replacement therapies for haemophilia A.

(5 marks) (5 marks) (5 marks)

4. A) Describe management of a case of pulmonary tuberculosis. B) Enumerate: 1. Causes of exudative pleural effusion. 2. Risk factors for COPD. 3. Complications of bronchiectasis.

(10 marks)

5. A) Ataxia: types and differential diagnosis. B) Generalized anaxiety disorders: definition, etiology, clinical picture and management.

(15 marks) (15 marks)

Good Luck

9

(4 marks) (3 marks) (3 marks)


Tanta University Faculty of Medicine Time allowed three hours

M.B.B.Ch. Internal Medicine Exam. Fifth Year Exam. Paper III September 2015

All questions must be answered (5 questions) (150 marks)/The exam is in two pages. 1. A) Enumerate 5 causes of the followings: (20 marks) (5 marks/question) 1. Pigmentation around the umbilicus. 2. Dullness in the 2nd intercostal space. 3. Constipation. 4. Acute chest pain. B) Enumerate 5 causes of the followings: (20 marks) (5 marks/question) 1. Mono-articular arthritis. 2. Sinus tachycardia. 3. Generalized oedma. 4. Congested neck veins. C) Enumerate 5 causes of the followings: (20 marks) (5 marks/question) 1. Impaired note over Kronig’s isthmus. 2. Non-chest causes of haemoptysis. 3. Pigmentation at butterfly area of the face. 4. Huge spleenomegaly. 2. Physical medicine: A) Enumerate 5 indications of electrodiagnosis. B) Give an account on physical management of peripheral nerve injury. 3. Dermatology: A) Mention papulosquamous skin diseases. B) Enumerate causes of organic erectile dysfunction. C) Give sort account on clinical varieties of scabies. D) Give short account on systemic and local treatment of acne vulgaris. E) Choose the correct answer: (10 marks)

10

(5 marks) (10 marks)

(5 marks) (5 marks) (5 marks) (5 marks)


1. All of the following are DNA viruses causing skin diseases except: a- Pox virus. b- Human papilloma virus. c- Herpes simplex virus. d- HIV. 2. All of the following are characters of impetigo contagiosum except: a- Caused by staphylococci, streptococci or both. b- Pearly white centrally umblicated papules. c- Treated by hot fomentation of potassium permangenate and antiseptic agent. d- Occur at any age but more common in children. 3. All of the followings are forms of physical urticaria except: a- Dermographism. b- Cholinergic urticaria. c- Angioedema. d- Aquagenic urticaria. 4. All are true concerning gonorrhea in adult females except: a- Asymptomatic in about 50% of patients. b- Cervix and vagina are involved. c- may be complicated by spontaneous abortion or premature labor. d- culture is mandatory for diagnosis. 5. Lepromatous leprosy is characterized by all of the following except: a- Prodromal symptoms. b- Mucous membrane involvement. c- Nodular lesions of the face and leonine appearance. d- Asemmetrical peripheral nerve affection. 6. All are treatment modalities of vitilligo except: a- Systemic corticosteroids. c- Pseudocatalase. b- Systemic antibiotics. d- Calcineurin inhibitors. 7. Systemic antifingal used in treatment of pityriasis versicolor: a- Grisoflavin. c- Itraconazole. b- Terbinafine. d- Amphotricin B. 8. Initial lesion of erythema multiformis is: a- Burrow. c- Herald patch. b- Iris pattern. d- Pearly white centrally umbilicated papule 9. All are causes of cicatricial alopecia include all of the followings except: a- Favus. c- Lichen planopilaris. b- Basal cell carcinoma. d- Secondary syphilis. 10. Chancre is characterized by the followings except: a- Painless eroded papule. b- Caused by treponema palladium. c- Tender matted regional lymphadenopathy. d- Diagnosed by dark-ground illumination microscope.

11


4. Clinical pathology (Choose the correct answer): (30 marks) 1. The characteristic erythrocyte found in pernicious anemia is: a- Microcytic. c- Hypochromic. b- Spherocytic. d- Macrocytic. 2. A characteristic morphologic feature in B Thalassaemia major is: a- macrocytosis. c- Spherocytosis. b- Target cells. d- Microcytosis with prominent target cells. 3. Howel-Jolly bodies are seen in: a- Red blood cells after splenectomy. b- Red blood cells after irradiation. c- White blood cells in megaloblastic anemia. d- None of the above. 4. Interpretation of hemoglobin is afected by diurnal variation: a- It’s the highest in the evening. b- It’s the lowest in the morning. c- It’s the highest in the morning. d- None of the above. 5. Ineffective erythropoiesis occurs in the following diseases except: a- Megaloblastic anemia. c- Sideroblastic anemia. b- Thalassaemia. d- Aplastic anemia. 6. Elevation of the total white blood cell count above 12,000/ul is termed: a- Absolute lymphocytosis. c- Relative lymphocytosis. b- Leukocytosis. d- Relative neutrophilia. 7. The presence of excessive rouleaux formation in blood smear is often accompanied by an increased: a- Reticulocytic count. b- Sedimentation rate. c- Hematocrite. d- Erythrocytic count. 8. A hypercellular marrow with an M/E ratio of 5:1 is most commonly due to: a- Lymphoid hyperplasia. b- Myeloid hyperplasia. c- Normoblastic hyperplasia. d- Myeloid hypoplasia. 9. Prothrombin is: a- A protein formed by the liver in the presence of Vitamin K. b- An enzyme that coverts fibrinogen into fibrin threads. c- The end product of the reaction between fibrinogen and thrombin. d- A protein released by platelets during coagulation. 10. Activation of protein C needs: a- Thrombin-anti-thrombin complex. b- Thrombin-thrombomodulin complex. c- Thrombomodulin and divalent ions. d- Thrombin-thrombomodulin complex in the presence of Ca+2.

12


11. In von Willebrand disease, platelets give an abnormal aggregation result in the presence of: a- ADP. b- Epinephrine. c- Collagen. d- Ristocetin. 12. Which of the following tests is to monitor red cell production: a- Packed cell volume b- Schilling test. c- Reticulocytic count d- Mean corpuscular hemoglobin 13. A bone marrow shows foam cells, vacuolated cytoplasm, containing sphingomeylin and is faintly PAS +, is mostly characteristic of: a- Gaucher disease b- Myeloma with Russell bodies c- DiGuglieimo disease d- Niemann-Pick disease 14. Mononuclear cell with abundant, pale blue cytoplasm and variable number of elongated projections is characteristic for: a- Acute myeloid leukemia b- Hairy cell leukemia c- Acute lymphoblastic leukemia d- Chronic lymphocytic leukemia 15. In the French-American-British “FAB� classification, myelomonocytic leukemia would be: a- M1 and M2 b- M3. c- M4 d- M5 16. Auer rods: a- Contain lactoferrin b- Are lysosome and acid phosphatase- positive c- Are found in leukemic phase of lymphoma d- Are found in acute lymphoblastic leukemia 17. All are giant cells in bone marrow Except: a- Megakaryocyte. b- Osteoclast. c- Proerythroblast. d- Reed-Stemberg cell. 18. All can cause artificial thrombocytopenia except: a- Platelet clumping by EDTA-platelet agglutinins. b- Platelet Satellitism. c- Giant platelet. d- Excess EDTA in blood sample.

13


19. Red cell diameter width (RDW) is a sensitive measurement of: a- Total cell count. b- Red cell anisocytosis. c- Packed cell volume. d- Red cell/ platelet ratio. 20. The following stain is used for reticulocytic count a- Leishman’s stain. b- Giemsa stain. c- Sudan black stain. d- None of the above. 5.

Radiology (Choose the correct answer): (15 marks) 1. Ionizing imaging techniques include: a- Ultrasonography. b- Magnetic Resonance Imaging (MRI). c- X-Rays. d- Isotope scans. e- C & D. 2. Straightening of the left cardiac border in a well-positioned PA chest x-ray is produced by the following: a- Prominence of the aortic knuckle. b- Pulmonary artery dilatation. c- Enlarged left atrial appendage. d- All of the above. e- B & C. 3. Signs of right ventricular enlargement in Chest & Heart X-Rays include: a- Downward & outward displacement of the cardiac apex. b- Outward displacement of the cardiac apex. c- Obliteration of the retrosternal air space in the lateral view. d- B & C. e- All of the above. 4. Signs of pericardial effusion in PA Chest & Heart X-Ray include: a- Enlargement of the cardiac shadow. b- Pulmonary venous congestion & edema. c- Clear lung fields. d- A & C. 5. Simple pleural effusion in PA Chest X-Ray is seen as: a- Homogenous density rising to the axilla. b- Obliteration of the costophrenic angle. c- Cardiac & mediastinal shift. d- Ail of the above. e- A & B.

14


6. The following conditions show an air-fluid level in Chest X-Ray: a- Lung abscess. b- Hydropneumothorax c- Pulmonary segmental collapse. d- A & B. e- A & C. 7. Intestinal obstruction in plain abdominal standing x-rays is seen as: a- Multiple air fluid levels in the bowel loops. b- Presence of air & fecal matter at the rectum. c- Air under the diaphragm. d- Radio-opaque shadows. e- A & C. 8. IV contrast is contraindicated in the following except: a- Pregnancy. b- Impaired renal function. c- Hypersensitivity. d- IVU studies. e- None of the above. 9. The esophageal cancer appears in barium swallow as: a- Moderate esophageal dilatation. b- Irregular stricture segment. c- Shouldering at the margins d- All of the above. e- A & B. 10. The diaphragmatic hernias include: a- Morgagni hernia. b- Hiatus hernia. c- Bockdalek hernia. d- All of the above. e- B & C. 11. Stricture at the ileocecal junction with cecal elevation & narrowing in Barium Enema is: a- Ileocecal TB. b- Carcinoma of the cecum. c- Crohn’s disease. d- A & C. e- All of the above. 12. Dilated renal calyces, renal pelvis & the ureter in IVU is caused by: a- Stricture lower end of the ureter. b- Vesicoureteric reflux disease c- Pelviureteric junction obstruction. d- A & B. e- B & C.

15


13. Bilateral hydroureteronephrosis in IVU occurs in: a- Bilateral vesicoureteric reflux. b- Prostatic enlargement. c- Urethral valve disease. d- All of the above. e- B & C. 14. Calyceal destruction in IVU is caused by: a- Renal cell carcinoma. b- Renal cystic disease. c- Radiolucent stone. d- B & C. e- All of the above. 15. Splenomegaly can be caused by: a- Portal .Hypertension. b- Lymphomas. c- Haemolytic anaemias. d- Enteric fever. e- All of the above.

16


Tanta University Faculty of Medicine Time allowed three hours

M.B.B.Ch. Internal Medicine Exam. Fifth Year Exam. Paper I October 2015

All questions must be answered (5 questions) (No: 4 & 5 are in the back of paper) – 150 marks 1. A) A 57 years-old male with a 30 years history of smoking presents with difficulty swallowing solid food, this has been since about five months and now has progressed to difficulty swallowing liquids. He has decreased appetite and lost about 10 Kg in three months. 1. Which of the followings is your provisional diagnosis (2 marks) and explain? (3 marks) a- Achalasia b- Pyloric obstruction c- Cancer esophagus d- Peptic ulcer 2. Describe definition (2marks) and enumerate causes of dysphagia (3 marks) 3. Describe diagnostic work up for dysphagia (5 marks) B) Enumerate 5 causes of cirrhosis (5marks) and describe 2 diagnostic investigations for each cause (5 marks). C) Describe pathology, (4 marks) clinical picture (6 marks) and complications of ulcerative colitis (5 marks) 2. A) A male diabetic patient 30 years-old who received his usual insulin therapy in the morning but he neglected to take his breakfast, short time later , he got blurring of vision, irritability, palpitation, sweating then passed into coma. 1. Which of the followings is your provisional diagnosis?(2 marks) and explain(3 marks) a- hypoglycemic coma b- Diabetic ketoacidosis coma c- Lactic acidosis coma d- Hyper-osmolar non ketotic coma 2. How can you prove your diagnosis? (2 marks) and how to treat him (3 marks) 3. Enumerate 5 indications of insulin therapy(5 marks) B) Enumerate: 1. 5 Clinical presentations of pheochromocytoma (5 marks) 2. 5 investigations of Cushing 'syndrome (5 marks) 3. 5 causes of Hirsutism (5 marks), C) Describe clinical picture (6 marks) and investigations of hypothyroidism (4 marks) 3. A) A 44 years-old women presented to the clinic because her fingers began to feel puffy and tight. She has noted loss of grip strength over the last 4 months and now cannot fully extend her hands. Over the last 10 years, she experienced colour changes of her hands on exposure to cold. She also suffers from heart burn especially on lying flat and she is using omeprazole to control it 1. Which of the followings is the most likely diagnosis? (2 marks) and explain (3 marks) a- Dermatomyositis b- Scleroderma c- Osteoarthritis d- Gout 2. Describe investigations of this case. (5 marks) 3. Describe treatment of this case. (5 marks) B) Describe 5 diagnostic criteria of Sjogren's syndrome (5 marks) C) Enumerate 5 disease modifying anti-rheumatic Drugs (DMARDs) and describe 2 side Effects of each drug (10 marks)

17


4. A) Describe differential diagnosis of ejection systolic murmur at the base of the heart. (10 marks) B) Describe clinical manifestations and management of infective endocarditis. (10 marks) 5. A) Meningitis: Causes, clinical and laboratory diagnosis, complications and prevention. (12 marks) B) Give an account on: 1- Definition and causes of dysentery. (3 marks) 2- Causes of fever with lymphadenopathy. (3 marks) 3- Causes of non-cirrhotic ascites. (2 marks)

Good Luck

18


Tanta University Faculty of Medicine Time allowed three hours

M.B.B.Ch. Internal Medicine Exam. Fifth Year Exam. Paper II 4/11/2015

All questions must be answered (5questions) – 150 marks 1. A) A 50 year old male was known to have long standing hypertension on irregular treatment, presented with history of generalized fatigue over the last 6 months. Over the last 2 months, he complained of nausea and occasional vomiting and has developed itching. On examination BP 170/110, skin is dry and cardiac examination revealed a systolic murmur. Creatinine, 9mg/dl, urea 80mg/dl, Ca 7.5mg/dl (reference range, 8.5-10 mg/dl), Phosphate is 6.5mg/dl (reference range 3.5-4.5mg/dl). Hb 8gm/dl, WBCs 5x103 ,platelets 190x103/dl 1. Which of the followings is your provisional diagnosis? (2 marks) and explain (3 marks) A- Acute kidney injury B- Chronic kidney disease C- Acute pyelonephritis D- Acute glomerulonephritis 2. How can you manage anemia in this case (5 marks)? 3. Discuss treatment strategies for this patient (5 marks)? B) Define acute kidney injury and describe 5 pre-renal causes of it. C) Describe 5 of: 1. Clinical manifestations of metabolic acidosis. (5 marks) 2. Investigations of reflux nephropathy. (5 marks) 3. Causes of hypokalemia. (5 marks)

(10 marks)

2. Enumerate: 1. 5 Complications of chronic cough. (5 marks) 2. 5 Causes of sinus bradycardia. (5 marks) 3. 5 causes of generalized lymphadenopathy. (5 marks) 4. 5 Contraindications to oral anticoagulant therapy. (5 marks) 3. A) 27 years-old women presented with episodes of epistaxsis and menorrhagia. She has diffused petechial hemorrhage scattered all over the body and bruises on both right and left lower limbs. On examination: no organomegaly, no lymphadenopathy. Hemoglobin was 10.5 gm. /dl, WBCs, 6x103/dl and platelets 25x103/dl. 1. Which of the followings is your provisional diagnosis? (2 marks) and explain. (3 marks) A-Acute myeloid leukemia B- Idiopathic thrombocytopenic purpura C-Hemophilia D-Aplastic anemia 2. Describe prognosis and bone marrow picture of the most probable diagnosis. (4 marks) 3. Enumerate causes of non-thrombocytopenic purpura (6 marks) B) Enumerate 5 causes (5 marks) describe 5 investigations for disseminated intravascular Coagulation. (5 marks) C) Describe: 1. 5 Hematological and biochemical findings of Hodgkin 'disease (5 marks) 2. 5 complications of massive blood transfusion. (5 marks) 3. 2 non-hematological manifestations and 3 investigations for pernicious anemia (5 marks) Question N.4 and N. 5 are in the back of the paper

19


4. A)Describe risk factors (3 marks) , types (3 marks) and diagnosis of bronchogenic Carcinoma. (4 marks) B) Enumerate: 1. Investigations of pulmonary embolism. (3 marks) 2. Complications of pneumonia. (4 marks) 3. Causes of bronchiectasis. (3 marks) 5. A) Causes, clinical picture and treatment of dissociative disorders. (15 marks) B) Definition, etiology, clinical picture, investigations and treatment of Guillain-Barre' Syndrome. (15 marks)

Good Luck

20


Tanta University Faculty of Medicine Time allowed three hours

M.B.B.Ch. Internal Medicine Exam. Fifth Year Exam. Paper III 9/11/2015

All questions must be answered (5 questions) – 150 marks

1. (A) Write short notes on (15marks) (5 marks /question) 1. Causes of acute chest pain. 2. Definition and complications of hypertension 3. Definition and causes of core pulmonale (B) Write short notes on: (15 marks) (5 marks /question) 1. Chest causes of acute dyspnea. 2. Causes of malignant pleural effusion. 3. Bronchodilators (C) Enumerate 5 causes of the followings: (15 marks) (5 marks /question) 1. Loss of weight 2. Cholestatic jaundice 3. Dry mouth. (D) Write short notes on: (15 marks) (5 marks /question 1. Chemoprophylaxsis of malaria. (5 marks) 2. Causes of acute diarrhea. (5 marks) 3. Clinical and laboratory diagnosis of HIV. (5 marks)

2. Physical medicine (A) Recent criteria for classification of rheumatoid arthritis. (5 marks) (B) Classification of osteoporosis and its imaging studies. (5marks) (C) Physical management of Bell's palsy. (5 marks)

3. Dermatology (A) (B) (C) (D) (E)

Mention topical treatment of acne vulgaris. (5 marks) Give an account on treatment of scabies. (5 marks) Enumerate causes of cicatricial alopecia. (5 marks) Enumerate clinical variants of psoriasis. (5 marks) Choose the correct answer: (10 marks)

21


1. All are viral skin diseases except: a- Molluscum contagiosum c- Erythema b- Herpes simplex. d- Verruca vulgaris 2. Acquired immune deficiency syndrome (AIDS) characterized by the following except: a- It is sexually transmissible retrovirus b- The virus is attacking the immune system. c- ELISA is very sensitive screening test. d- It is treated by reverse transcriptase stimulant. 3. Favus is causes by: a- Microsporum canis. c- Trichophyton violaceum. b- Trichophyton schoenleinii. d- Corynebacterium minutissimum. 4. All are trot concerning type II Lepra reaction except: a- It occurs predominantly in lepromatous leprosy. b- It b due to antigen antibody combination. c- Systemic corticosteroid has no role in its management. d- It develops systemic manifestations. 5. Atopic dermatitis is characterized by all the followings except: a- It is related to type I hypersensitivity reaction. b- It occurs in genetically predisposed persons. c- It is either irritant or allergic dermatitis. d- Topical immunomodulation are effective. 6. Initial lesion of pityriasis rosea is: a- Burrow. c- Target lesion b- Herald patch. d- Wheal. 7. Koebner phenomenon is characteristic for the following diseases except: a- Verruca plana. c- Pityriasis alba b- Lichen planus. d- Vitiligo. 8. Chancroid is caused by: a- Treponema pallidum. c- Staphylococcus aureus b- Haemophilus ducreyi. d- Streptococcus pyogens 9. Early secondary stage of syphilis is characterized by the following except: a- Generalized constitutional manifestations. b- Moth-eaten alopecia. c- Generalized lymphadenopathy. d- Mucous membrane involvement. 10. Differential diagnose of leucoderma includes the following except: a- Vitiligo. c- Lichen planus. b- Psoriasis. d- Pityriasis versieolor.

22


4. Clinical pathology (Choose the correct answer): (30 marks) 1. Stimulation of erythropoietin is caused by: a- Tissue hypoxia. b- Hypervolemia. c- Inflammation. d- Infection. 2. Acquired aplastic anemia is caused by: a- Benzene or benzene derivatives. b- Ionizing radiation. c- Purine or pyrimidine analogues. d- All of the above. 3. In iron deficiency anemia, the erythrocytic indices are typically: a- MCV increased, MCH decreased & MCHC decreased. b- MCV decreased, MCH decreased & MCHC decreased. c- MCV decreased, MCH increased & MCHC decreased. d- MCV decreased, MCH decreased & MCHC normal. 4. Anemia of inflammation can result from: a- Inappropriately decreased erythropoietin. b- Suppression of erythropoiesis by cytokines from activated macrophages & lymphocytes. c- Impaired iron metabolism. d- All of the above. 5. In case of classic pernicious anemia, the patient has: a- Leukopenia. b- Hypersegmented neutrophils. c- Anemia. d- All of the above. 6. Acquired hemolytic anemia can be caused by: a- Chemicals or drugs. b- Infectious organism. c- Antibody reaction. d- All of the above. 7. Paroxysmal nocturnal hemoglobinuria exhibits sensitivity of one population of red cells: a- Warm antibody. b- Cold antibody. c- Complement. d- Either A or B. 8. In sickle cell disease, the abnormality is related to: a- The rate of synthesis of hemoglobin. b- An abnormal molecular structure of hemoglobin. c- An acquired defect. d- A membrane dysfunction. 9. Thalassemia are characterized by: a- Abnormal hemoglobin molecules. b- Defective Îą-globulin structure. c- Abnormality in the rate of synthesis. d- Skeletal membrane defects.

23


10. The major function of neutrophilic granulocytes is: a- Antibody production. b- Destruction of parasites. c- Phagocytosis. d- Suppression of inflammation. 11. Characteristics of FAB M1 includes: a- Leukocytosis with maturation of the myelocytic cell line in the peripheral blood. b- Leukocytosis with maturation of the lymphocytic cell line in the peripheral blood. c- Leukocytosis without maturation of the myelocytic cell line in the peripheral blood. d- Leukocytosis with many mature leukocytes in the peripheral blood. 12. In chronic myelocytic leukemia, the total leukocyte (WBC) count is: a- Extremely increased. b- Slightly increased. c- Usually normal. d- Slightly decreased. 13. Disseminated intravascular coagulation (DIC) is characterized by: a- Microvascular thrombosis. b- Fibrin deposition. c- Active fibrinolysis. d- All of the above. 14. Prothrombin time: a- Monitor oral anticoagulant therapy & screens for one or more deficiencies in the extrinsic pathway. b- Increased in hemophilia A. c- Monitors heparin therapy. d- Is increased in thrombocytopenia. 15. The following are causes of thrombocytopenia except: a- Aplastic anemia. b- Acute leukemia. c- Quinidine therapy. d- Hemophilia A. 16. Polyuria occurs in: a- Diabetes mellitus. b- Diabetes insipidus. c- Excess coffee drink. d- All of the above. 17. Which of these is not ketone body: a- Acetone. b- Acetoacetic acid. c- Butyric acid. d- β-hydroxy butyric acid. 18. Hypertension is found in all of the following endocrinal diseases except: a- Cushing's syndrome. b- Pheochromocytoma. c- Adrenal medulla hyperplasia. d- Addisson's disease.

24


19. Bile duct obstruction can be diagnosed by: a- AST. b- T. Bilirubin. c- Bilirubin in urine. d- None of the above. 20. Vit K administration correct prolonged prothrombin time in a- Liver cirrhosis. b- Hemolytic anemia c- Obstructive jaundice d- All of the above 21. Elevation of tissue enzymes in serum may be used to detect: a- Presence of toxins. b- Infectious diseases. c- Tissue necrosis or damage. d- Diabetes mellitus. 22. C-peptide is of value in all EXCEPT: a- Measuring exogenous insulin. b- Measuring B- cell function. c- Measuring endogenous insulin. d- In fasting hypoglycemia. 23. A factor, other than average plasma glucose values, that determines the glycosylated hemoglobin level is: a- Serum ketone bodies level. b- Red blood cell life spam c- Ascorbic acid intake. d- Increased triglyceride levels. 24. In Nephrotic syndrome: a- Increase serum lipid b- Decrease serum albumin c- Increase urinary protein d- All of the above 25. Alfa feto-protein is a tumor maker of: a- Cancer breast b- Cancer pancreas c- Hepatocellular carcinoma d- Lymphoma . 26. The plasma protein mainly responsible for maintained colloidal pressure in vivo is: a- Hemoglobin b- Fibrinogen c- Alfa 2 macroglobulin d- Albumin 27. Lag oral glucose tolerance curve occurs in the flowing: a- Diabetes mellitus. b- After gasterctomy. c- In hypothyroidism. d- In malabsorption.

25


28. The isoenzyme of LDH (LD1 and LD2) are elevated in: a- Pulmonary embolism. b- Liver disease. c- Renal disease. d- Myocardial infarction. 29. Hypothyroidism is generally associated with all of the flowing Except: a- Weight gain. b- An elevation of TSH levels. c- Low free T3 and Free T4 levels. d- Decrease cholesterol level. 30. Pre -renal azotemia is cause by: a- Congestive heart failure. b- Chronic renal failure. c- Renal tumors d- Glomerulonephritis.

26


5.

Radiology (Choose the correct answer): (15 marks)

1. Non-ionizing imaging techniques Include: a- Ultrasonography. b- Magnetic Resonance Imaging (MRI). c- X-Rays. d- Isotope scans. e- A & B. 2. Downward K outward displacement of the cardiac apex in FA Chest & Heart x-ray is produced by the following: a- Left ventricular dilatation. b- Right ventricular dilatation. c- Enlarged left atrial appendage. d- Enlarged right atrium. e- All of the above. 3. Signs of right ventricular dilatation in Chest & Heart X-Rays include: a- Downward & outward displacement of the cardiac apex. b- Outward displacement of the cardiac apex. c- Obliteration of the retrosternal air space in the lateral view. d- B & C. e- All of the above. 4. Signs of left ventricular failure in PA Chest & Heart X-Ray include: a- Left atrial & left ventricular dilatation. b- Pulmonary venous congestion & interstitial edema. c- Clear lung fields. d- All of the above. e- A & B. 5. Simple pleural effusion in PA Chest X-Ray is seen as: a- Obliteration of the costophrenic angle. b- Homogenous density rising to the axilla. c- Redistribution in the supine position. d- All of the above. e- A & B. 6. The following conditions are associated with mediastinal shift in Chest X-Rays: a- Pulmonary TB. b- Tension pneumothorax. c- Pulmonary collapse. d- All of the above. e- A & C. 7. Colonic obstruction in standing abdominal x-rays is- seen: a- Multiple air fluid levels in the small & large bowel loops. b- Presence of air or fecal matter at the rectum. c- Air under the diaphragm. d- Radio-opaque shadows. e- A & B.

27


8. Injection of IV contrast is contraindicated in: a- Pregnancy b- Impaired renal function c- Hypersensitivity d- All of the above e- A & C. 9. Achalasia of the esophagus appears in barium swallow as: a- Marked esophageal dilatation. b- Very short stricture segment. c- Shouldering at the margins. d- All of the above. e- A & B. 10. The diaphragmatic hernias include: a- Morgagni hernia. b- Hiatus hernia. c- Bockdalek hernia. d- Rupture Diaphragm. e- All of the above. 11. Double contrast studies of the colon are usually performed by: a- Barium sulphate solution and air after evacuation of the barium. b- Barium and water-soluble oral contrast material. c- Both of the above. d- Water soluble oral contrast materials. e- All of the above. 12. Stricture at the sigmoid colon with luminal narrowing in Barium Enema (Apple core appearance) occurs in: a- Acute colitis. b- Colonic carcinoma. c- Crohn’s disease. d- A & C. e- All of the above. 13. A sclerotic bony metastatic lesion in a plain x-ray for the urinary tract occurs in patients with: a- Benign enlargement of the prostate. b- Prostatic carcinoma. c- Urinary bladder carcinoma d- All of the above. e- B & C. 14. Hydrocalycosis & Hydropelvis without ureteric dilatation in IVUs is caused by: a- Stricture lower end of the ureter. b- Vesicoureteric reflux disease. c- Peiviureteric junction obstruction. d- A & B. e- B & C. 15. Unilateral mediastinal lymphadenopathy is caused by: a- Hodgkin’s disease. b- Non-Hodgkin’s lymphoma. c- Metastatic lymph node enlargement in bronchogenic carcinoma. d- All of the above. e- A & C.

28


Tanta University Faculty of Medicine Time allowed three hours

M.B.B.Ch. Internal Medicine Exam. Fifth Year Exam. Paper I September 2014

All questions must be answered (4 questions) – 150 marks 1. A) A 45 year old female was referred to the outpatient clinic with 8 months history of pruritis. She had felt generally tired and she also complains that her eyes seem dry and sore, there was no loss of weight or appetite, no abdominal pain and no history of drug intake. On examination: She was (jaundiced) and had (clubbing) of her fingers. There was no lymphadenopathy or glandular enlargement. The skin was dry and pigmented and she had scratch marks all over her body with bilateral xanthelasmas. Abdominal examination revealed hepatomegaly and the spleen was 2 fingers below the costal margin. There was also shifting dullness. Investigations: Albumin 2 g/dL, total bilirubin 3.8 mg/dL, direct bilirubin 3.0 mg/dL, alkaline phosphatase 640 U/L, pothrombin time is 15 seconds. Hepatitis viral markers and anti-nuclear antibodies are negative. 1. What is the probable diagnosis and explain? (5 marks) 2. What further investigations would you like to perform? (5 marks) 3. What is the treatment? (5 marks) B) Describe aetiology, clinical picture, and investigations of ulcerative colitis. (10 marks) C) Enumerate: 1. 5 risk factors for gastric cancer. (5 marks) 2. 5 investigations for diagnosis of acute pancreatitis. (5 marks) 3. 5 clinical manifestations of Wilson's disease. (5 marks) 2. A) Describe aetiology, clinical picture and diagnosis of pan-hypopituitarism. (15 marks) B) Describe definition, clinical features and investigations of Conn's syndrome. (10 marks) C) Enumerate: 1. 5 chronic complications of diabetes mellitus. (5 marks) 2. 5 causes of hyperphosphatemia. (5 marks) 3. 5 diagnostic tests for thyrotoxicosis. (5 marks) 3. A) Describe definition, types, and complications of systemic hypertension. B) Enumerate: 1. 5 early complications of acute myocardial infarction. (5 marks) 2. 5 causes of aortic regurgitation. (5 marks) 3. 5 investigations for heart failure (5 marks) 4. A) Describe clinical picture, diagnosis and treatment of amoebic liver abscess. (15 marks) B) Give short notes on diagnosis and treatment of: 1. Cerebral malaria. (5 marks) 2. Brucellosis. (5 marks) 3. Intestinal ascariasis. (5 marks) 4. Schistosoma mansoni. (5 marks)

29


Tanta University Faculty of Medicine Time allowed three hours

M.B.B.Ch. Internal Medicine Exam. Fifth Year Exam. Paper II September 2014

All questions must be answered (4 questions) – 150 marks 1. A) A 29-year-old man presented with spontaneous bruising of his legs and arms. He had three recent attacks of epistaxis but no other bleeding. He was not taking any drugs and there were no physical signs apart from (bruises) and scattered petechiae on his legs. The spleen was not palpable. Investigations: haemoglobin 13gm/dl, white cell count 5,400 per mm3, platelet count 10,000 per mm3, ESR 6 mm h1st and direct Coombs test was negative. Antinuclear and DNA-binding antibodies and rheumatoid factor were absent. Bone marrow contained an increased number of normal megakaryocytes but was otherwise normal. 1. What is the probable diagnosis and explain? (5 marks) 2. What is the treatment for this patient? (5 marks) B) Mention definition and describe clinical picture and investigations of chronic myeloid Leukaemia. (15 marks) C) Enumerate: 1. 5 contraindications of oral anticoagulant therapy. (5 marks) 2. 5 complications of sickle cell anemia. (5 marks) 3. 5 causes of neutropenia. (5 marks) 2. A) Describe causes, clinical picture and investigations of acute tubular necrosis. (15 marks). B) Mention definition, aetiology and stages of chronic kidney disease. (10 marks) C) Enumerate: 1. 5 complications of haemodialysis. (5 marks) 2. 5 non glomerular causes of haematuria. (5 marks) 3. 5 investigations to diagnose reflux nephropathy. (5 marks) 3. A) Discuss aetiology, clinical pictures, investigations and treatment of pleural effusion. (20 marks) B) Discuss aetiogenesis of bronchogenic carcinoma, (7 marks) C) Mention side effects of anti tuberculous drugs. (8 marks) 4. A) Describe Clinical picture and treatment of Parkinson's disease. (10 marks) B) Mention etiologies of hemiplegia. (10 marks) C) Enumerate types of schizophrenia and its treatment. (8 marks) D) Enumerate indications of electroconvulsive therapy (ECT). (7 marks)

30


Tanta University Faculty of Medicine Time allowed three hours

M.B.B.Ch. Internal Medicine Exam. Fifth Year Exam. Paper III September 2014

All questions must be answered (6 questions) – 150 marks 1. A) Mention definition and describe clinical picture, investigations of dermatomyositis. (20 marks) B) Enumerate: 1. 5 common features of seronegative spondyloarthropathies. (5 marks) 2. 5 serological investigations of for scleroderma. 3. 5 non immunological causes of arthropathy. 2. Enumerate: 1. 5 causes of loss of weight. (5 marks) 2. 5 extra-hepatic manifestations of chronic hepatitis. (5 marks) 3. 5 indications of insulin therapy. (5 marks) 4. 5 cardiac causes of chest pain. (5 marks) 5. 5 side effects of thiazides diuretic therapy. (5 marks) 3. Physical medicine: Describe clinical and radiological manifestations of ankylosing spondylitis. 4. Dermatology: A) Causes of cicatricial alopecia. (5 marks) B) Treatment of chronic generalized psoriasis. (5 marks) C) Management of chancre. (10 marks) D) Choose only one answer for each question. (10 marks)  MCQ not included. 5. Clinical pathology (30 Marks)  MCQ not included. 6. Radiology (15 Marks)  MCQ not included.

31


Tanta University Faculty of Medicine Time allowed three hours

M.B.B.Ch. Internal Medicine Exam. Fifth Year Exam. Paper I December 2014

All questions must be answered (4 questions) – 150 marks 1. A) A 15 year old boy presented, to the emergency room with, abdominal pain, repeated vomiting not responding to antiemetic drugs for the last_2 days. His mother noticed him with polyuria and polydipsia in the last month. On examination he was conscious, irritable, dehydrated, his pulse was 110 beat/min, BP. was 90/60 mmHg. Hb 13 gm/dl blood urea 75 mg/dl, arterial PH 6.9, HCO3 12 mmol/1, serum Na 135 mmol/L Random blood glucose was 500mg/dl C 1. What is the probable diagnosis and explain? (5 marks) 2. What further investigations would you like to perform? (3 marks) 3. What is the treatment? (7 marks) B) Describe causes and clinical picture of Cushing's syndrome. (10 marks) C) Enumerate: 1. 5 causes of hypothyroidism. (5 marks) 2. 5 investigations for diagnosis of acromegaly. (5 marks) 3. 5 clinical manifestations of hypocalcaemia. (5 marks) 2. A) Describe clinical picture, investigations and treatment of achalasia of the esophagus. (15 marks) B) Describe causes and clinical manifestations of fulminant hepatic failure. (10 marks) C) Enumerate: 1. 5 complications of peptic ulcer. (5 marks) 2. 5 causes of chronic diarrhoea. (5 marks) 3. 5 clinical manifestations of chronic pancreatitis. (5 marks) 3. A) Describe management of patient with acute cardiac chest pain in emergency room. (20 marks) B) Enumerate: 1. 5 clinical manifestations of mitral stenosis (5 marks) 2. 5 causes of left sided heart failure (5 marks) 3. 5 investigations for acute rheumatic fever (5 marks) 4. A) Describe clinical picture, investigations and treatment of typhoid fever. (20 marks) B) Give short notes on investigations and treatment of: 1. Toxoplasmosis. (7 marks) 2. Meningitis. (8 marks)

32


Tanta University Faculty of Medicine Time allowed three hours

M.B.B.Ch. Internal Medicine Exam. Fifth Year Exam. Paper II December 2014

All questions must be answered (4 questions) – 150 marks 1. A) A 58 years male presented with pitting oedema in extremities and puffiness of the eye lids. He has a history of work related heavy metals exposure in the past with no previous history of diabetes cm hypertension. Examination revealed BP is 140/80 mmHg. Lab tests revealed proteinuria 5.0g/24 hours, / total cholesterol 350 mg/dL, triglycerides 370 mg/dL, LDL cholesterol 210 mg/dL and serum albumin 2.6 gm/dL. Hepatitis markers are negative. 1. What is the most likely diagnosis? (3 marks) and explain (3 marks) 2. What are further investigations needed to identify the aetiology? (4 marks) 3. What is the treatment of this condition? (5 marks) B) Describe causes and diagnosis of chronic interstitial nephritis. (10 marks) C) Enumerate: 1. 5 causes of hypernatremia. (5 marks) 2. 5 Causes of renal anemia. (5 marks) 3. 5 stages of vesicoureteric reflux. (5 marks) 2. A) Describe definition, clinical pictures and complications of Polycythemia rubra Vera. (15 marks) B) Describe investigations and treatment of Vitamin B12 deficiency. (10 marks) C) Enumerate: 1. 5 causes of thrombocytosis. (5 marks) 2. 5 hazards of blood donation. (5 marks) 3. 5 investigations for acute myeloid leukemia. (5 marks) 3. A) Describe types, clinical-picture and treatment of pneumothorax. (20 marks) B) Give an account on bronchodilators. (7 marks) C) Describe treatment of COPD. (8 marks) 4. A) Describe definition, clinical picture and treatment of generalized anxiety disorders. (20 marks) B) Parkinsonism: definition, etiology and treatment of Parkinson's disease (15 marks)

33


Tanta University Faculty of Medicine Time allowed three hours

M.B.B.Ch. Internal Medicine Exam. Fifth Year Exam. Paper III December 2014

All questions must be answered (6 questions) – 150 marks 1. A) Mention diagnostic criteria, investigations and complications of Sjogren's syndrome. (15 marks) B) Enumerate: 1. 5 Musculoskeletal manifestations of rheumatoid arthritis. (5 marks) 2. 5 Serological and immunological tests for diagnosis of SLE. (5 marks) 2. Enumerate: 1. 5 causes, of cholestatic jaundice, (5 marks) 2. 5 diagnostic criteria of metabolic syndrome. (5 marks) 3. 5 causes of functional proteinuria. (5 marks) 4. 5 investigations for disseminated intravascular coagulation. (5 marks) 5. 5 causes of localized oedema. (5 marks) 3. Physical medicine: Describe types of juvenile idiopathic arthropathy and its management. (15 marks). 4. Dermatology: A) Clinical types and treatment of herpes zoster. (10 mark) B) Primary lesion and complications of lichen planus. (5 marks) C) Clinical manifestations of gonorrhea in female. (5 marks) D) Choose only one answer for each question. (10 marks)  MCQ not included. 5. Clinical pathology (30 Marks)  MCQ not included. 6. Radiology (15 Marks)  MCQ not included.

34


Tanta University Faculty of Medicine Time allowed three hours

M.B.B.Ch. Internal Medicine Exam. Fifth Year Exam. Paper I August 2013

All questions must be answered (4 questions) – 150 marks 1. A) A 19-y-old male patient presented to the outpatient clinic with 4 day^ history of fever, anorexia, vomiting, right upper quadrant pain, myalgia associated with right knee arthralgia. Examination revealed temperature 38c°, pulse 110/min, ABP 90/70 mm hg, icterus, tender hepatomegaly, but no shifting dullness or other organomegaly. There was normal blood count, markedly elevated transaminases, elevated serum bilirubin (8 times the ULN) but normal serum albumin. 1. What is the most likely diagnosis? (5 marks) 2. What further investigations needed to confirm the diagnosis? (5 marks) 3. What are the most possible complications? (10 marks) B) Mention the definition, clinical picture, diagnosis and treatment of achalasia of the cardia. (10 marks) C) Enumerate: 1. 5 most common causes of upper gastrointestinal bleeding in Egypt (5 marks) 2. 5 most common complications of acute pancreatitis (5 marks) 2. A) Describe the etiology, clinical picture, diagnosis and treatment of acromegaly. (15 marks) B) Describe the etiology, clinical features, diagnosis and treatment of primary adrenocortical insufficiency (15 marks) C) Enumerate: 1. 5 components of the metabolic syndrome. (5 marks) 2. Oral antidiabetic drugs in 2013. (5 marks) 3. A) Describe the different complications of acute myocardial infarction. (20 marks) B) Enumerate: 1. Complications of rheumatic mitral stenosis (5 marks) 2. Peripheral signs of aortic regurge (5 marks) 3. Non pharmacologic therapies of hypertension (5 marks) 4. A) Mention the definition, causes and diagnostic approach of pyrexia of unknown origin. (15 marks) B) Give short notes on diagnosis and treatment of: 1. Giardiasis (5 marks) 2. Oxyuris infection (5 marks) 3. Enteric fever (5 marks) 4. Mennigeococcal meningitis (5 marks)

35


Tanta University Faculty of Medicine Time allowed three hours

M.B.B.Ch. Internal Medicine Exam. Fifth Year Exam. Paper II August 2013

All questions must be answered (4 questions) – 150 marks 1. A) A 22-y-oId male presented to the outpatient clinic with fatigue, dyspnoea; bleeding gums and recurrent fever over the last 2 months. Examination revealed; fever 38°c, purpuric eruption all over the body including inner cheeks but no organomegaly or lymphadenopathy. His haemoglobin was 5.1 gm/dl, RBCs 1.9x106/dl, leucocytic count 2xl03/dl, platelets 25xl03/dl and the reticulocytes were 0.01% at that time. 1. What is the most likely diagnosis? (5 marks) 2. What is your next diagnostic confirmatory test? (5 marks) 3. What are the different therapeutic modalities? (5 marks) B) Mention the definition clinical features and treatment of essential thrombocytopenia (10 marks) C) Enumerate: 1. 5 Causes of thrombophilia. (5 marks) 2. 5 components of hypersplenism. (5 marks). 3. Antithrombotic therapies. (5 marks) 2. A) Mention the indications, advantages and disadvantages of renal replacement therapy. (15 marks) B) Describe the clinical investigations and treatment of metabolic acidosis. (15 marks) C) Enumerate: 1. Manifestations of nephrotic syndrome (5 marks) 2. Different modalities to control microalbuminuria (5 marks) 3. A) Describe the etiology, clinical picture, diagnosis and treatment of community acquired pneumonia (20 marks) B) Diagnosis of pulmonary embolism. (7 marks) C) Antituberculous drugs. (8 marks) 4. A) Mention the etiology, clinical picture and differential diagnosis of hemiplegia. (20 marks) B) Describe the clinical picture and management of major depressive disorder. (15 marks)

36


Tanta University Faculty of Medicine Time allowed three hours

M.B.B.Ch. Internal Medicine Exam. Fifth Year Exam. Paper III August 2013

All questions must be answered (4 questions) – 150 marks 1. A) Definition, pathogenesis, clinical features, diagnosis and treatment of polyarteritis nodosa. (15 marks) B) Enumerate: 1. 5 Musculoskeletal manifestations of rheumatoid arthritis. (5 marks) 2. 5 Serological and immunological tests for diagnosis of SLE. (5 marks) 2. Mention briefly medical diseases that cause the following: 1. Diagnostic criteria of rheumatoid arthritis (5 marks) 2. Components of the CREST syndrome (5 marks) 3. Skin manifestations of systemic lupus erythematosus (5 marks) 4. Therapeutic modalities of acute gouty arthritis (5 marks) 3. Physical medicine: Describe the etiopathogenesis and treatment of osteoarthritis (15 marks). 4. Dermatology: A) Describe the clinical features of tuberculoid leprosy (5 marks) B) Enumerate the differential diagnosis of a circinate skin lesion (5 marks) C) Characterize the clinical manifestations of secondary stage of syphilis (10 marks) D) Choose only one answer for each question. (10 marks)  MCQ not included. 5. Clinical pathology (30 Marks)  MCQ not included. 6. Radiology (15 Marks)  MCQ not included.

37


Tanta University Faculty of Medicine Time allowed three hours

M.B.B.Ch. Internal Medicine Exam. Fifth Year Exam. Paper I November 2013

All questions must be answered (4 questions) – 150 marks 1. A) A 45-y-old female patient presented to the outpatient clinic with increasing darkening of e skin, easy fatigability, nausea, and occasional vomiting and weight loss over 8 months prior to presentation. Physical examination revealed hyperpigmentation on the fade, oral mucosa, and palmar creases. BP: 100/60 mmHg supine and 70/40 mmHg sitting. Apex beat is normal. All other systems are essentially normal. Laboratory investigations: haemoglobin 10gm%, fasting blood sugar 67 mg%, serum Na is 132 mEq/l 1. What is the most likely diagnosis? (5 marks) 2. What further investigations needed to confirm the diagnosis? (10 marks) 3. What is the treatment for this patient? (5 marks) B) Mention the definition, clinical picture, diagnosis and treatment of diabetes insipidus. (10 marks) C) Enumerate: 1. Vascular complications of diabetes mellitus. (5 marks) 2. 5 clinical manifestations of pheochromocytoma . (5 marks) 2. A) Describe the etiology, clinical picture, diagnosis and treatment of gastroesophageal reflux disease. (15 marks) B) Describe the classification, clinical features, serological diagnosis and treatment of autoimmune hepatitis (15 marks) C) Enumerate: 1. 5 components of the metabolic syndrome. (5 marks) 2. Oral antidiabetic drugs in 2013. (5 marks) 3. A) Mention the definition, clinical types, investigations and treatment of angina. (20 marks) B) Enumerate: 1. Causes of atrial fibrillation. (5 marks) 2. Modifiable risk factors for atherosclerosis. (5 marks). 3. Complications of infective endocarditis. (5 marks) 4. A) Describe the pathology, clinical picture, investigations and treatment of amoebic liver abscess. (20 marks) B) Give short notes on diagnosis and treatment of: 1. Diagnosis and treatment of Scistosomal mansoni infection. (5 marks) 2. Complications of malaria. (5 marks) 3. Diagnosis of HIV (human immune deficiency virus) infection. (5 marks]

38


Tanta University Faculty of Medicine Time allowed three hours

M.B.B.Ch. Internal Medicine Exam. Fifth Year Exam. Paper II November 2013

All questions must be answered (4 questions) – 150 marks 1. A) A 60-y-old female presented to the outpatient clinic with back pain, weakness and decreased appetite for one weak. On physical examination, there is pallor, limited movement on the lumbar spine, no focal neurological changes. Other systems are essentially normal. Her haemoglobin is 2.7 mg/dl, platelets 106 x103/dl, creatinine 2.7 mg/dl, Calcium is 16.6 mg/dl, ESR 1st h: 50, 2nd h: 70. 1. What is the most likely diagnosis? (5 marks) 2. What is your next diagnostic confirmatory test? (5 marks) 3. What are the different therapeutic modalities? (5 marks) B) Mention the definition, clinical features and treatment of pernicious anaemia. (10 marks) C) Enumerate 5 of: 1. Causes of thrombocytopenia (5 marks) 2. Complications of massive bipod transfusion (5 marks) Hematological findings in Hodgkin's disease (5 marks) 2. A) Mention the definition, etiology and clinical picture of acute interstitial nephritis. (15 marks) B) Mention the etiology, clinical features and treatment of metabolic alkalosis. (15 marks) C) Enumerate: 1. Causes of acute tubular necrosis (5 marks) 2. Investigations of chronic pyelonephritis (5 marks) 3. A) Describe the etiology, clinical picture, diagnosis and treatment of lung abscess. (20 marks) B) Non cardiac causes of chest pain. (7 marks). C) Management of pneumothorax. (8 marks) 4. A) Describe clinical picture and management of Guillain-Barre’ syndrome. (20 marks) B) Describe the clinical picture and management of panic disorder. (15 marks)

39


Tanta University Faculty of Medicine Time allowed three hours

M.B.B.Ch. Internal Medicine Exam. Fifth Year Exam. Paper III November 2013

All questions must be answered (4 questions) – 150 marks 1. A) Describe pathogenesis, Classification, clinical picture, and treatment of scleroderma. (20 marks) B) Mention clinical picture and treatment of Behcet syndrome. (15 marks) 2. Enumerate 5 of: 1. Extra-intestinal manifestations of inflammatory bowel diseases. (5 marks) 2. Renal complications of hypertension. (5 marks) 3. Non-chest presentations of bronchogenic carcinoma. (5 marks) 4. Causes of syncope. (5 marks) 5. Causes of dullness in the second left intercostal space. (5 marks) 3. Physical medicine: Describe musculoskeletal manifestations of systemic lupus erythematosus. (15 marks) 4. Dermatology: A) Give an account on clinical types of scabies. (5 marks) B) Enumerate topical therapy of psoriasis. (5 marks) C) Describe clinical features and diagnosis 61 primary syphilis (chancre). (10 marks) D) Choose only one answer for each question. (10 marks)  MCQ not included. 5. Clinical pathology (30 Marks)  MCQ not included. 6. Radiology (15 Marks)  MCQ not included.

40


Tanta University Faculty of Medicine Time allowed three hours

M.B.B.Ch. Internal Medicine Exam. Fifth Year Exam. Paper I August 2012

All questions must be answered (4 questions) – 150 marks 1. A) 3 weeks ago a 50-year-old diabetic, male with known compensated liver cirrhosis presented with fever during his regular follow-up visit in the outpatient clinic. One week after this visit the patient presented in the emergency department with persist ent fever, right upper quadrant pain and moderate ascites. His haemoglobin was 13.5 gm/dl, alpha-fetoprotein was 15 ng/ml, ascetic fluid leucocytic count was less than 10/Cmm, but he was HCV antibody +ve. 1. What is the most likely diagnosis? (5 marks) 2. What further investigations needed to confirm the diagnosis? (5 marks) 3. Mention the different modalities of therapy. (10 marks) B) Give an account on clinical picture and diagnosis of acute pancreatitis. (10 marks) C) Enumerate: 1. 5 most common causes of lower gastrointestinal bleeding in Egypt (5 marks) 2. 5 complications of peptic ulcer disease (5 marks) 2. A) Give an account on the etiology, clinical picture, and management of hypercalcaemia. (15 marks) B) Causes, types, clinical picture and treatment of diabetes insipidus. (15 marks) C) Enumerate: 1. Causes of hyperprolactinemia (5 marks) 2. Gouty syndromes (5 marks) 3. A) Give an account on causes, clinical picture, diagnosis and treatment of pericardial effusion. (20 marks) B) Give short notes on etiology, diagnosis and treatment of atrial fibrillation.(15 marks) 4. A) Definition causes and diagnostic approach of pyrexia of unknown origin (PUO) (20 marks) B) Give short notes on diagnosis and treatment of: 1. Cerebral malaria (5 marks) 2. Schistosomiasis mansoni (5 marks) 3. Ascariasis (5 marks)

41


Tanta University Faculty of Medicine Time allowed three hours

M.B.B.Ch. Internal Medicine Exam. Fifth Year Exam. Paper II August 2012

All questions must be answered (4 questions) – 150 marks 1. A) A 55-year-old male who is heavy smoker presented to the Emergency Department with epistaxis, increasing tiredness, throbbing headache, tinnitus and dizziness. On examination, he is mildly plethoric and hypertensive (BP = 170/110 mmHg). Haemoglobin 9.81 gm/dl White blood cell count 15600/cmm Platelets 441000/cmm 1. What is the most likely diagnosis? (5 marks) 2. What is your next diagnostic confirmatory test? (5 marks) 3. What are the different therapeutic options? (5 marks) B) Give an account on types and indications of anti-thrombotic therapy. (15 marks) C) Aetiology, clinical features, diagnosis and treatment of idiopathic thrombocytopenic purpura (ITP). (10 marks) 2. A) Skill, cardiovascular and neuropsychiatric manifestations of chronic renal failure (15 marks) B) Causes, clinical manifestations and treatment of hyponatraemia. (15 marks) C) Investigations and treatment of nephrotic syndrome. (10 marks) 3. A) Types, clinical features, diagnosis and treatment of pneumothorax. (20 marks) B) Aetiology, clinical picture, diagnosis and treatment of mycoplasma pneumonia. (5 marks). C) Clinical features, diagnosis and treatment of bronchial adenoma. (5 marks) 4. A) Clinical picture and treatment of Parkinson's disease. (20 marks) B) Clinical picture and treatment of obsessive compulsive disorder. (20 marks)

42


Tanta University Faculty of Medicine Time allowed three hours

M.B.B.Ch. Internal Medicine Exam. Fifth Year Exam. Paper III August 2012

All questions must be answered (4 questions) – 150 marks 1. A) Pathogenesis, clinical features, investigations and treatment of progressive systemic sclerosis (PSS) (20 marks) B) Mention the clinical features of: 1. Acute gouty arthritis (5 marks) 2. Behecet’s disease (5 marks) 3. Polyarteritis nodosa (5 marks) 2. Mention briefly medical disease that cause the following: 1. Clubbing of fingers (5 marks) 2. Somnolence (5 marks) 3. Hoarseness of voice (5 marks) 4. Nocturia (5 marks) 5. Asterixis (5 marks) 3. Physical medicine: Lines of treatment of spasticity (15 marks) 4. Dermatology: A) Scaly versus black dotted tinea capitis (cause and diagnosis). (10 marks) B) Treatment of acne vulgaris (enumerate only). (10 marks) C) Cutaneous manifestations and serological tests of AIDS. (10 marks) 5. Clinical pathology (30 Marks)  MCQ not included. 6. Radiology (15 Marks)  MCQ not included.

43


Tanta University Faculty of Medicine Time allowed three hours

M.B.B.Ch. Internal Medicine Exam. Fifth Year Exam. Paper I November 2012

All questions must be answered (4 questions) – 150 marks 1. A) A 25 - year - old female presented in the outpatient clinic with colicky post-prandial abdominal pain associated with fever (38°C) and mild diarrhoea (5 / day) sometimes associated with blood. The patient referred to similar attacks with 10 kg weight loss over the last year. Her haemoglobin was 9.3 gm/dl, ESR (1st hour 75 and 2nd hour 116) with numerous pus cells in stool abdominal ultrasound, revealed only a group of lymph nodes in the right iliac fossa. 1. What is the most likely diagnosis? (5 marks) 2. What further investigations needed to confirm the diagnosis? (5 marks) 3. What’s the ideal management? (90 marks) B) Describe the clinical features, diagnosis and treatment of chronic virus B Hepatitis. (15 marks) C) Enumerate: 1. Causes of oropharyngeal dysphagia (5 marks) 2. Complications of liver cirrhosis (5 marks) 2. A) Describe the etiology, clinical picture, and management of hypocalcaemia. (15 marks) B) Describe the clinical picture, diagnosis and treatment of primary aldosteronism. (15 marks) C) Enumerate: 1. Eye manifestations of thyroid disease (3 marks) 2. Skin manifestations of Grave’s disease (3 marks) 3. Types of diabetic neuropathy (4 marks) 3. A) Describe the different atherosclerotic risk factors and their possible management. (15 marks) B) Enumerate: 1. Complications of mitral valve stenosis (5 marks) 2. The big 5 causes of atrial fibrillation (5 marks) 3. Causes of surgically curable hypertension (5 marks) 4. Different types of cardiomyopathy (5 marks) 4. A) Describe the etiology, clinical picture, diagnosis and treatment of Brucellosis. (15 marks) B) Mention briefly the diagnosis and treatment of: 1. Black water fever (5 marks) 2. Schistosomal dysentery (5 marks) 3. Toxoplasmosis (5 marks) 4. Amoebic hepatitis (5 marks)

44


Tanta University Faculty of Medicine Time allowed three hours

M.B.B.Ch. Internal Medicine Exam. Fifth Year Exam. Paper II November 2012

All questions must be answered (4 questions) – 150 marks 1. A) A 40 - year - old male patient presented to the emergency department with epistaxis. The patient past medical history was remarkable for night fever, sweats and 8Kgs weight loss over the past 6 months. Examination revealed pallor, fever (38C) and splenomegaly (reaching the umbilicus). Haemoglobin 185 gm/dl WBCS marked leucocytosis with myelocytes & myeloblasts Platelets 38000/cmm 1. What is the most likely diagnosis? (5 marks) 2. What further investigations needed to confirm the diagnosis? (5 marks) 3. What are the different therapeutic options? (5 marks) B) Describe the etiology, clinical picture, diagnosis and treatment of aplastic anemia. (15 marks) C) Enumerate: 1. Acquired coagulopathies (5 marks) 2. Hazards of massive blood transfusion (5 marks) 2. A) Describe the gastrointestinal, hematologic and musculoskeletal manifestations of chronic renal failure. (15 marks) B) Mention the definition, etiology, clinical features and treatment of hypokalaemia. (15 marks) C) Enumerate: 1. Renal function tests (5 marks) 2. Complications of nephrotic syndrome. (5 marks) 3. A) Describe the clinical picture and investigations of bronchogenic carcinoma. (20 marks) B) Mention the definition, clinical characteristics and lines of treatment of acute severe asthma (status asthmaticus) (10 marks). 4. A) Mention the classification and management of different types of epilepsy. (20 marks) B) Describe the clinical picture and management of schizophrenia. (20 marks)

45


Tanta University Faculty of Medicine Time allowed three hours

M.B.B.Ch. Internal Medicine Exam. Fifth Year Exam. Paper I September 2011

All questions must be answered (5 questions) – 150 marks 1. A) A 25 year old female presents with recently discovered diabetes mellitus, she complains of months of weight gain, fatigue, amenorrhea and worsening acne. On examination, she is noted to have facial fullness, truncal obesity with purple striae across both flanks. Her blood pressure 150/95 mmHg, fasting blood glucose 250 mg/dl, serum sodium 150 mEq/L and serum potassium 3.3 mEq/L. 1. What is the likely diagnosis? (5 marks) 2. What laboratory tests and imaging would you order to reach proper diagnosis? (10 marks) B) Give an account on Diabetic Ketoacidosis, precipitating factors, clinical picture and management. (15 marks) C) Mention the management of Myxedema coma. (10 marks) 2. A) Give an account on etiology, clinical picture, complications and treatment of Chronic Pancreatitis. (15 marks) B) Mention the manifestations of Hepatocellular failure. (15 marks) C) Enumerate 5 causes of: 1. Acute gastritis. 2. Intrahepatic cholestasis. 3. Dysentery. 3. A) Mention the diagnostic criteria for Systemic Lupus Erythematosus. (15 marks) B) Enumerate: 1. Complications of mitral valve stenosis (5 marks) 2. Seronegative Spondyloarthropathies (SNSA). (5 marks) 3. Mention the common features of SNSA. (10 marks) 4. A) Give an account on causes, clinical picture and complications of Aortic Stenosis. (10 marks) B) Discuss definition, types and complications of systemic Hypertension. (10 marks) 5. A) Give an account on causes and diagnosis of Fever of unknown origin (FUO). (10 marks) B) Mention the management of bleeding Esophageal Varices. (10 marks)

46


Tanta University Faculty of Medicine Time allowed three hours

M.B.B.Ch. Internal Medicine Exam. Fifth Year Exam. Paper II September 2011

All questions must be answered (5 questions) – 150 marks 1. A) A 28 year old male has a 3 month history of general fatigue and easy bruising. On physical examination, temperature 38°C, blood pressure 110/70 mmHg. Purpuric eruptions distributed all over the body and ecchymoses are noted on the upper thighs. There is no lymphadenopathy or organomegaly. Laboratory studies: Hb 5.4gm/dL, RBCs 1.9x10 /dL, total leucocytic; count 2x103/dL, platelet count 25x103/dL and the reticulocytes is 0.2%. Bone marrow examination shows hypoplastic bone marrow with decreased bone marrow cellularity, no evidence of abnormal cells or infiltrative disease. 1. What is the most likely diagnosis? (5 marks) 2. What is the differential diagnosis of the case? (5 marks) 3. Mention the line of treatment of the case? (5 marks) B) Give a short account on manifestations, diagnosis and treatment of Haemophilia A. (10 marks) C) Enumerate 5 causes of: 1. Haemopoietic growth factors. (5 marks) 2. Steps to diagnose multiple myeloma. (5 marks) 3. Causes of non-thrombocytopenic purpura. (5 marks) 2. A) Give an account on etiology, clinical picture, investigations and treatment of Acute Pyelonephritis. (15 marks) B) Mention the conservative treatment of chronic kidney disease (CKD). (15 marks) C) Give a short account on causes and clinical picture of Metabolic Alkalosis. (10 marks) 3. Give an account on: 1. Causes, clinical types and picture, diagnosis and treatment of Pneumothorax. (10 marks) 2. Definition, etiology and clinical picture of Cor-pulmonale (10 marks)

4. Give an account on: 1. Etiology, clinical picture and diagnosis of Neuropathy. (15 marks) 2. Etiology, clinical picture and management of generalized anxiety disorder. (15 marks) 5. Give an account on: 1. Lymphadenopathy. (10 marks) 2. Bronchodilators.

(10 marks)

47


Tanta University Faculty of Medicine Time allowed three hours

M.B.B.Ch. Internal Medicine Exam. Fifth Year Exam. Paper III September 2011

All questions must be answered (6 questions) – 150 marks 1. (Chest) Give an account on: (15 marks) 1. Manifestations of life threatening asthma. 2. Paramalignant syndrome of bronchogenic carcinoma. 3. Causes of lung abscess. 2. (CVS) Mention short notes on: (15 marks) 1. Causes and clinical picture of infective endocarditis. 2. Causes and clinical picture of left ventricular failure. 3. Differential diagnosis of mid diastolic murmur. 3. (Neuro. & Psych.) Give an account on: (15 marks) 1. Diagnosis and differential diagnosis of tonic-clonic seizures. 2. Management of multiple sclerosis^ 3. Clinical picture of dementia. 4. (Tropical medicine) Give an account on: (15 marks) 1. Differential diagnosis of huge splenomegaly. 2. Malaria prophylaxis before travel. 3. Clinical manifestations of portal hypertension. 5. (Physical medicine) Mention briefly: (15 marks) 1. New criteria for diagnosis of rheumatoid arthritis. 2. Types and physiologic effects of thermotherapy. 3. Clinical manifestations of psoriatic arthritis. 6. (Dermatology) Discuss in brief: (15 marks) 1. Clinical and laboratory manifestations of 2ry stage Syphilis. 2. Clinical varieties and management of Tinea Capitis. 3. Clinical types and systemic treatment of Psoriasis.

48


Tanta University Faculty of Medicine Time allowed three hours

M.B.B.Ch. Internal Medicine Exam. Fifth Year Exam. Paper I September 2010

All questions must be answered (5 questions) – 150 marks 1. A) A 60 year old male presented with fatigue, malaise, anorexia with loss of 8 kg in the last few weeks. He noticed pale colour of stools and suffered from generalized itching. On exam: heart rate 68 b/min., blood pressure 130/75 mmHg., no fever. Icterus of his sclera and skin. The liver was felt 3 fingers below costal margin in midclavicular line, smooth, soft and not tender. No palpable spleen or ascites. Abdominal sonography revealed a distended gall bladder and dilated intrahepatic and cystic ducts. 1. What is the likely diagnosis? (5 marks) 2. What further investigations would to order? (5 marks) 3. What is the differential diagnosis of the case? (5 marks) B) Give an account on clinical picture, complications, diagnosis and treatment of Crohn’s disease. (15 marks) C) Mention diagnosis and treatment of Peptic ulcer. (10 marks) 2. A) Give an account on causes, clinical picture, investigations and treatment of Hypothyroidism. (15 marks) B) Mention oral Antidiabetic drugs. (15 marks) C) Enumerate 5 of: (10 marks) 1. Endocrine causes of hypertension 2. Causes of stunted growth 3. Causes of hypercalcaemia 4. Causes of secondary diabetes mellitus. 3. A) Discuss causes, clinical picture, diagnosis and treatment of Pleural Effusion. (10 marks) B) Mention the causes and clinical manifestations of Mediastinal Syndrome. (10 marks) 4. Give an account on: 1. Etiology, clinical picture and treatment of Migraine. (15 marks) 2. Manifestations, types and treatment of Schizophrenia. (15 marks) 5. Give an account on: 1. Anticoagulant drugs. (10 marks) 2. Antituberculous drugs. (10 marks)

49


Tanta University Faculty of Medicine Time allowed three hours

M.B.B.Ch. Internal Medicine Exam. Fifth Year Exam. Paper II September 2010

All questions must be answered (5 questions) – 150 marks 1. A) A 60 year old male presented with cervical lymph adenopathy. On examination he had a palpable liver 2 fingers below costal margin in the midclavicuiar line and a firm palpable spleen. Lymph nodes were detected in the. axillaes and the groins. Blood picture showed anaemia and leucocytic count of 120000/cmm composed mainly of lymphocytes. 1. What is the likely diagnosis? (5 marks) 2. What are the further investigations you would order? (5 marks) 3. What is the line of treatment of such condition? (5 marks) B) Mention the causes and clinical picture of Neutropenia. (10 marks) C) Enumerate 5 of: (15 marks) 1. Causes of Eosinophilia. 2. Causes of Thrombophilia. 3. Complications of Chemotherapy. 2. A) Give a short account on causes and differential diagnosis of Proteinuria. (10 marks) B) Give an account on causes, clinical picture, diagnosis and treatment of Acute Renal Failure. (15 marks) C) Enumerate 5 of: (15 marks) 1. Indications for Dialysis. 2. Causes of Anemia with Chronic Renal Failure. 3. Causes of Metabolic Acidosis. 3. Give in short notes: 1. Clinical features, investigations and treatment of Gout. (15 marks) 2. Types, clinical features, investigations and treatment of 3. Sjogren‘s syndrome8 (90 marks) 4. Criteria for diagnosis of Rheumatoid arthritis. (5 marks) 4. A) Give an account on causes, clinical picture, diagnosis and treatment of Aortic regurge. (10 marks) B) Mention complications of acute myocardial infarction. (10 marks) 5. Give an account on: 1. Clinical picture, complications, diagnosis and treatment of Typhoid fever. (10 marks) 2. Fever with splenomegaly. (10 marks)

50


Tanta University Faculty of Medicine Time allowed three hours

M.B.B.Ch. Internal Medicine Exam. Fifth Year Exam. Paper III September 2010

All questions must be answered (6 questions) – 150 marks 1. (Chest) Give an account on: (15 marks) 1. Recurrent pneumonia. 2. Differential diagnosis of suppurative lung syndrome. 3. Various types of pneumothorax. 2. (CVS) Mention short notes on: (15 marks) 1. Clinical picture of pulmonary hypertension. 2. Differential diagnosis of pansystolic murmur on the apex. 3. Management of atrial fibrillation. 3. (Neuro. & Psych.) Give an account on: (15 marks) 1. Clinical picture of sciatica. 2. Types of tremors. 3. Clinical picture of manic episodes. 4. (Tropical medicine) Give an account on: (15 marks) 1. Diagnosis of Schistosoma mansoni infection 2. Definition and causes of dysentery. 3. Methods of transmission of Brucella infection. 5. (Physical medicine) Mention briefly: (15 marks) 1. Diagnostic criteria-of Systemic lupus erythematosis. 2. Management of Bell s’ palsy 3. Rehabilitation program fora patient with peripheral nerve injury. 6. (Dermatology) Discuss in brief: (15 marks) 1. Clinical and laboratory manifestations of 2ry stage Syphilis. 2. Clinical varieties and management of Tinea Capitis. 3. Clinical types and systemic treatment of Psoriasis.

51


Tanta University Faculty of Medicine Time allowed three hours

M.B.B.Ch. Internal Medicine Exam. Fifth Year Exam. Paper I 12/7/2009

All questions must be answered (4 questions): 1. Give an account on: a) Causes and clinical picture Pulmonary Hypertension.

(15)

b) Syncope.

(10)

c) Clinical types of Angina, and its diagnosis.

(15)

2. Discuss briefly: a) Causes and diagnosis of" Diabetes Insipidus.

(15)

b) Insulin Resistance syndrome.

(15)

c) Osteomalacia.

(10)

3. Discuss: a) Causes and diagnosis of Malabsorption syndrome.

(15)

b) Autoimmune Hepatitis.

(10)

c) Gaslro-csopliagcal reflux disease.

(15)

4. Give short notes on: a) Infectious Mononucleosis.

(10)

b) Fever with rash.

(4)

c) Ankylosing spondylitis.

(10)

52


Tanta University Faculty of Medicine Time allowed three hours

M.B.B.Ch. Internal Medicine Exam. Fifth Year Exam. Paper II 15/7/2009

All questions must be answered (4 questions): 1. Give an account on: a) Manifestations, complications, and investigations of Nephrotic syndrome.

(16)

b) Metabolic acidosis.

(10)

c) Causes and treatment of Acute Tubular Necrosis.

(10)

2. Give short notes on: a) Causes, clinical features and complications of “Acute Empyema ”

(14)

b) Adult Respiratory Distress Syndrome.

(12)

c) Status Asthmaticus.

(10)

3. Discuss: a) Manifestations, diagnosis, and management of “Multiple Myeloma"

(14)

b) Differential Diagnosis of Microcytic Anemia.

(10)

c) Causes and diagnosis of “Acquired Coagulation Disorders”

(12)

4. Give short notes on: a) Causes and Treatment of Parkinsonism.

(18)

b) Clinical picture and treatment of Obsessive Compulsive Disorder.

(18)

53


Tanta University Faculty of Medicine Time allowed three hours

M.B.B.Ch. Internal Medicine Exam. Fifth Year Exam. Paper I 22/10/2008

All questions must be answered (4 questions): 1. Give an account on: a) Differential diagnosis of pansystolic murmur on the apex. (10) b) Causes and clinical picture of constrictive pericardities.

(15)

c) Treatment of pulmonary embolism.

(15)

2. Give an account on: a) Gastroesophageal reflux disease. (Heartburn).

(12)

b) Ulcerative colitis.

(12)

c) Liver transplant.

(12)

3. Give an account on: a) Thyrotoxic crises.

(10)

b) Causes and clinical picture of hypocalcemia.

(20)

c) Hypoglycemia unawarness.

(10)

4. Give an account on: a) Treatment of typhoid fever.

(10)

b) Clinical picture of Sjogren's syndrome.

(12)

c) Enumerate diffuse connective tissue disease.

(6)

54


Tanta University Faculty of Medicine Time allowed three hours

M.B.B.Ch. Internal Medicine Exam. Fifth Year Exam. Paper II 26/10/2008

All questions must be answered (4 questions): 1. Give an account on: a) Abnormalities in rate and rhythm of breathing.

(10)

b) Causes and clinical picture of corpulmonale.

(15)

c) Treatment of respiratory failure.

(10)

2. Give an account on: a) Causes of pathological proteinurea.

(10)

b) Differential diagnosis and treatment of nephrotic syndrome.

(15)

c) Hyperkalemia.

(15)

3. Give an account on: a) Oral anticoagulant.

(10)

b) Different type of haemolytic crisis.

(15)

c) Hyposplenesim.

(10)

4. Give an account on: a) Causes, clinical picture of transient ischemic attack.

(10)

b) Anxiety neurosis.

(10)

c) Causes of chronic recurrent headache.

(4)

55


Tanta University Faculty of Medicine Time allowed three hours

M.B.B.Ch. Internal Medicine Exam. Fifth Year Exam. Paper I 8/11/2007

All questions must be answered (4 questions): 1. A) A 32 year old man came to the GP complaining of feeling unwell, anorexia and nausea. He had been discharged from hospital 6 weeks ago where he had been admitted with ruptured spleen following car accident. He had been on tranquilizer? since this accident. On examinations lie was jaundiced with cervical lymphadenopathy. Chest and heart exam were normal. Abdominal examination revealed scar of recent splenectomy and palpable tender liver 3 cm below his right costal margin. (24 marks) 1. Suggest three causes of his jaundice. 2. What are the investigations needed for him? 3. What is the treatment of the most probable diagnosis? B) Complications and treatment of peptic ulcer. (12 Marks) 2. Discuss briefly: 1. Types and management of bradyarrythmia. (18 marks) 2. Clinical picture and management of acute pulmonary embolism. (18 marks) 3. Give short account on: 1. Causes and clinical picture of hyperparathyroidism. (18 marks) 2. Acute complications of diabetes mellitus. (18 marks) 4. Write short notes on: 1. Diagnostic approach to fever of unknown aetiology. (18 marks) 2. Clinical picture and treatment of systemic lupus erythematosus. (18 marks)

56


Tanta University Faculty of Medicine Time allowed three hours

M.B.B.Ch. Internal Medicine Exam. Fifth Year Exam. Paper II 12/11/2007

All questions must be answered (4 questions): 1. A) A 43 old nurse working in radiology departments came to her doctor complaining of easy fatigability and purpuric spots on her limbs. There was past history of fever since 2 months for which she had received antibiotics for 2 weeks and was improved. Examinations revealed nothing except pallor, purpuric spots. No organomegaly. CBC; lib 6.2 gm% WBCs 1 800/mm3 platelets 60 000/mm3. (24 marks) 1. What is the diagnosis of this case? 2. What are the most probable causes? 3. What further investigations and treatment of this case? B) Causes, clinical picture and diagnosis of disseminated intravascular coagulation. (12 marks) 2. Discuss briefly: 1. Differential diagnosis of unilateral dullness over the chest. (18 marks) 2. Anti tuberculous drugs. (18 marks) 3. Give short account on: 1. Acidosis. (18 marks) 2. Causes and clinical picture of nephrotic syndrome. (18 marks) 4. Write short notes on: 1. Anxiety neurosis. (18 marks) 2. Peripheral neuropathy. (18 marks)

57


Tanta University Faculty of Medicine Time allowed three hours

M.B.B.Ch. Internal Medicine Exam. Fifth Year Exam. Paper I 1/8/2006

All questions must be answered (4 questions): 1. A) 42 years old female goes to her GP because of gaining weight, 8. Kg over the past 5 months, with easy bruising and with difficulty in getting up from chair.' She feels depressed. She lias had no previous physical or psychiatric illness. On examination, she is overweight particularly in the abdominal region with dusky red face. There are purple stretch marks on her abdomen and thigh. Pulse 82/ min. BP 170/100, there is peripheral edema, her chest and abdominal examinations are normal. Investigations: Hb: 15.5g/ dL. Na: 138mcq/ L, K: 3.2 / mcq/L, urea 40 mg/ dL, Creatinine: 0.8mg/ dL, albumin: 39 g/L (N 35-50 g/L) fasting glucose: 160 mg/dL.Alanine transaminase 24 1U/L (N5-35 IU/L), urine glucose ++. (24 marks) 1. What is the likely diagnosis of this case? And what is the causes. 2. How would you investigate and manage this patient. B) Ketoacidosis. (12 Marks) 2. Write short note on: 1. Manifestations of hepatocellular failure. (12 marks) 2. Clinical picture and complications of ulcerative colitis. (12 marks) 3. Causes and clinical picture of acute pancreatitis. (12 marks)

3. Give concise account on: 1. Causes of cardiac arrest. (12 marks) 2. Types of tachyarrhythmia. (12 marks) 3. Clinical picture of pericardial effusion. (12 marks) 4. Give short notes on: 1. Clinical picture and treatment of systemic lupus. (15 marks) 2. Toxoplasmosis. (15 marks) 3. Behcet’s syndrome8 (6 marks)

58


Tanta University Faculty of Medicine Time allowed three hours

M.B.B.Ch. Internal Medicine Exam. Fifth Year Exam. Paper II 5/8/2006

All questions must be answered (4 questions): 1. A) 38 years old man presents with history of progressive swelling in his legs. Me is otherwise normal and had no significant-previous medical illness. On examination, there is pitting oedema in both legs extending to the groin, puffy7 face. His pulse is 75 b/m BP 140/90 mmHg. His jugular vein is not congested. Examination of the heart, chest abdomen and neurological system are normal. Investigations: HB: 13.3 g/dL. Sodium: 138 niEq/L. Potassium: 3.8 mEq/L Urea: 39 mg/dL creatinine: 1.1 mg/dL bilirubin: 1 mg/dL alanine transaminase: 33 IU/L (n 5-35 IU/L), cholesterol: 380 mg/dL (n < 200 mg/dL) urine: +++ protein – no blood (24 marks) 1. What disease these findings indicate? 2. What is the most common causes of the disease? And what is the differential diagnosis? 3. What is the further investigations which confirm the diagnosis? B) Osteomalacia. (12 Marks) 2. Give short account on: 1. Causes of thrombophilia. (15 marks) 2. Complications of blood transfusion. (15 marks) 3. Antioxidants. (6 marks)

3. Give short notes on: 1. Types and causes of respiratory failure. (12 marks) 2. Clinical picture of mediastinal syndrome. (12 marks) 3. Anti-tuberculous drugs. (12 marks) 4. Give short account on: 1. Causes and clinical picture of polyneuropathy. (18 marks) 2. Clinical picture and treatment of obssessional neurosis. (18 marks)

59


Tanta University Faculty of Medicine Time allowed three hours

M.B.B.Ch. Internal Medicine Exam. Fifth Year Exam. Paper I 15/11/2005

All questions must be answered (4 questions): 1. A) A 20 year aid female patient went to her OP; complaining of increased irritability and anxiety with difficulty on concentration since 3 months. She has lost 6 Kg in weight despite a good appetite. She has also noticed an increased frequency of' bowel movements. Her period have become lighter and shorter. She feels extremely tired, sweats profusely and cannot tolerate hot weather. On examination: her hands are sweaty and tremulous. Her eyes appear prominent. Pulse: 104 /min, regular, BP: 130/60. Examination is otherwise normal, investigations: Hb: 13.2 Ig/dL, WBC: 4,700 /mm3, platelets: 260,000/ mm3, urea 40 mg/dL, urine analysis is normal (24 marks) 1. What is the most likely diagnosis? And differential diagnosis of this case? 2. How would you investigate and manage this patient. B) Insulin therapy. (12 Marks) 2. Give short account on: 1. Clinical picture and diagnosis of coarctation of the aorta. (12 marks) 2. Clinical features of constrictive pericarditis. (12 marks) 3. Management of myocardial infarction. (12 marks)

3. Discuss briefly: 1. Portosystemic encephalopathy. (18 marks) 2. Dysentery. (18 marks) 4. Write short notes on: 1. Glandular fever. (15 marks) 2. Rheumatoid variants. (15 marks) 3. Pseudogout. (8 marks)

60


Tanta University Faculty of Medicine Time allowed three hours

M.B.B.Ch. Internal Medicine Exam. Fifth Year Exam. Paper II 18/11/2005

All questions must be answered (4 questions): 1. A) A 47 years-old woman has consulted her doctor because she awoke in the night with severe pain in the right side of her chest. The pain vas severe, sharp in character and exacerbated by breathing. She also complained of breathlessness and was sweating profusely. Four months previously she had a hysterectomy for fibroids and was complicated by deep vein thrombosis of her right caIf for which anticoagulants were prescribed. On examination; pulse: 110/min. BP: 10/60. JVP was raised 5 cm. accentuated 2nd heart sound on the pulmonary area. There were no abnormalities on examination of the chest or the abdomen. (24 marks) 1. Give differential diagnosis of this case. 2. What investigations would you do for her? 3. Outline your immediate management of this case. B) Status asthmaticus. (12 Marks) 2. Give short account on: 1. Idiopathic thrombocytopenic purpura. (18 marks) 2. Types and applications of bone marrow transplantation. (18 marks)

3. Discuss briefly: 1. Drug nephrotoxicity. (15 marks) 2. Hyperkalemia. (15 marks) 3. Hypervitaminosis D. (6 marks) 4. Discuss briefly: 1. Causes and clinical picture of sciatica. (18 marks) 2. Clinical picture and treatment of manic depression. (18 marks)

61


Tanta University Faculty of Medicine Time allowed three hours

M.B.B.Ch. Internal Medicine Exam. Fifth Year Exam. Paper I October/2004

All questions must be answered (4 questions): 1. Give an account on: 1. Complication and treatment of myocardial infarction. 2. Anti-arrhythmic drugs.

2. Discuss: 1. Pernicious anemia 2. Acidosis.

3. Discuss causes, Clinical Picture, investigation and treatment of Pneumothorax. 4. Discuss: 1. Neurological complications of Diabetes Mellitus. 2. Clinical picture, investigations and Treatment of Addison's disease.

62


Tanta University Faculty of Medicine Time allowed three hours

M.B.B.Ch. Internal Medicine Exam. Fifth Year Exam. Paper II October/2004

All questions must be answered (4 questions): 1. Discuss: 1. Acute viral hepatitis. 2. Dysentery. 2. Give an account on: 1. Brucellosis. 2. Causes, clinical picture, investigation and treatment of acute renal failure.

3. Discuss: 1. Causes and clinical picture of polyneuropathy, 2. Clinical picture and treatment of obsessional neurosis. 4. Discuss: 1. Treatment of Gout. 2. Investigation of a case of polyarthritis.

63


Tanta University Faculty of Medicine Time allowed three hours

M.B.B.Ch. Internal Medicine Exam. Fifth Year Exam. Paper I December/2003

All questions must be answered:

Give an account on: 1. Causes, clinical picture, diagnosis and treatment of mitral stenosis. 2. Definition and complication cardiac cachexia. 3. Causes, investigation, Differential diagnosis and treatment of lower gastrointestinal bleeding. 4. Drugs used in the treatment of peptic ulcer. 5. Hypothyroidism; types, caused, clinical picture, and differential diagnosis. 6. Enumerate diabetic complications. 7. Addisonian anaemia; pathogenesis, clinical picture, and investigation, 8. Treatment of Rhumatoid arthritis.

64


Tanta University Faculty of Medicine Time allowed three hours

M.B.B.Ch. Internal Medicine Exam. Fifth Year Exam. Paper II December/2003

All questions must be answered:

Give an account on: 1. Cor-puimonale; definition. Types, and clinical picture. 2. Dyspnea; definition, types and cause. 3. Nephrotic syndrome; definition, causes, manifestations, investigations complication, deferential diagnosis and treatment. 4. Causes, clinical picture, arid diagnosis of Parkinsonism. 5. Hysteria; causes and management. 6. Definition, mode of infection, clinical picture, and complications typhoid. 7. Treatment of gout.

65


Tanta University Faculty of Medicine Time allowed three hours

M.B.B.Ch. Internal Medicine Exam. Fifth Year Exam. Paper I December/2002

All questions must be answered:

Give an account on: 1. Classification, causes, diagnosis and treatment of cardiomyopathy, 2. Etiology, clinical picture and management of tetany. 3. Causes of type 1 diabetes mellitus. 4. Classification, clinical picture and diagnosis of malabsorbtion syndrome. 5. Causes, complications and treatment of constipation. 6. Clauses, clinical picture diagnosis and treatment of vitamin K deficiency. 7. Clinical picture of systemic lupus erythematosis.

66


Tanta University Faculty of Medicine Time allowed three hours

M.B.B.Ch. Internal Medicine Exam. Fifth Year Exam. Paper II December/2002

All questions must be answered:

Give an account on: 1. Clinical picture, pathology, pathogenesis, investigations and management of adult respiratory distress syndrome. 2. Causes and types of dyspnea. 3. Discuss syndromes of drug nephrotoxicity, causes and classification. 4. Causes and investigations of haematuria. 5. Give the clinical picture, diagnosis and treatment of cerebrospinal meningitis.

6. Describe the clinical picture and management of Hysteria. 7. Give the causes, clinical picture, diagnosis and treatment of cholera. 8. Give an account on Adisonian anaemia pathogenesis, clinical picture and diagnosis.

67



Hematological diseases

 General Questions:  Enumerate Haemopoietic Growth Factors. (Sep 2011)  Discuss types & application of Stem cell transplantation. (Oct 2009 , Nov 2006)

 Anemia: 1) Enumerate 4 causes of Anemia. (Midyear) 2) Enumerate 5 complications of sickle cell anemia. (Sep 2014) 3) Mention the definition, clinical features and treatment of pernicious anemia . (Nov 2013) 4) Enumerate 5 causes of normocytic anemia. (Dec 2011) 5) Discuss DD of microcytic anemia. (Nov 2010 , Jul 2009) 6) Discuss Iron deficiency anemia : def , causes , investigation & TTT 7) Discuss classification & diagnosis of hemolytic anemia. (Oct 2009) 8) Enumerate different causes of hemolytic anemia. (Jan 2009) 9) Give the causes & abnormal finding of hemolytic anemia. (Jun 2002) 10) Give short note on different types of hemolytic anemia. (Oct 2008) 11) Enumerate 5 causes of folate deficiency. (Dec 2011) 12) Discuss pernicious anemia. (Oct 2004) 13) Give an account on etiology, clinical picture & diagnosis of pernicious anemia. (Dec 2011 , Aug 2008) 14) Give short notes on: Addisonian anemia: pathogenesis, clinical picture & diagnosis. (Oct 2002) 15) Give an account on causes, clinical picture, diagnosis & TTT of aplastic anemia. (Nov 2012 ,Sep 2004) 16) Causes & DD of pancytopenia. (Dec 2011)

 Myeloproliferative disorders: 1) Give an account on causes & diagnosis of polycythemia. (Jan 2009 , Oct 2000) 2) Describe definition, clinical pictures and complications of Polycythemia rubra Vera. (Dec 2014) 3) Give an account on causes, pathology & clinical picture of polycythemia. (Dec 2001) 4) Enumerate 5 causes of secondary polycythemia. (Nov 2010)

69


ď ś Leukocytes: 1) 2) 3) 4) 5) 6) 7) 8) 9) 10) 11) 12) 13)

Give an account on leucopenia. (Jan 2009) Describe: 5 diagnostic investigations for multiple myeloma. (Aug 2015) Mention the causes & clinical picture of Neutropenia. (Sep 2010) Give short notes on Neutropenia. (Aug 2008) Enumerate 5 causes of Neutrophilia. (Sep 2014, Nov 2010) Enumerate 5 causes of eosinophilia. (Sep 2010) Enumerate 5 investigations for acute myeloid leukemia. (Dec 2014) Give an account on clinical picture, diagnosis & TTT of chronic myeloid leukemia. (Sep 2014, Nov 2010) Enumerate 5 complications of chemotherapy. (Sep 2010) Discuss manifestations, diagnosis & management of multiple myeloma. ( Sep 2011 ,Jul 2009) Enumerate 3 common causes of cervical lymphoadenopathy. (Sep 2004) Give short account on Lymphoadenopathy. (Sep 2011 , Dec 2011) Complications of massive bipod transfusion Hematological findings in Hodgkin's disease. (Nov 2013)

ď ś Bleeding Disorders: 1) Describe: 5 lines of treatment of idiopathic thrombocytopenic purpura. (Aug 2015) 2) 5 causes of thrombocytosis. (Dec 2014) 3) Mention the definition clinical features and treatment of essential thrombocytopenia. (Aug 2013) 4) Enumerate acquired coagulopathies. (Nov 2012) 5) Enumerate 2 causes of thrombophilia. (Sep 2010, Aug 2006) 6) Discuss Thrombocytopenic purpura. (Oct 2009) 7) Describe etiology, clinical picture, diagnosis & TTT of ITP. (Aug 2012) 8) Give short note on ITP. (Nov 2006) 9) Enumerate 5 causes of thrombocytopenia. (Nov 2013, Nov 2010) 10) Enumerate causes of non-thrombocytopenic purpura. (Sep 2011) 11) Enumerate 5 causes & describe 5 investigations for disseminated intravascular coagulation. (Nov 2015) 12) Discuss causes & diagnosis of Acquired coagulation disorders. (Jul 2009) 13) Enumerate Antithrombic therapy. (Aug 2013) 14) Give short account on types & indications of anti-thrombotic therapy. (Aug 2012) 15) Enumerate 5 Contraindications to oral anticoagulant therapy. (Nov 2015, Sep 2014) 16) Give short notes on Anticoagulant drugs. (Sep 2010) 17) Give short notes on oral anticoagulant. (Oct 2008) 18) Give short account on causes, clinical picture & diagnosis of disseminated intravascular coagulation. (Nov 2007)

70


ď ś Spleen: 1) Give short notes on Hyposplenism. (Oct 2008) 2) Enumerate 5 components of hypersplenism. (Aug 2013)

ď ś Blood transfusion: 1) Give short notes on complications of blood transfusion. (Nov 2010 , Aug 2008,Aug 2006) 2) Enumerate hazards of massive blood transfusion. (Nov 2015, Dec 2014, Nov 2012)

71


Gastrointestinal & Hepatological diseases

 General Questions: 1) Give an account on: DD of recurrent abdominal pain. (Jul 2008) 2) Give 4 causes of right-sided abdominal pain. (Mid-year) 3) Discuss: causes, complications & TTT of constipation. (Oct 2002) 4) Enumerate 5 causes of constipation. (Nov 2011) 5) Enumerate 5 causes of chronic diarrhea. (Nov 2011) 6) Enumerate 4 causes of hematemesis. (Mid-year) 7) Discuss: causes, investigations, DD & TTT of lower GIT bleeding. (Dec 2003) 8) Short note on: management of Upper GIT bleeding. (Aug 2007) 9) Discuss: causes, investigations, manifestations & TTT of Upper GIT bleeding. 10) Enumerate 5 most common causes of Lower GIT bleeding in Egypt. (Aug 2012) 11) Enumerate 5 most common causes of Upper GIT bleeding in Egypt. (Aug 2013) 12) Enumerate 4 causes of Hepatomegaly. (Mid-year)

 Diseases of the Esophagus: 1) Describe clinical picture, investigations and treatment of achalasia of the esophagus. (Dec 2014) 2) Mention the definition, clinical picture, diagnosis and treatment of achalasia of the cardia. (Aug 2013) 3) Give an account on Dysphagia. (Nov 2010) 4) Enumerate causes of oropharyngeal dysphagia. (Nov 2012) 5) Discuss Gastro-esophageal reflux disease (Heartburn). (Nov 2013, Jul 2009 , Oct 2008)

 Diseases of the stomach & duodenum: 1) 2) 3) 4) 5) 6) 7)

Enumerate 5 causes of acute gastritis. (Sep 2011) Enumerate 5 risk factors for gastric cancer. (Sep 2014) Mention diagnosis & TTT of peptic ulcer. (Sep 2010) Give an account on TTT of peptic ulcer. (Jul 2008, Dec 2001) Give an account on Drug therapy of peptic ulcer. (Jan 2009, Dec 2003) Discuss complications of peptic ulcer. (Dec 2014, Nov 2007,May 2000) Enumerate 5 complications of peptic ulcer. (Aug 2012)

72


ď ś Diseases of the intestine: 1) Give an account on: clinical picture, complications, diagnosis & TTT of Crohn's disease. (Aug 2015, Sep 2010) 2) Describe pathology, clinical picture & complications of ulcerative Colitis. (Oct 2015, Sep 2014) 3) Give short note on: Ulcerative colitis. (Oct 2008, Aug 2006) 4) Enumerate5 causes of chronic diarrhea. (Dec 2014) 5) Discuss: causes & diagnosis of malabsorption syndrome. (Jul 2009) 6) Give an account on: Etiological classification, clinical picture & diagnosis of malabsorption syndrome. (Nov 2011, Oct 2002) 7) Discuss briefly: Dysentery. (Nov 2006, Oct 2004) 8) Enumerate causes of dysentery. (Sep 2000, Sep 2011) 9) Give an account on: Causes & DD of Dysentery & TTT of 2 of them. (Dec 2001)

ď ś Diseases of pancreas: 1) 2) 3) 4)

Enumerate 5 causes of acute pancreatitis. (Sep 2014, Nov 2010) Enumerate 5 most common complications of acute pancreatitis. (Aug 2013) Discuss briefly: Acute pancreatitis. (Oct 2009) Give short note about Causes, C P & diagnosis of acute pancreatitis. (Aug 2015, Aug 2006, Aug 2012) 5) Describe the C/P & investigations of chronic pancreatitis. (Dec 2014, May 2000) 6) Give an account on: etiology, C/P, complications & TTT of chronic pancreatitis. (Sep 2011)

73


ď ś Diseases of the liver: 1) 2) 3) 4) 5) 6) 7) 8)

Enumerate 5 clinical manifestations of Wilson's disease. (Sep 2014) Enumerate 5 causes of intrahepatic cholestasis. (Sep 2011) Enumerate 3 common causes of jaundice. (Sep 2004) Discuss: def, types, causes & diagnostic approach of jaundice. (Jun 2002) Discuss: Acute viral hepatitis. (Oct 2004) Describe the C P , diagnosis & TTT of chronic viral hepatitis B. ( Nov 2012) Discuss autoimmune hepatitis. (Nov 2013, Jul 2009) Enumerate 5 causes of cirrhosis & describe 2 diagnostic investigations for each cause. (Oct 2015) 9) Enumerate causes of liver cirrhosis. (Jan 2009, Nov 2010) 10) Enumerate complications of liver cirrhosis. (Nov 2012) 11) Describe causes and clinical manifestations of fulminant hepatic failure. (Dec 2014) 12) Give an account on: causes & clinical picture of hepatocellular failure. (Jul 2008, Aug 2006, Sep 2011) 13) Discuss briefly: Causes, clinical picture & diagnosis of portal hypertension. (Aug 2015, Oct 2009, Jan 2009, Nov 2010) 14) Mention the management of Bleeding Esophageal varices. (Sep 2011) 15) Enumerate 5 causes of Ascites. (Nov 2011) 16) Discuss briefly: Portosystemic encephalopathy. (Nov 2006) 17) Give short note on: Liver transplant. (Oct 2008, Aug 2007) 18) Discuss briefly: Amoebic liver abscess. (Oct 2008)

74


Endocrinal diseases & Metabolism

ď ś Pituitary gland: 1) Enumerate 5 causes of Hirsutism. (Oct 2015) 2) Describe etiology, clinical picture and diagnosis of pan-hypopituitarism. (Sep 2014) 3) Discuss causes of hyperpitutrism & Discuss clinical features, diagnosis & TTT of Acromegaly. 4) Describe clinical picture & enumerate 5 investigations of acromegaly. (Aug 2015, Dec 2014) 5) Describe the etiology, clinical picture, diagnosis and treatment of acromegaly. (Aug 2013) 6) Enumerate 5 causes of stunted growth. (Sep 2010) 7) Enumerate causes of short stature. (Jan 2009) 8) Give short notes on Hyperprolactinaemia. (Jul 2008) 9) Enumerate causes of hyperprolactinaemia. (Aug 2012) 10) Discuss briefly causes, types, clinical picture, diagnosis & TTT of diabetes insipidus. (Nov 2013, Aug 2012, Jul 2009)

ď ś Thyroid gland: 1) Enumerate causes of enlargement of thyroid gland. (Oct 2000) 2) Enumerate eye manifestations of thyroid disease. (Nov 2012) 3) Enumerate skin manifestations of Grave's disease. (Nov 2012) 4) Discuss thyrotoxic crisis. (Oct 2009 , Oct 2008) 5) Give short account on causes & diagnosis of hypothyroidism. (Dec 2014, Jan 2009) 6) Describe clinical picture of hyperthyroidism. (May 2000) 7) Give an account on Diagnosis of Thyrotoxicosis. (Sep 2014, Dec 2001 , Oct 2000) 8) Describe TTT of Thyrotoxic crisis. (Jun 2002) 9) Discuss investigations & TTT of thyroid disease. (Jan 2005) 10) Give an account on clinical picture & TTT of hypothyroidism. (Oct 2015, Jun 2007) 11) Give an account on Types, causes, clinical picture, investigations & DD of hypothyroidism. (Sep 2010 , Dec 2003) 12) Mention management of Myxedema Coma. (Sep 2011)

75


ď ś Parathyroid glands: 1) Enumerate 5 causes of hypercalcemia. (Aug 2015, Sep 2010) 2) Give an account on etiology, clinical picture & management of hypercalcemia. (Aug 2015, Aug 2012 , Sep 2010) 3) Enumerate 5 causes of hyperphosphatemia. (Sep 2014) 4) Give an account on causes & clinical picture of hyperparathyroidism. (Nov 2007) 5) Give an account on causes & clinical picture of Hypocalcemia. (Dec 2014, Oct 2008 , Jul 2008) 6) Give short note on causes, clinical picture & TTT of Hypocalcaemia. (Nov 2012 ,Nov 2010) 7) Give short note on causes, clinical picture & management of Tetany. (Nov 2011 ,Oct 2002)

ď ś Adrenal glands: 1) Enumerate 5 Clinical presentations of pheochromocytoma. (Oct 2015, Nov 2013) 2) Enumerate 5 investigations of Cushing 'syndrome. (Oct 2015) 3) Describe definition, clinical features and investigations of Conn's syndrome. (Sep 2014) 4) Describe causes and clinical picture of Cushing's syndrome. (Dec 2014) 5) Describe the etiology, clinical features, diagnosis & TTT of primary adrenocortical insufficiency. (Aug 2013) 6) Enumerate 5 causes of 1ry adrenal insufficiency. (Nov 2010) 7) Give the causes & investigations of adrenocortical insufficiency. (May 2000) 8) Describe clinical picture, diagnosis & TTT of 1ry aldosteronism. (Nov 2012) 9) Enumerate 5 complications of Corticosteroid therapy. (Nov 2010) 10) Discuss clinical picture, investigations & TTT of Addison's disease. (Oct 2004)

76


 Pancreas: 1) Enumerate 5 causes of hypoglycemia. (Nov 2010) 2) Mention causes, classification, manifestations & TTT of hypoglycemia. (Nov 2011 ,Jun 2002) 3) Give short account on causes of Type 1 Diabetes Mellitus. (Oct 2002) 4) Enumerate 5 causes of 2ry D M. (Sep 2010) 5) Enumerate vascular complications of diabetes mellitus. (Nov 2013) 6) Discuss diagnosis of Diabetes Mellitus. (Nov 2010 ,Oct 2009) 7) Give an account on complications of Diabetes Mellitus. (Aug 2015, Sep 2014, Jun 2007,Nov 2007, Dec 2003 , Oct 2000) 8) Discuss Neurological complications of Diabetes Mellitus. (Oct 2004) 9) Enumerate types of diabetic neuropathy. (Nov 2012) 10) Give short account on Disturbed conscious in Diabetes Mellitus. (Nov 2011) 11) Give an account on Ketoacidosis. (Aug 2006) 12) Give an account on Diagnosis & TTT of ketoacidosis. (Jul 2008 ,Dec 2001) 13) Give an account on Diabetic ketoacidosis: precipitating factors, manifestations & management. (Sep 2011) 14) Mention Oral antidiabetic drugs. (Aug 2015, Aug 2013, Sep 2010) 15) Give an account on indications & complications of Insulin therapy. (Oct 2015, Jan 2009 , Nov 2006) 16) Discuss briefly Insulin Resistance Syndrome. (Jul 2009 , Aug 2007) 17) Give short note on Hypoglycemia unawareness. (Oct 2008)

 Obesity, Gout & Lipid metabolism: 1) 2) 3) 4) 5)

Enumerate 5 diagnostic criteria of metabolic syndrome. (Oct 2015, Aug 2013) Write short account on Pseudogout. (Nov 2006) Discuss clinical features, investigations & TTT of Gout. (Sep 2010) Give the TTT of Gout. (Oct 2004 , Dec 2003 ,Dec 2001 , May 2000) Give the TTT of acute gout. (Oct 2000)

 Vitamins: 1) 2) 3) 4) 5)

Discuss briefly: Osteomalacia. (Jul 2009) Discuss briefly Hypervitaminosis D. (Nov 2006) Give the clinical picture of deficiency of Fat soluble vitamins. (Oct 2000) Enumerate causes of Vitamin K deficiency. (Nov 2011) Give an account on causes, clinical picture, diagnosis & TTT of vitamin K deficiency. (Jun 2007 , Oct 2002) 6) Give an account on causes, clinical picture & TTT of pellagra. (Dec 2001) 7) Describe the clinical picture & TTT of pellagra. (Jun 2002)

77


Nephrological diseases

ď ś General Questions: 1) Define micro-albuminuria, describe its significance & methods of its detection. (Aug 2015) 2) 5 stages of vesicoureteric reflux. (Dec 2014) 3) Enumerate Different modalities to control microalbuminuria. (Aug 2013) 4) Give short note on causes & DD of proteinuria. (Sep 2010) 5) Give short note on causes of pathological proteinuria. (Oct 2008) 6) Enumerate renal function tests. (Nov 2012) 7) Give an account on polyuria. (Jan 2009, Aug 2008, Dec 2001) 8) Give 4 causes of polyuria. (Nov 2010) 9) Discuss causes & investigations of hematuria. (Sep 2014, Aug 2015, Oct 2002) 10) Differentiate between Acute & Chronic renal failure.

ď ś Glomerular diseases: 1) Give short note on acute glomerulonephritis. (Oct 2009) 2) Give short account on causes, clinical picture & TTT of Acute glomerulonephritis. (Jun 2007, Dec 2001) 3) Discuss nephritic syndrome: def, etiology, clinical manifestations, diagnosis, complications & TTT. (Jun 2002) 4) Enumerate complications of nephritic syndrome. (Aug 2015, Nov 2012) 5) Give an account on manifestations, complications & investigations of nephrotic syndrome. (Jul 2009) 6) Investigations & TTT of nephrotic syndrome. (Aug 2012) 7) Write short note on causes, clinical picture & diagnosis of nephrotic syndrome. (Aug 2013, Jan 2009, Nov 2007, Sep 2000) 8) Give short note on DD & TTT of nephrotic syndrome. (Oct 2008) 9) Enumerate 5 causes of nephrotic syndrome. (Nov 2010) 10) Discuss nephrotic syndrome: def, causes, manifestations, investigations, complications, DD & TTT. (Dec 2003)

78


 Acute Kidney Injury: 1) Define acute kidney injury and Describe 5 pre-renal causes of it. (Nov 2015) 2) Give the etiology, clinical picture, diagnosis & TTT of acute renal failure. (Sep 2010, Oct 2000) 3) Give short note on causes, clinical picture, investigations & TTT of acute renal failure. (Oct 2004, Nov 1999) 4) Mention causes of Acute Tubular Necrosis. (Nov 2013, Dec 2011, Nov 2010) 5) Give an account on causes & TTT of Acute Tubular Necrosis. (Jul 2009)

 Chronic Kidney Diseases: 1) Mention definition, etiology and stages of chronic kidney disease. (Sep 2014) 2) Describe causes and diagnosis of chronic interstitial nephritis. (Dec 2014) 3) Mention the definition, etiology and clinical picture of acute interstitial nephritis. (Nov 2013) 4) Give short note on causes & TTT of chronic irreversible renal failure. (Aug 2008) 5) Enumerate 5 causes of Anemia with chronic renal failure. (Sep 2010) 6) Describe skin, cardiovascular & Neuropsychiatric manifestations of chronic renal failure. (Aug 2012) 7) Describe the gastrointestinal, hematological & musculoskeletal manifestations of chronic renal failure. (Nov 2012) 8) Mention the conservative TTT of chronic renal failure. (Sep 2011) 9) Mention the TTT of End-stage Renal Diseases. (Dec 2011) 10) Give short account on indications, types & complications of Dialysis. (Sep 2014, Nov 2010) 11) Mention the indications, advantages and disadvantages of renal replacement therapy. (Aug 2013)

 Interstitial & Tubular Diseases: 1) Give short account on Reflux Nephropathy. (Sep 2014, Oct 2009) 2) Describe 5 of Investigations of chronic pyelonephritis. (Aug 2015, Aug 2013) 3) Describe causes, clinical picture and investigations of acute tubular necrosis. (Sep 2014) 4) Discuss Pyelonephritis: def., types, clinical picture, investigations & TTT. 5) Give an account on etiology, clinical picture, investigations & TTT of Acute Pyelonephritis. (Sep 2011) 6) Discuss briefly Drug Nephrotoxicity. (Nov 2006) 7) Discuss Syndrome of drug nephrotoxicity causes & classification. (Oct 2002)

79


ď ś Water, Electrolytes & Acid-Base Balance: 1) Give short note on Hyperkalemia. (Oct 2009, Oct 2008 , Nov 2006) 2) Enumerate 5 causes of hypernatremia. (Dec 2014) 3) Enumerate 5 Causes of renal anemia. (Dec 2014) 4) Mention causes, clinical picture & TTT of Hyperkalemia. (Nov 2010) 5) Give short note on Hypokalemia. (Aug 2008) 6) Mention def, etiology, clinical picture & TTT of Hypokalemia. (Nov 2015, Nov 2012) 7) Causes, clinical picture & TTT of Hyponatremia. (Aug 2012) 8) Write short note on Acidosis. (Nov 2007) 9) Give an account on metabolic acidosis. (Nov 2015, Aug 2013, Jul 2009) 10) Enumerate 5 causes of metabolic acidosis. (Sep 2010) 11) Give short note on causes & clinical picture of metabolic alkalosis. (Nov 2013, Sep 2011)

80


Rehumatological diseases

 General Questions: 1) 2) 3) 4)

Enumerate diffuse connective tissue diseases. (Oct 2008) Give an account on DD of chronic polyarthritis. (Nov 1999) Enumerate 5 non immunological causes of arthropathy. (Sep 2014) Mention definition and describe clinical picture, investigations of dermatomyositis. (Sep 2014)

 Rheumatoid arthritis: 1) Enumerate 5 disease modifying anti-rheumatic Drugs (DMARDs) & describe 2 side Effects of each drug. (Oct 2015) 2) Enumerate 5 extra-articular manifestations of rheumatoid arthritis. (Aug 2015) 3) Enumerate 5 Musculoskeletal manifestations of rheumatoid arthritis. (Dec 2014, Aug 2013) 4) Give an account on criteria for diagnosis of rheumatoid arthritis. (Sep 2011 , Sep 2010) 5) Mention the extra-articular manifestations of rheumatoid disease. (Nov 2012 ,Nov 2011) 6) Give an account on TTT of rheumatoid arthritis. (Jan 2009 , Aug 2007 , Dec 2003 , Oct 2000) 7) Give an account on investigations & TTT of rheumatoid arthritis. (Jun 2007) 8) Write short note on rheumatoid variants. (Nov 2006)

 Systemic Lupus Erythromatosus (SLE): 1) Give an account on diagnostic criteria & TTT of SLE. (Aug 2015, Jul 2008, Aug 2006) 2) Enumerate 5 Serological and immunological tests for diagnosis of SLE. (Dec 2014, Aug 2013) 3) Write short note on clinical picture & complications of SLE. (Jun 2002 , Nov 2007) 4) Mention investigations & TTT of SLE. (Nov 2011) 5) Describe the pathogenesis, clinical picture, investigations & TTT of SLE. (Nov 2012)

81


 Scleroderma: 1) Give short account on scleroderma (progressive systemic sclerosis). (Aug 2015, Oct 2009) 2) Describe pathogenesis, Classification, clinical picture, and treatment of scleroderma. (Nov 2013) 3) Enumerate 5 serological investigations of for scleroderma. (Sep 2014) 4) Pathogenesis, clinical features, investigations & TTT of progressive systemic sclerosis. (Aug 2012)

 Sjogren's syndrome: 1) Give an account on types, clinical features, investigations & TTT of Sjogren's syndrome. (Dec 2014, Sep 2010) 2) Give an account on clinical picture of Sjogren's syndrome. (Oct 2015, Oct 2008, Aug 2007)

 Behcet's disease: 1) Give an account on on Behcet's disease. (Nov 2013, Jan 2009 , Aug 2006) 2) Mention the clinical picture of Behcet's disease. (Aug 2012)

 Seronegative Spondylarthropathies: 1) 2) 3) 4) 5) 6) 7)

Enumerate the seronegative spondylarthropathies. Enumerate 5 causes of seronegative spondylarthritis. (Aug 2015, Sep 2014) Mention the common features of SNSA. (Sep 2011) Give an account on Reiter's syndrome: def, clinical picture & management. Give short note on Ankylosing spondylitis. (Jul 2009) Enumerate the clinical characteristics of ankylosing spondylitis. (Nov 2012) Mention the clinical features of polyarthritis nodosa. (Aug 2013, Aug 2012)

82


Infectious diseases

 Viral infections: 1) Give short notes on Infectious Mononucleosis. (Nov 2011 ,Jul 2009) 2) Write short note on Glandular fever. (Nov 2006)

 Bacterial infections: 1) Give shirt notes on mode of infection, clinical picture, complications, diagnosis & TTT of Typhoid fever. (Dec 2014, Sep 2010 ,Jul 2008, Dec 2003) 2) Describe the pathology, clinical picture, investigations and treatment of amoebic liver abscess. (Sep 2014, Nov 2013) 3) Give an account on TTT of typhoid fever. (Oct 2008 , Dec 2001 , Oct 2000) 4) Describe etiology, clinical picture, diagnosis & TTT of brucellosis. (Sep 2014, Nov 2012 , Oct 2009 , Jan 2009 ,Oct 2004 ,Jun 2002) 5) Describe clinical picture & diagnosis of brucellosis. (May 2000) 6) Give the causes, clinical picture, giagnosis & TTT of cholera. (Oct 2002) 7) Give concise account on Meningococcal meningitis. (Oct 2015, Dec 2014, Aug 2013, Nov 2010) 8) Give the clinical picture, diagnosis & TTT of cerebrospinal meningitis. (Jun 2002)

 Parasitic infections: 1) Give short note on Toxoplasmosis. (Dec 2014, Aug 2006) 2) Mention briefly diagnosis & TTT of toxoplasmosis. (Nov 2012) 3) Give short note on diagnosis & TTT of schistosomiasis mansoni. (Sep 2014, Nov 2013, Aug 2012) 4) Schistosomal Hepatic fibrosis. (Nov 2010) 5) Give an account on TTT of Malaria. (Nov 1999) 6) Give short account on diagnosis, complications & TTT of cerebral malaria. (Sep 2014, Nov 2013, Aug 2012) 7) Mention briefly diagnosis & TTT of black water fever. (Nov 2012) 8) Mention clinical picture, complications & TTT of Amoebic colitis. (Nov 2012) 9) Mention briefly diagnosis & TTT of Amoebic hepatitis. (Nov 2012) 10) Give short notes on diagnosis & TTT of Ascariasis. (Sep 2014, Aug 2012)

83


Cardiovascular diseases

 General Questions: 1) Mention short notes on DD of Pansystolic murmur. (Sep 2010) 2) Describe differential diagnosis of ejection systolic murmur at the base of the heart. (Oct 2015) 3) Describe management of patient with acute cardiac chest pain in emergency room. (Dec 2014) 4) Mention short notes on DD of Pansystolic murmur over the apex. (Oct 2008) 5) Mention short notes on DD of Mid diastolic murmur. (Sep 2011) 6) Mention short notes on DD of Mid diastolic murmur over the apex. (Jan 2009) 7) Give short account on: Syncope. (Jul 2009) 8) Give an account on: DD of acute cardiovascular pain. (Jul 2008, Nov 2010) 9) Enumerate 3 common causes of:  Displacement of the apex beat.  Congested neck veins.  Hemoptysis. 10) Enumerate 3 mechanisms of:  Orthopnia  Palpitations  Edema 11) Enumerate causes of:  Chest pain  Dyspnea  Dullness in the 2nd intercostal spaces.

 Heart Failure: 1) 2) 3) 4) 5) 6) 7) 8) 9)

Mention short notes on: precipitating causes of heart failure. (Dec 2011) Enumerate 5 investigations for heart failure. (Sep 2014) Enumerate 5 causes of Lt sided heart failure. (Dec 2014) Mention short notes on: causes & clinical picture of Lt ventricular failure. (Sep 2011) Give short account on: TTT & diagnosis of acute pulmonary edema. (Jul 2008, 1999) Give an account on causes, C P & TTT of Acute pulmonary edema. (Nov 2011) Give short account on definition & complications of cardiac cachexia. (Dec 2003) Give short account on causes & TTT of intractable heart failure. (Aug 2007) Discuss causes, clinical picture & TTT of heart failure. (Jan 2005)

84


 Valvular heart diseases: 1) 2) 3) 4) 5) 6)

Mention complications of mitral stenosis. (Aug 2013, Nov 2010, Nov 2012) Enumerate 5 clinical manifestations of mitral stenosis. (Dec 2014) Enumerate 5 causes of aortic regurgitation. (Sep 2014) Discuss causes, clinical picture, diagnosis & TTT of mitral stenosis. (Dec 2003) Enumerate manifestation of Aortic regurge. (Aug 2013, 1999) Give short notes on causes, clinical picture, and diagnosis & TTT of Aortic regurge. (Sep 2010) 7) Give an account on causes, clinical picture & complications of Aortic stenosis. (Sep 2011)

 Congenital Heart diseases: 1) Give short notes on Clinical picture & diagnosis of Coacrtation of aorta. (Nov 2006) 2) Enumerate congenital heart diseases & discuss the clinical picture, diagnosis & TTT of PDA. (Dec 2001)

 Diseases of the pericardium: 1) Give an account on causes & clinical picture of constrictive pericarditis. (Nov 2006 , Oct 2008 , Oct 2009) 2) Give an account on causes, clinical picture, diagnosis & TTT of pericardial effusion. (Jun 2002 , Jul 2008 , Oct 2000 ,Aug 2012 , Aug 2006)

 Rheumatic fever: 1) Give an account on Manifestations & TTT of Rheumatic fever. (Sep 2004) 2) Enumerate 5 manifestations of acute rheumatic fever. (Dec 2014) 3) Enumerate manifestations of Rheumatic fever. (Sep 2000)

 Infective endocarditis: 1) Give an account on clinical manifestations of infective endocarditis. (Jan 2009 , Aug 2007) 2) Mention short notes on causes, clinical picture & management of infective endocarditis. (Oct 2015, Nov 2010 , Sep 2011) 3) Complications of infective endocarditis. (Nov 2013)

85


 Coronary Heart disease: 1) Discuss in brief: diagnosis & TTT of angina pectoris. (Nov 2011) 2) Describe the different atherosclerotic risk factors & their possible management. (Nov 2012) 3) Mention complications of acute myocardial infarction. (Sep 2014, Aug 2015, Sep 2010 ,Dec 2011 , Oct 2004) 4) Give short notes on management of myocardial infarction. (Aug 2015, Oct 2004 ,Oct 2009 , Nov 2006) 5) Give short note on clinical types of Angina & its diagnosis. (Nov 2013, Jul 2009)

 Systemic Hypertension: 1) 2) 3) 4) 5)

Give short notes on Antihypertensive drugs. (Aug 2007) Enumerate Non pharmacologic therapies of hypertension. (Aug 2013) Enumerate causes of surgically curable hypertension. (Nov 2012) Mention short notes on causes of 2ndry hypertension. (Dec 2011) Describe def, causes, clinical picture, complications & TTT of systemic hypertension. (May 2000) 6) Discuss Def, types & complications of 2ndry systemic hypertension. (Sep 2014, Sep 2011)

 Pulmonary Hypertension & Embolism: 1) Mention short notes on causes, clinical picture of pulmonary hypertension. (Sep 2010 , Jul 2009) 2) Give short notes on TTT of pulmonary embolism. (Oct 2008) 3) Discuss briefly clinical picture & management of pulmonary embolism. (Nov 2007)

 Arrhythmia: 1) Mention short notes on management of atrial fibrillation. (Sep 2010 , Jun 2002 , Oct 2000) 2) Give an account on: Atrial fibrillation. (Oct 2009) 3) Enumerate the big 5 causes of atrial fibrillation. (Nov 2013, Nov 2012 ,Jun 2002 , Oct 2000, Sep 2000 ) 4) Describe causes, diagnosis, clinical picture & TTT of atrial fibrillation. (Aug 2015, Mid-year 2002, Aug 2012 , Nov 2010 ) 5) Discuss briefly Types & management of bradyarrythmia. (Nov 2007) 6) Give concise account on: causes of cardiac arrest. (Aug 2006) 7) Give concise account on: Types of tachyarrythmia. (Aug 2006) 8) Give an account on: Def, causes & TTT of cardiac arrest. 9) Give an account on: Anti-arrhythmic drugs. (Oct 2004)

86


ď ś Cardiomyopathy: 1) Give an account on: classification, causes, diagnosis & TTT of cardiomyopathy. (Oct 2002) 2) Enumerate Different types of cardiomyopathy. (Nov 2012)

87


Respiratory diseases

 General Questions: 1) 2) 3) 4)

5) 6) 7) 8) 9) 10)

Give an account on: causes & management of hemoptysis. (Jan 2009) Enumerate 5 complications of chronic cough. (Nov 2015) Non cardiac causes of chest pain. (Nov 2013) Enumerate causes of :  Hemoptysis  Dyspnea  Chest pain Give an account on: def, causes & types of dyspnea. (Oct 2002, Dec 2003) Give an account on: Wheezy chest. (Aug 2008) Give an account on: Abnormalities in rate & rhythm of breathing. (Oct 2008) Discuss briefly DD of unilateral dullness over the chest. (Nov 2007, Sep 2004) Give short account on: various types of crepitations. (Nov 2010) Give short account on: Clubbing of the fingers due to chest causes. (Dec 2011)

 Diseases of the pleura: 1) Give short account on: causes of pleural effusion. (Dec 2011) 2) Enumerate Causes of exudative pleural effusion. (Aug 2015) 3) Discuss causes, Clinical picture, diagnosis & TTT of pleural effusion. (Sep 2014, Sep 2010) 4) Give short account on: def , clinical picture, causes & complications of acute Empyema. (Nov 2010 , Jul 2009) 5) Give short account on: clinical picture & TTT of pneumothorax. (Dec 2014, Nov 2013, Jan 2009) 6) Discuss causes, types, clinical picture, investigations & TTT of pneumothorax. (Oct 2004 , Aug 2012)

88


ď ś Diseases of bronchi: 1) Discuss causes, clinical picture, diagnosis & TTT of bronchial asthma. (Nov 2010) 2) Risk factors for COPD. (Aug 2015) 3) Give short note on: Status Asthmaticus (life-threatening asthma). (Jul 2009 , Nov 2006, Sep 2011) 4) Mention def, clinical characteristics & lines of TTT of acute severe asthma (Status Asthmaticus). (Nov 2012) 5) Give short account on Bronchodilators. (Dec 2014, Sep 2011) 6) Give short account on Emphysema (types, etiology, pathogenesis, clinical features & TTT). (Dec 2011) 7) Mention Def, clinical picture & TTT of chronic obstructive lung diseases. (Dec 2014, Dec 2011) 8) Describe causes, clinical picture, diagnosis & complications of Bronchiectasis effusion. (Jun 2002) 9) Complications of bronchiectasis. (Aug 2015) 10) Causes of bronchiectasis. (Nov 2015)

ď ś Infective diseases of the lung: 1) Give short account on: Antituberculous drugs. (Aug 2013, Sep 2010, Jan 2009 , Nov 2007 , Aug 2006) 2) Complications of pneumonia. (Nov 2015) 3) Discuss briefly prevention & TTT of pulmonary T B. (Aug 2015, Oct 2009) 4) Mention side effects of anti- TB drugs. (Sep 2014) 5) Describe the etiology, clinical picture, diagnosis and treatment of community acquired pneumonia. (Aug 2013) 6) Give short account about recurrent pneumonia. (Sep 2010) 7) Describe the etiology, clinical picture, diagnosis & TTT of mycoplasma pneumonia. (Aug 2012) 8) Give short account on DD of suppurative lung syndrome. (Nov 1999,Sep 2010) 9) Give short account on causes & DD of suppurative lung syndrome & Discuss clinical picture & TTT of one of them. (Jan 2005, Dec 2001) 10) Give short notes about causes & clinical picture of lung abscess. (Sep 2011 , Aug 2008) 11) Discuss causes, clinical picture, diagnosis & TTT of lung abscess. (Nov 2013, Dec 2011)

89


 Tumors of the bronchi: 1) Discuss briefly extra-thoracic manifestations of bronchogenic carcinoma. (Oct 2009 , Aug 2008) 2) Discuss etiology, pathology & diagnosis of bronchogenic carcinoma. (Nov 2015, Sep 2014, Nov 2012, Aug 2006,Oct 2000) 3) Give short notes on Paramalignant syndrome of bronchogenic carcinoma. (Sep 2011) 4) Give short notes on causes of solitary pulmonary nodule (Nov 2010) 5) Describe clinical picture, diagnosis & TTT of bronchial adenoma. (Aug 2012)

 Pulmonary heart diseases (cor-pulmonale): 1) 2) 3) 4)

Mention short notes on Cor-pulmonale def & etiology. (Nov 2010, Dec 2003) Give short note on causes & clinical picture of Cor-pulmonale. (Oct 2008) Enumerate Investigations of pulmonary embolism. (Nov 2015) Diagnosis of pulmonary embolism. (Aug 2013)

 Adult Respiratory distress syndrome: 1) Give short note on Adult Respiratory distress syndrome. (Jul 2009) 2) Give short note on clinical picture, pathology, pathogenesis, investigations & management of Adult Respiratory distress syndrome. (Oct 2002)

 Respiratory Failure: 1) Give short note on Pulmonary function tests in Obstructive & Restrictive hypoventilation. (Dec 2011) 2) Give short notes on TTT of respiratory failure. (Oct 2008) 3) Give short note on Types & causes of respiratory failure. (Aug 2006) 4) Give short note on clinical picture of Mediastinal Syndrome. (Aug 2006)

90


Neuropsychiatry

ď ś Neurology: 1) Give short note on types of aphasia. (Nov 2010) 2) Definition, etiology, clinical picture, investigations & treatment of Guillain-Barre' Syndrome. (Nov 2015, Nov 2013) 3) Ataxia: types and differential diagnosis. (Aug 2015) 4) Enumerate indications of electroconvulsive therapy. (Sep 2014) 5) Enumerate 3 common causes of abnormal gait. (Sep 2004) 6) Give an account on causes, clinical picture & TTT of hemiplegia. (Sep 2014, Aug 2013, Jan 2009) 7) Give short note on causes, clinical picture of transient Ischemic attack. (Oct 2008) 8) Give an account on causes, clinical picture & TTT of Transient Ischemic Attack. (Nov 1999) 9) Give short note on types of tremors. (Sep 2010) 10) Enumerate 4 causes of tremors. (Midyear 2002) 11) Give short note on causes, clinical picture, diagnosis & TTT of Parkinsonism. (Sep 2014, Dec 2014 Aug 2012, Jul 2009) 12) Give an account on clinical picture, diagnosis & TTT of Rheumatic chorea. (Dec 2011) 13) Give an account on etiology, clinical picture & diagnosis of Neuropathy. (Sep 2011) 14) Give short note on polyneuropathy. (Oct 2009 , Aug 2008, Aug 2006, Oct 2004) 15) Give short note on causes of Peripheral Neuritis. (Dec 2011) 16) Discuss Peripheral Neuropathy. (Jun 2007 , Nov 2007) 17) Give an account on classification, clinical picture, diagnosis & TTT of epilepsy. (Nov 2010) 18) Mention classification & management of different types of epilepsy. (Nov 2012) 19) Give short note on DD of Tonic-clonic seizures. (Sep 2011) 20) Give short note on management of Multiple Sclerosis. (Sep 2011) 21) Give short note on causes of chronic recurrent headache. (Oct 2008, Aug 2008) 22) Give short note on TTT of Migraine. (Dec 2011) 23) Give an account on etiology, clinical picture & TTT of migraine. (Sep 2010) 24) Give short account on clinical picture of Sciatica. (Sep 2010) 25) Discuss briefly causes, types & clinical picture of Sciatica. (Nov 2010, Nov 2006) 26) Discuss briefly clinical picture TTT of Sciatica. (Dec 2001) 27) Give concise account on Meningococcal meningitis. (Aug 2007) 28) Give clinical picture, diagnosis & TTT of cerebrospinal meningitis. (Jun 2002, Oct 2002)

91


ď ś Psychiatry: 1) Give short note on Anxiety neurosis. (Oct 2008, Nov 2007) 2) Causes, clinical picture and treatment of dissociative disorders. (Nov 2015) 3) Give an account on etiology, clinical picture & management of Generalized Anxiety Disorders. (Aug 2015, Dec 2014, Sep 2011) 4) Describe the clinical picture and management of major depressive disorder. (Aug 2013) 5) Describe the clinical picture and management of panic disorder. (Nov 2013) 6) Give short on clinical picture & TTT of Obsessive compulsive Disorder. (Aug 2012 , Jul 2009 , Aug 2006 ,Oct 2004 , Dec 2001, Oct 2000) 7) Give short on causes, clinical picture & management of Obsessive compulsive neurosis. (Nov 2010) 8) Give short account on mental manifestations of Hysteria. (Nov 2010) 9) Give an account on clinical picture & TTT of hysteria. (Jan 2009, Jun 2002 , Oct 2002) 10) Give an account on causes & management of hysteria. (Dec 2003) 11) Give short account on clinical picture of Manic Episodes. (Sep 2010) 12) Give short note on Endogenous Depression. (Aug 2008) 13) Discuss clinical picture & TTT of Manic depressive psychosis. (Jun 2007 , Nov 2006 , Nov 1999) 14) Give short note on TTT of major depression. (Dec 2011) 15) Give an account on etiology, manifestations, types & TTT of Schizophrenia. (Sep 2014, Dec 2011,Sep 2010 , Oct 2009) 16) Give short note on clinical manifestations & management of Schizophrenia. (Nov 2012) 17) Give short account on clinical picture of Dementia. (Sep 2011)

92


Tropical Medicine 1) Describe the transmission, clinical picture & complications of brucellosis. (Aug 2015) 2) Give an account on Amebic liver abscess management. (Aug 2015) 3) Give an account on Causes of fever with splenomegaly. (Aug 2015) 4) Enumerate 5 causes of acute abdominal pain. (Aug 2015) 5) Enumerate 5 complications of chemotherapy. (Aug 2015) 6) Write short note on Chemoprophylaxis of malaria. (Nov 2015) 7) Write short note on Causes of acute diarrhea. (Nov 2015) 8) Write short note on Clinical and laboratory diagnosis of HIV. (Nov 2015, Nov 2013) 9) Enumerate 5 causes, of cholestatic jaundice. (Dec 2014) 10) Diagnosis of Hepatitis B virus infection. (Dec 2011) 11) Diagnosis of Typhoid fever. (Dec 2011) 12) Clinical characters of Schistosomal liver. (Dec 2011) 13) DD of Huge splenomegaly. (Sep 2011) 14) Malaria prophylaxis before travel. (Sep 2011) 15) Clinical manifestations of portal hypertension. (Sep 2011) 16) Diagnosis of a case with malaria. (Nov 2010) 17) Complications of typhoid fever. (Nov 2010) 18) TTT of Amebic liver abscess. (Nov 2010) 19) Diagnosis of Schistosoma Mansoni infection. (Sep 2010) 20) Definition & causes of Dysentery. (Sep 2010) 21) Methods of transmission of Brucella infection. (Sep 2010) 22) Describe patterns of fever. (Jul 2008) 23) Give account on causes of prolonged fever. 24) Give an account investigations of prolonged fever. (Jun 2007) 25) Write short note on def & causes of fever of unknown origin. (Aug 2013) 26) Write short note on diagnostic approach of fever of unknown origin. (Aug 2013, Nov 2007) 27) Give an account on causes & diagnosis of fever of unknown origin. (Sep 2011) 28) Def., causes & diagnostic approach of pyrexia of unknown origin. (Aug 2012) 29) Give short note on fever with splenomegaly. (Sep 2010) 30) Give short note on fever with rash. (Jul 2009)

93


Physical Medicine 1) 2) 3) 4) 5) 6) 7) 8) 9)

Musculoskeletal manifestations of SLE. (Dec 2011) Give an account on physical management of peripheral nerve injury. (Aug 2015) Enumerate 5 indications of electrodiagnosis. (Aug 2015) Enumerate 5 causes of Mono-articular arthritis. (Sep 2015) Classification of osteoporosis and its imaging studies. (Nov 2015) Describe clinical and radiological manifestations of ankylosing spondylitis. (Sep 2014) Describe types of juvenile idiopathic arthropathy and its management. (Dec 2014) Describe the etiopathogenesis and treatment of osteoarthritis. (Aug 2013) Describe musculoskeletal manifestations of systemic lupus erythematosus. (Nov 2013) 10) DD of Ankylosing Spondylitis. (Dec 2011) 11) Cryotherapy rehabilitation practice. (Dec 2011) 12) New criteria for diagnosis of rheumatoid arthritis. (Nov 2015, Sep 2011) 13) Types & physiological effects of thermotherapy. (Sep 2011) 14) Clinical manifestation of psoriatic Arthritis. (Sep 2011) 15) Diagnostic criteria of Systemic Lupus Erythromatosis. (Sep 2010) 16) Management of Bell's palsy. (Nov 2015, Sep 2010) 17) Rehabilitation program for a patient with peripheral nerve injury. (Sep 2010) 18) Clinical picture of Ankylosing Spondylitis. (Nov 2010) 19) Management of spasticity. (Nov 2010) 20) Methods of Electrodiagnosis. (Nov 2010)

94


Dermatology & Sexual transmitted infections 1) 2) 3) 4) 5) 6) 7) 8) 9) 10) 11) 12) 13) 14) 15) 16) 17) 18) 19)

Mention papulosquamous skin diseases. (Sept 2015) Enumerate causes of organic erectile dysfunction. (Sept 2015) Give short account on clinical varieties of scabies. (Sept 2015) (Nov 2013) Give short account on systemic and local treatment of acne vulgaris. (Nov 2015) (Sept 2015) Give an account on treatment of scabies. (Nov 2015) (Aug 2012) Enumerate causes of cicatricial alopecia. (Nov 2015) (Sept 2014) Enumerate clinical variants of psoriasis. (Nov 2015) (Sept 201 Treatment of chronic generalized psoriasis. (Sept 2014) (Nov 2013) Management of chancre. (Sept 2014) Clinical types and treatment of herpes zoster. (Nov 2014) Primary lesion and complications of lichen planus. (Nov 2014) Clinical manifestations of gonorrhea in female. (Nov 2014) Describe the clinical features of tuberculoid leprosy. (Aug 2013) Enumerate the differential diagnosis of a circinate skin lesion. (Aug 2013) Characterize the clinical manifestations of secondary stage of syphilis. (Aug 2013) (Sept 2011) Describe clinical features and diagnosis of primary syphilis (chancre). (Nov 2013) Scaly versus black dotted tinea capitis (cause and diagnosis). (Aug 2012) Cutaneous manifestations and serological tests of AIDS. (Aug 2012) Clinical varieties and management of Tinea Capitis. (Sept 2011)

N.B. Radiology and Clinical pathology are MCQ questions so, no collections for them review any source for MCQ on these branches.

95



Tanta University Faculty of Medicine Time allowed: 3 hours

Examination for 5th year in Pediatrics 1st paper October 4, 2015

All questions should be tried. Q1) Discuss in details etiology clinical picture, laboratory diagnosis, differential diagnosis and treatment of iron deficiency anemia. (20 marks) Q2) Give short account about each of the followings: 1. Clinical types and treatment of brachial palsy. 2. .Pathophysiological mechanisms and treatment of persistent diarrhea. 3. Doses, route of administration and duration of anti-hypertensive drugs. 4. Manifestations and etiology of laryngeal obstruction. 5. Etiology and pathogenesis of acute renal failure.

(50 marks)

Q3) Give short answer for each of the followings: 1. Treatment of constipation 2. Causes of cholestasis. 3. Ethical issues in neonatal resuscitation. 4. Indications of plasma exchange. 5. Indications of anti-convulsant drug monitoring.

(25 marks)

Q4) Problem solving: (27 marks) 1. One day aged full term baby presented with jaundice. The investigations revealed, total bilirubin 15mg/dL, Hb 7mg/dL, reticulocyte count 15%. His blood group was B RH +ve and his mother blood group was A RH -ve. A- What’s the possible diagnosis? B- What’re the normal values of the mentioned laboratory investigations? 2. Seven years old child presented with severe abdominal pain, maculopapular purpuric skin eruption in the buttocks and lower extremities. Laboratory investigations revealed normal complete blood count, normal ESR and elevated ASOT titer. A- What’s the most acceptable diagnosis? B- Give two possible complications? 3. A new born infant develops respiratory distress immediately after birth. His abdomen is scaphoid. No breath sound are heard on the left side of his chest, but they’re audible on the right. Immediate intubation is successful with little or no improvement. A- What’s the diagnosis? B- What’s the score used to assess respiratory distress?

97


4. Ten years old female child presented to the outpatient clinic with dark colored urine & puffiness of the eyelids. Physical examination revealed systemic hypertension, mild edema. Investigations revealed hematuria, granular casts and mild proteinuria. A- What’s the most acceptable diagnosis? B- What’re the emergency complications that may occur in this case? 5. Ten months old girl presented with fever, vomiting and diarrhea. On examination, she was lethargic, cold mottles skin with peripheral cyanosis. The skin pinch goes back very slowly. The eyes were very sunken, tears were absent and tongue was very dry. A- What’s the degree of dehydration in this girl? B- What’s the treatment of choice? 6. Four years old child presented in the emergency room with generalized tonic-clonic convulsion lasting 7 minutes. Examination revealed, temperature 40.4 oC, tonsillitis and no abnormal neurological signs. No family history of epilepsy. A- What’s the diagnosis? B- Lines of treatment of this condition? 7. A 4-year old boy presented to the outpatient clinic with abdominal pain, vomiting and yellowish discoloration of the skin and mucous membrane. Examination revealed enlarged tender liver, jaundice. Investigations revealed: elevated liver enzymes, total serum bilirubin was 15.5 mg/dL with a direct component of 6.5 mg/dL. A- What’s the possible diagnosis? B- Mention two non-invasive diagnostic tools in the diagnosis of this condition. 8. A female patient aged 12 years presented by weakness, loss of weight since 6 weeks. On examination, temperature orally was 39 oC, pallor and hepatosplenomegaly. Complete blood count demonstrate, Hb= 5.9 g/dL, white blood cells count 28.0 * 109 /L, platelets count 35*109 /L. A- What’s the diagnosis? B- Explain two causes of anemia. 9. A 4-year old child was referred to hospital because of sudden onset of respiratory distress and dysphagia. The child is feverish and toxic. He sits tripod. Leaning forward with open drooling mouth and slightly protruded tongue. There’s copious mucous saliva. A- What’s the most likely diagnosis? B- What’s the causative agent? Q5) MCQs (Choose only one answer): (28 marks) 1. The role of captopril in treatment of congestive heart failure in children is: a. Inotropic. b. Decrease preload. c. Decrease afterload. d. All of the above.

98


2. All of the following statements are true as regarded to pathologic jaundice EXCEPT: a. Unconjugated bilirubin level > 12.9 mg/dL in term infants. b. Unconjugated bilirubin level > 15 mg/dL in preterm infants. c. Jaundice in the first day of life. d. Elevation of the serum unconjugated bilirubin during the first week of life and resolve spontaneously. 3. Infant diabetic mother liable to the following complications EXCEPT: a. Hypoglycemia. b. Hypothermia. c. Cardiac anomalies. d. Respiratory distress syndrome. 4. Abdominal distension in an infant with acute diarrhea may be suggestive of all of the following EXCEPT: a. Hypokalemia. b. Paralytic ileus. c. Intussusception. d. Hyperkalemia. 5. The following are modes of transmission of hepatitis C EXCEPT: a. Intravenous drug use. b. contaminated food and water. c. Occupational and sexual exposure. d. Blood transfusion. 6. Stridor is a characteristic feature of all of the following diseases EXCEPT: a. Laryngeal diphtheria. b. Angioneurotic edema. c. Adenoid hypertrophy. d. Bilateral abductor vocal cord paralysis. 7. Common triggers of asthma in the children include all of the following EXCEPT: a. Tobacco smoking. b. cold air. c. Pollens. d. gelatin. 8. Thrombocytopenia has recognized association with the following EXCEPT: a. Henoch-Schonlein. b. TP. c. Aplastic anemia. d. Hypersplenism. 9. Wering-Hoffman diseases is characterized by the following EXCEPT: a. Hypotonia. b. Tongue fasciculation. c. Absent deep tendon reflex. d. Pseudobulbar palsy. 10. All of the following drugs can cause constipation EXCEPT: a. Vincristine. b. Streptomycin. c. Morphine. d. Antidepressants. 11. All of the following viruses are causes of acute bronchiolitis of infancy EXCEPT: a. Rota virus. b. Adenovirus. c. parainfluenza. d. Respiratory syncytial virus. 12. All of the following statements are true as regard to the normal urine EXCEPT: a. Contains protein up to 4mg/m2/hour. b. Amber yellow in color. c. Urine volume in children ranges from 1.5 - 3.5 cc/kg/h. d. Urine specific gravity ranges from 1030 - 1040. 13. All of the following statements are true as regard to the typical febrile convulsion EXCEPT: a. Generalized tonic-clonic. b. Duration from few seconds to 10 minutes. c. Abnormal EEG in between the attacks.

99


d. Age of presentation from 9 months to 5 years.

100


Examination for 5th year in Pediatrics 2nd paper October 11, 2015

Tanta University Faculty of Medicine Time allowed: 2 hours

Q1) Discuss in details the common behavioral disorders.

(20 marks)

Q2) Give short account about each of the followings: 1. Recurrence risk of Down syndrome. 2. .Fluid therapy in diabetic ketoacidosis. 3. Non-vitamin D deficiency rickets. 4. Etiological classification of fever. 5. Indications of hepatitis B vaccine.

(30 marks)

Q3) Give short answer for each of the followings: 1. Stages of human growth and development. 2. Contraindications of breast feeding. 3. Treatment of amoebiasis. 4. Prognosis of comatosed child. 5. Period of recommended isolation of common viral infections.

(20 marks)

Q4) Problem solving: (15 marks) 1. An infant who sits with only minimal support, attempts to attain a toy beyond reach, and rolls over from supine to prone position, but doesn't have a pincer grasp. A- What’s the developmental level of this child? B- What’s the suspected normal weight and head circumference of this child? 2. The parents of a 6-month-old boy out that he has just cut his first tooth, and wonder which teeth will come next. A- What’s the correct sequence for eruption of deciduous teeth? B- What’s the first permanent tooth will be appeared and at what age? 3. A 7-year old girl experiences a sudden onset of vesicles beginning first on the trunk and then spreading to the face and scalp. She gave a history of mild fever and anorexia before 1 day. The vesicular lesions are superficial. Some vesicles form a crust and the lesions are extremely pruritic. Later on the girl developed drowsiness, convulsions and bizarre movements. A- What’s the most likely diagnosis? B- What’s the treatment of this condition? 4. A 6-year old girl present with short stature and school under-achievement. Examination of the girl confirms a height well below the third centile and weight on the 25th centile. Bone age was delayed at 2 years of age. The parent reported roughness of her skin. Further follow up revealed a decrease in height.

101


A- What’s the most likely diagnosis?

102


B- What’re the investigations of this condition? 5. A two day old male infant is referred from a community hospital for bilious vomiting and a heart murmur. The baby was born at 37 weeks gestation to a G4P3 39 years old woman who had no prenatal care. Exam: temperature 37.1 (axillary), pulse 150, respiratory rate 45, blood pressure 75/50, oxygen saturation 99% in room air. Height, weight and head circumference are at the 5th percentile. He appears jaundiced and has a flat facial rofile; short upslanting palpebral fissures and single palmar creases. His lungs are clear to auscultation. His heart is tachycardic with a loud holosystolic murmur. His abdomen is non-distended. Generalized hypotonia is present. An abdominal radiograph show a “double-bubble sign”. A- What’s the diagnosis? B- What’re the investigations needed in this case? 5) MCQs (Choose only one answer): (20 marks) 1. Head circumference at birth is: a. 30 cm. b. 35 cm. c. D37 cm. d. 39 cm. 2. All these milestones occur at the age of 9 months EXCEPT: a. Crawling on abdomen. b. Primitive pincer grasp. c. Waves Bye Bye. d. Rolls from supine to prone position. 3. Following successful BCG vaccination, small purple forms, enlarges crusts and disappears in: a. 2 weeks. b. 4 weeks. c. 4-8 weeks. d. 16 weeks. 4. Major reactions to DPT include the following EXCEPT: a. High fever > 40.5 oC for > 4hrs. b. Irritability. c. Hepatitis. d. Shock like state. 5. All of the following are correct as regards colostrum EXCEPT: a. It’s secreted from breast for 4 days after birth. b. It’s amount per day is 10-40 ml. c. It’s deep lemon yellow in color. d. Its reaction is more acidic than mature breast milk. 6. Goat milk is low in: a. Calcium. b. Phosphorus. c. Unsaturated fatty acids. d. Folic acid. 7. Inheritance pattern of hereditary spherocytosis is: a. Autosomal dominant. b. Autosomal recessive. c. Sex-linked recessive. d. Sex-linked dominant. 8. Koplik’s spots may be seen in buccal mucosa in prodromal period of: a. Measles. b. German measles. c. Chicken pox. d. Scarlet fever. 9. Dose of atropine in resuscitation is: a. 0.002 mg/kg b. 0.02 mg/kg. c. 0.2 mg/kg d. 2 mg/kg.

103


10. Neonatal screening for congenital hypothyroidism may miss: a. 1% of cases. b. 5% of cases. c. 10% of cases. d. 20% of cases.

104


Examination for 5th year in Pediatrics 1st paper November 24, 2015

Tanta University Faculty of Medicine Time allowed: 3 hours

Q1) Discuss in details etiology, types, diagnosis, prognosis and treatment of neonatal seizures. (20 marks) Q2) Give short account about each of the followings: 1. Status epilepticus. 2. Transformation modalities. 3. Acquired complete atrio-ventricular block. 4. Prevention of tuberculosis. 5. Etiology and epidemiology of hepatitis B.

(50 marks)

Q3) Give short answer for each of the followings: 1. Modified oral rehydration solution. 2. Predisposing factors of urinary tract infections. 3. Tests for evaluation of hemostatic mechanisms. 4. Aphthous ulcers. 5. Classification of congestive heart failure.

(25 marks)

Q4) Problem solving: (27 marks) 1. An 11 years old girl had fever, abdominal pain, anorexia, vomiting and yellowish discoloration of the sclera. Serum ALT= 745 U/L, Serum AST= 850 U/L, Total serum bilirubin= 15mg/dL, with direct bilirubin= 11 mg/dL. PT= 24 secs. A- What’re the normal values? B- What’s the diagnosis and how to confirm it? 2. A 4 years old child was presented with generalized edma, ascites. Investigations revealed: Urinary output= 0.4 cc/kg/h, Urinary protein= 65 mg/m2/hr, Serum albumin= 1.4 g/dL and serum cholesterol= 350 mg/dL. A- What’re the normal values? B- What’s the diagnosis? 3. A 7 years old bog presented with maculopapular purpuric eruption mainly on lower limbs, no pallor, no organomegally, microscopic hematuria and arthritis. Investigations showed: RBCs= 4.5*106/cm3, platelets= 220*103/cm3, WBCs= 8.5*103/cm3 and reticulocytes= 1%. A- What’re the normal values? B- What’s the probable diagnosis? 4. A 9 m infant, 8 kg, presented with severe dehydration after 4 days of watery diarrhea, Investigations revealed: serum sodium= 126 mEq/L, Serum potassium= 1.2 mEq/L, Aeterial

105


blood

106


pH= 7.15, urine volume= 120 ml/day. A- Mention the normal values. B- Mention the complications that may ocur for this infant. 5. A full term neonate, 6 days old, 3.6 kg born SVD after premature rupture of membrane for > 36 hrs, was noticed to be inactive and have poor suckling, jaundice and mottled skin. Investigations revealed: PMNLs= 1.6*103/mm3, Platelets= 65*103/mm3, Total serum bilirubin= 17 mg/dL and CRP= 48 mg/dL. A- What’s the diagnosis? B- Mention 3 important complications? 6. A 3 years old girl was presented with fever, vomiting, abdominal pain, dysuria, her urine analysis revealed: 24 hrs urinary protein: 15mg mg/m2/hr, pus cells= 70-80 cells/HPF, RBCs= 20-30 cells/HPF. A- What’re the normal values? B- What’re the investigations? 7. Four years old child presented in emergency room with generalized tonic clonic convulsions lasting 7 minutes. Examination revealed: temperature 40.4 oC, tonsillitis, and no abnormal neurological signs. No family history of epilepsy. A- What’s the diagnosis? B- Lines of treatment of this condition? 8. A 9 years old girl with bronchial asthma. On average she uses salbutamol inhaler 3 times per week, but for the past 10 days, she has wheezing both day and night and is using inhaler 3-4 Times per day. On examination, you note working alae nasi O2 saturation is 90%. A- Classify according to severity of paroxysm? B- Mention two mandatory investigations to this girl. 9. A 10 years old girl with rheumatic heart disease, presented with palpitation, temperature 37.8 oC, heart rate 120/minute, pallor and heart examination revealed ansystolic murmur on the apex propagated to the axilla. Her ESR was 50/90 mm. A- What’s the most acceptable diagnosis? B- Give two possible complications. Q5) MCQs (Choose only one answer): (28 marks) 1. The following are modes of transmission of hepatitis C EXCEPT: a. Intravenous drug use. b. Contaminated food and water. c. Occupational and sexual exposure. d. Blood transfusion. 2. All of the following statements are true as regard to pathologic jaundice EXCEPT: a. Unconjugated bilirubin level > 12.9 mg/dL in term infants. b. Unconjugated bilirubin level > 15 mg/dL in preterm infants. c. Jaundice in the first day of life.

107


d. Elevation of the serum unconjugated bilirubin during the first week of life and resolve spontaneously. 3. The following cells release a diverse array of mediators of allergic inflammation: a. Eosinophils. b. Basophils. c. Mast cells. d. Th2 cells. 4. Abdominal distension in an infant with acute diarrhea may be suggestive of all of the following EXCEPT: a. Hypokalemia. b. Paralytic ileus. c. Intussusception. d. Hyperkalemia. 5. Patency of ductus arteriosus depends on: b. Placenta prostaglandins. a. Low PO2. c. Both of them. d. None of them. 6. Pulmonary tuberculosis may present with all of the following EXCEPT: a. Bronchopneumonia. b. Pulmonary cavitation. c. Chronic pneumonia. d. Acute lobar pneumonia. 7. The followings are manifestations of pulmonary venous congestion EXCEPT: a. Palpitation. b. Dyspnea. c. Cough. d. Tachypnea. 8. Thrombocytopenia has recognized association with the following EXCEPT: a. Henoch-Schonlein purpura. b. ITP. c. Aplastic anemia. d. Hypersplenism. 9. Possible causes of convulsions in child with nephrotic syndrome include: a. Meningoencephalitis. b. Hypertensive encephalopathy. c. Stroke. d. All of the above. 10. Werdnig-Hoffman disease is characterized by the following EXCEPT: a. Hypotonia. b. Tongue fasciculation. c. Absent deep tendon reflex. d. Pseudobulbar palsy. 11. The normal developmental quotient (DQ) is: a. Above 50. b. Above 60. c. Above 70. d. Above 80. 12. Causes of microcytic hyphochromic anemia are al of the following EXCEPT: a. Sickle cell anemia. b. Iron deficiency anemia. c. Lead poisoning. d. Thalassemia. 13. All of the following statements are true as regard to the normal urine EXCEPT: a. Contains protein up to 4 mg/m2/hr. b. Amber yellow in color. c. Urine volume in children range from 1.5 - 3.5 cc/kg/hr. d. Urine specific gravity ranges from 1030 - 1040. 14. The following are types of simple partial seizures EXCEPT: a. Motor. b. Sensory. c. Automatic. d. Atonic.

108


Examination for 5th year in Pediatrics 2nd paper November 30, 2015

Tanta University Faculty of Medicine Time allowed: 2 hours

Q1) Discuss in details diagnostic approach, treatment and prognosis of the comatose child. (20 marks) Q2) Give short account about each of the followings: 1. Counseling of the parents of children with behavioral problems. 2. .Acquired hypothyroidism. 3. Preparations available for passive immunity. 4. Dried powdered milk. 5. Special vaccinations.

(30 marks)

Q3) Give short answer for each of the followings: 1. Etiology of encephalitis. 2. Teratogenesis. 3. Daily vitamin requirements for infants and children. 4. Non nutritional marasmus. 5. Deciduous and permanent teeth.

(20 marks)

Q4) Problem solving: (15 marks) 1. Female infant aged one year, stopped breast feeding and fed surgery fluids. She’s 7 kg body weight with anorexia, generalized edema, muscle wasting and slin manifestations. A- What’s the diagnosis? B- What’s the normal weight, length and head circumference for this age? 2. Male child aged 10 years developed in the last months polyuria, polyphagia, multiple skin abscesses and loss of weight. He’s now presented to emergency room with disturbed consciousness, rapid deep breathing and dehydration. A- What’s the original disease? B- What’s the complication developed? 3. Down syndrome child aged 5 years developed severe pallor, purpuric spots and hepatosplenomegaly. A- What’s the complication occurred? B- How to diagnose Down syndrome? 4. Newborn infant sleeps much, cries little and chocking occurs during feeding, he has constipation and prolonged jaundice. A- What’s the diagnosis? B- Mention the required investigations.

109


5. Male child aged 11 years developed mild fever, headache and malaise for one day, then developed swelling between mastoid and posterior border of the mandible and extends under the back of the ear lobule. A- What’s the diagnosis of this disease? B- What’re the important complications for this disease? Q5) MCQs (Choose only one answer): (20 marks) 1. Social smile occurs at the age of: a. one month. b. Two months. c. Three months. d. Four months. 2. Normal mid-arm circumference in children 1-5 years old is: a. Less than 12.5 cm. b. Less than 13.5 cm. c. Between 13.5 and 14.5 cm. d. Above 14.5 cm. 3. All of the following are correct regarding enuresis except: a. Punishment is not effective. b. Encouragement is very effective. c. The effect of drugs is temporary. d. The problem will continue by time. 4. Contraindications for MMR vaccine are the following except: a. Minor forcible illness. b. Infants recently received immunoglobulin or blood. c. Immunocompromised infants. d. History of anaphylaxis to neomycin or eggs. 5. BCG vaccine: a. Live attenuated vaccine. b. Killed vaccine. c. Toxoid. d. Non of the above. 6. Prebiotic is: a. Beneficial live microorganisms. b. Non-digestible food stimulates growth of commensal bacteria. c. Killed bacteria. d. Bacteria producing antibiotics. 7. Ber-beri is caused by deficiency of: a. Vit A1. b. Vit B1. c. Vit B2. d. Vit C. 8. Antidote to opiates is: a. Sodium nitrate. b. Atropine. c. Naloxone. d. Desferoxamine. 9. The proper dose of thyroxin for a newborn with hypothyroidism is: a. 15 mg/kg/day. b. 15 15 ug/kg/day. c. 15 15 mg/day. d. 15 15 ug/day. 10. In sex linked recessive diseases, the pedigree is: a. Horizontal. b. Vertical. c. Oblique. d. Sporadic.

110


Examination for 5th year in Pediatrics 1st paper October 27, 2014

Tanta University Faculty of Medicine Time allowed: 3 hours

Q1) Discuss in details Diagnostic criteria, assessment, grades, etiology, investigations, prevention and treatment of mental retardation. (20 marks) Q2) Give short account about each of the followings: 1. Stomatitis. 2. Pneumothorax. 3. Apheresis. 4. Definition and etiology of infective endocarditis. 5. Patterns, etiology and prognosis of neonatal seizures.

(50 marks)

Q3) Give short answer for each of the followings: (25 marks) 1. Investigations of urinary tract infections. 2. Indications of anti-convulsant drugs monitoring. 3. Indications of immediate exchange transfusion in hemolytic disease of the newborn. 4. Mid-night croup. 5. Etiology and epidemiology of hepatitis E. Q4) Problem solving: (27 marks) 1. A 24 hour term infant presented with jaundice. The baby’s blood group was A Rh +ve while the mother’s wae O Rh -ve. His laboratory studies reveals the following: total serum bilirubin= 19 mg/dL, direct bilirubin= 0.8 mg/dL and CRP 4mg/l . A- What’re the normal values? B- What’s the diagnosis? 2. A newborn infant develops respiratory distress immediately after birth. His abdomen is scaphoid. No breath sounds are heard on the left side of his chest, but they’re audible on the right. Immediate intubation is successful with little or no improvement. A- What’s the diagnosis? B- What’s the score used to assess respiratory distress? 3. A 11 years old boy with a history of diarrhea since three days presented to the emergency department with a-6 hour history of vomiting. Colicky abdominal pain, irritability with passage of currant jelly stool. On examination a sausage-like mass is palpable in the right upper quadrant of the abdomen. A- What’s the most likely diagnosis? B- What’s the most appropriate next step? 4. A 9 years old girl presented with a complaint of nausea, poor appetite, and fever for one week. She had non-bilious vomiting, itching and abdominal pain on the right side. The

111


color of urine is darker than normal with pale stool. On examination, her skin is jaundiced and the liver is moderately enlarged and tender with sight enlargement of the spleen. A- What’s the most likely diagnosis? B- What’re the suspected laboratory abnormalities in this condition? 5. A 4 years old child with a history of running nose, cough and fever since three days. then he presented to the hospital with difficult breathing. On examination, he is alert, in moderate distress with decreased air entry over right lobe. His blood count shows, white blood cells 20,000 with 70% segmented and 11% bands. A- What’s the most probable diagnosis? B- What’s the proper treatment needed? 6. Child aged 3 years presented with bruising in body, pallor, hepatosplenomegaly and lymphadenopathy. His laboratory investigations reveal RBCs count 2.5 million/mm3, lymphoblast in peripheral blood: 1% and platelet count: 2000/mm3. A- What’re the normal values? B- What’s the diagnosis? 7. An 8 years old male present with complaint of dark cola colored urine, puffiness of the face abdominal pain for the past 2 days. He had a sore throat and fever 2 weeks. He is also complaining of headache and blood pressure of 130/90. Urine analysis shows that RBCs are too numerous to count. Serum complement C3 level is low. A- What’s the most likely diagnosis? B- What’re the suspected laboratory investigations abnormalities in this condition? 8. A 3 months old infant girl is brought to the clinic with severe weakness and marked hypotonia, limb weakness, poor feeding and weak cough and cry. The tendon reflexes are absent and there’s a visible tongue fasciculations. She has normal mentality. A- What’s the most likely diagnosis? B- What’s the investigation needed for confirmation of the diagnosis? 9. A 7-month-old infant gets episodes of intermittent cyanosis and breathlessness which is worse during feeding. Examination show minimal cyanosis around the lips. On auscultation, there is a grade III ejection systolic murmur maximally heard in the second and third left intercostal spaces with single second heart sound. A chest X-ray shows boot-shaped heart. A- What’s the most probable diagnosis? B- What’re the investigations needed for this infant? Q5) MCQs are not included.

112


Examination for 5th year in Pediatrics 1st paper December 28, 2014

Tanta University Faculty of Medicine Time allowed: 3 hours Q1) Discuss in details neonatal birth injuries.

(20 marks)

Q2) Give short account about each of the followings: 1. Serofibrinous pluerisy. 2. Clinical manifestations and diagnosis of generalized malabsorption. 3. Transfusion modalities. 4. Types, doses and route of administrations of anti-hypertensive drugs. 5. Clinical presentations and treatment of acute renal failure.

(50 marks)

Q3) Give short answer for each of the followings: 1. Intractable epilepsy. 2. Criteria of severity of asthma paroxysm. 3. Etiology and classification of pulmonary valve stenosis. 4. Criteria for diagnosis of flobby infant syndrome. 5. Idiopathic neonatal hepatitis.

(25 marks)

Q4) Problem solving: (27 marks) 1. A 5-week-old present to the hospital with vomiting after each feeding during the last two weeks. His birth weight is 2.750 kg. The mother stated that the vomiting is projectile, non-bilious but she feels that he has a good suck and swallow. She observed that the infant after vomiting is hungry and want to feed again. Examination revealed firm olive-like mass felt to the right of umbilicus. A- What’s the most appropriate diagnosis? B- What’s the treatment of this condition? 2. A- three day-old term infant born at home exclusively breast-fed, presented to the neonatatology unit with lethargy, bulging anterior fontanel and bright red blood from rectum. Laboratory investigations revealed prolonged PT and PTT and clotting time with normal bleeding time and normal capillary fragility. Platelet count was 350,000/mm3. A- What’s the most probable diagnosis? B- What’s the treatment of this condition? 3. A 6-year-old boy present with history of cough, fever, wheeze and weight loss since weeks. Examination reveals audible rhonchi over the right side of the chest. Chest X-ray shows enlarged right hilar lymph nodes and a circular mass 1 cm in diameter in the lung parenchyma, adjacent to the pleura in the right midzone. A- What’s the most appropriate diagnosis? B- What’s the treatment of this condition?

113


4. A 7 years old girl with a body weight of 26 kg presents with tender and swollen left knee as well as more recently appearing swollen right wrist. She also had fever. There are firm, painless, nonpurpuric, mobile nodules was observed on the extensor surface of joints, scalp and spine. Laboratory investigations reveals ESR= 70 in the first hour, CRP: 24mg/L, ASO titer: 800 IU/ml and ECG show prolonged P-R interval. a- What’s the most appropriate diagnosis? b- What’s the treatment of this condition? 5. A previously well 4-year old boy was brought to hospital with nausea, vomiting, his parents noticed that his eyes looked yellow with dark red urine. He has recently been treated with co-trimoxazole for urinary tract infetion. AST: 25u/L, ALT: 20 u/L hemoglobin electrophoresis- sand osmotic fragility are normal His hemoglobin is 7.1 g/l. A- What’s the most appropriate diagnosis? B- What’s the treatment of this condition? 6. A 15 months old boy presents with tonic-clonic convulsions. He has been in good health except for high fever that developed today to about 38 C. He has a slight cough and mild nasal congestion. Just prior to the seizure, he was playing with some toys. Past medical history is unremarkable. Examination shows congested throat mucosa and congested ear drums. Neurological examination is normal. A- What’s the most likely diagnosis? B- What’s the treatment of this condition? 7. A 5 month-old boy presents in winter with a day history of fever, rhinorrhea and cough. Since last night, she is unable to feed with increasing rate of breathing. On examination: temperature 38.5 oC, RR: 60/m, with intercostal reactions, audible wheezing and decreased aeration at lung bases. Chest X-ray show bilateral interstitial infiltrate and hyperinflation. A- What’s the most likely diagnosis? B- What’re the complications of this condition? 8. A 4-year old boy present with a complaint of swollen face, vomiting, mild diarrhea and abdominal pain. On examination: heart lung and abdominal findings are normal. His hands and feet are quit edematous. A urine test is markedly positive for protein but negative for blood. Blood analysis reveals mild anemia, blood urea: 35 mg/dL, serum creatinine: 0.8 mg/dL, serum cholesterol: 350 mg/dL, serum calcium: 7 mg/dL and normal serum levels of C3. A- What’s the most likely diagnosis? B- What’re the complications of this condition? 9. A 3-year old boy from rural area with poor socioeconomic background presented with nauseas, vomiting, abdominal distension, mild discoloration of the sclera. Examination reveals hepatomegaly, ascites, dilated anterior abdominal wall veins. No edema in lower limbs and no palpable spleen. Investigations were done and revealed normal liver function tests. Abdominal ultrasonography shoed enlarged liver, narrowed hepatic veins and massive ascites. Liver biopsy show central lobular congestion. A- What’s the most likely diagnosis? B- What’s the treatment of this condition? Q5) MCQs are not included.

114


Examination for 5th year in Pediatrics 1st paper October 3, 2013

Tanta University Faculty of Medicine Time allowed: 3 hours

Q1) Discuss the etiology, predisposing factors, clinical presentations, laboratory investigations and treatment of urinary tract infection in infants and children. (20 marks) Q2) Give short account about each of the followings: 1. Infantile spasms. 2. Treatment of neonatal sepsis or meningitis. 3. Persistent diarrhea (definition, clinical importance and pathophysiology). 4. Sydenham’s syndrome. 5. Laryngeal obstruction (manifestations and etiology).

(50 marks)

Q3) Give short answer for each of the followings: 1. Enumerate the indications of nasogastric rehydration. 2. Mention the etiology of secondary aplastic anemia. 3. Mention the clinical picture of supraventricular tachycardia. 4. Describe the features of ataxic cerebral palsy. 5. Mention differential diagnosis of congenital hypertrophic pyloric stenosis.

(25 marks)

Q4) Problem solving: (27 marks) 1. You’re evaluating a three hour old infant with respiratory distress. The infant has cyanosis in the room air, severe retraction, audible grunting without stethoscope, barely audible air entry and respiratory rate 90/min. A- Mention the name of the score used to assess respiratory distress? B- Calculate the degree of the score in this infant and its meaning? C- Mention two mandatory investigations in this case. 2. A 9-month boy presented with acute diarrhea for the last 5 days. He showed signs of some dehydration. After clinical examination the physician prescribed ORS to be given to the baby. The mother did not find ORS and gave light tea and rice water instead for rehydration. The patient became more unwell and at arrival to hospital he was shocked. A- Explain why the patient had deteriorated like. B- What disturbances do you expect to occur in this patient? C- Mention 3 manifestations done later to standard WHO ORS formula. 3. A 3-year old male child was referred to hospital because of sudden onset of respiratory distress and dysphagia. The child is feverish and toxic. He sits in the bed leaning forward with open drooling mouth, and slightly protruded tongue. There’s copious mucous saliva. A- What’s the most likely diagnosis? B- What’s the common causative organism? C- What’s the emergency treatment?

115


4. An 8 years old girl complains for 4 days history of low back pain. There was a recent history of viral infection. On examination you noticed bilateral symmetrical weakness in her lower limbs and absent deep tendon reflexes. Sensation showed glove and socks hypothesia, EMG and MRI spine are normal. a- Mention the most likely diagnosis? b- Differential diagnosis. c- Two mandatory investigations. 5. An 8 yrs old boy has unexplained weight loss since 6 mo. with unexplained recurrent fever (>38 oC) with excessive night sweat. He has enlarged, painless, firm and discrete cervical and axillary lymph nodes. There’s moderately hepatosplenomegaly. Chest X-ray showed mediastinal mass. A- What’s the most likely diagnosis? B- What’s the most important investigation needed? C- What’re the different lines of treatment? 6. An 11 yrs old girl, had upper respiratory infection and 2 weeks later developed abdominal pain, arthralgia and rectal bleeding. She has non itchy maculopapular rash with ecchymoses and purpura. She has platelet count of 370*103/cm3, PT of 12 secs and APTT of 32 sec. A- What’s the most propable diagnosis? B- Mention a further important investigation? C- Does she need corticosteroid, why? 7. A ten years old full term boy, has been circumcised and bleeding didn’t stop although he 3 3 had received vitamin K1 immediately after birth. Platelet count was 250*10 /cm , APTT of 65 sec, clotting time of 18 min, bleeding time of 5 min, and PT of 12 sec. A- What’s the diagnosis? B- How to confirm your diagnosis? C- Mention the different methods of prenatal diagnosis. 8. An 8-month old infant presented with respiratory distress, her mother reported that her baby has nasal discharge in the last three days with fever and anorexia. The past medical and family histories are unremarkable. Examination revealed marked tachypnea, working alae nasi, chest retraction with audible bilateral wheezes. CBC showed WBCs 8*103/cm3 with 72% lymphocytes and chest X-ray showed diffuse hyperinflation. A- Mention the possible diagnosis. B- Mention the most common organisms. C- Mention differential diagnosis. 9. A girl 7 yrs old had recurrent attacks of sudden stopping her voluntary acts, with starring and rapid blinking of the eyes. Each attack lasts about 30 sec. She didn’t fall on the ground during any attack. She has poor school performance. A- What’s the most probable diagnosis? B- Mention the proper drug for this case. C- What’s the prognosis?

Q5) MCQs are not included.

116


Examination for 5th year in Pediatrics 2nd paper August 17, 2010

Tanta University Faculty of Medicine Time allowed: 2 hours

Q1) Discuss management of bacterial meningitis in infants and children.

(20 marks)

Q2) Give short account about each of the followings: 1. Metabolic coma. 2. .Imaging studies in trisomy 21. 3. Active prevention of hepatitis B. 4. Vitamin A deficiency and excess. Q3) Give short answer for each of the followings: 1. Enumerate ossification centers present at birth. 2. Enumerate 4 anti-infective properties of breast milk. 3. Enumerate warning signs of dehydration. 4. Enumerate 4 diagnostic tools of portal hypertension in pediatrics. 5. Enumerate 8 monitoring data in pediatric shock. Q4) Problem solving: 1. 10 months old infant presents with one day history of confluent maculopapular rash which started on his face and now covers his entire body. He is miserable with conjunctivitis and fever of 39.5 oC. The illness started with runny nose and cough 5 days ago. Q1- What’s the most likely diagnosis? A. Scarlet fever. B. Measles. C. German measles. D. Chicken pox. Q2- Incubation period of this disease is: A. 10 +- 2 days. B. 5 +- 2 weeks. C. 5 +- 2 days. D. 8 +- 2 weeks. 2. 18 months old boy came to the emergency department with rapid respiration and drowsiness. He had a history of vomiting and diarrhea for 3 days before the onset of his condition. By examination, he was severely dehydrated. HR was 140/min, RR was 60/min, temperature was 38 oC and blood pressure was 60/40. Capillary refill time was 6 seconds. Q1- What’s the most likely diagnosis? A. Early compensated hypovolemic shock. B. Advanced decompensated hypovolemic shock. C. Established hypovolemic shock. D. Irreversible hypovolemic shock. Q2- What’s the most urgent action to be done: A. Urgent X-ray chest. B. start inotropes. C. Blood gases & electrolytes levels. D. Administration of IV bolus normal value.

117


3. An infant can move his head from side to side while following moving object, can lift his head from prone position 45 degrees off the examination table, and smiles when encouraged. He can sit with support. Q1. The most likely age of this infant is: A. 3 months. B. 9 months. C. 6 months. D. 2 months Q2. If this infant has manifestations of untreated congenital hypothyroidism the most likely age is: A. 3 months. B. 6 months. C. 9 months. D. 24 months. 4. 1 year old infant is complaining of delayed sitting and repeated chest infections, on examination there’s prominent costo-chondral junctions, he is exclusively breast fe. He received a lot of injections for treatment of this condition. Q1. Al of the following are early signs of this disease EXCEPT: A. Increased serum alkaline phosphatase. B. Chest beads. C. Macrocephaly. D. Radiologic findings of active disease. 5. 18 years old female was noticed to get gradually increasing edema of dorsum of both hands and feet, pallor, angular stomatitis and became irritable with poor appetite and lost smiling. Her weight 7 kg, last weight record was 9 kg 3 months ago. She has weaned mainly on starchy foods 2 months ago. She has mild hepatomegaly, ascites and no jaundice. Subcutaneous fat was preserved. Her serum K was 2.5 mEq/L, Na 131 mEq/L, blood sugar 59 mg/dL, plasma albumin 2 gm/dL and Hb was 8.3 g/dL. Q1. The most appropriate diagnosis of this case is: A. Nephrotic syndrome. B. Marasmic Kwashiorkor. C. Liver cirrhosis. D. Non of the above. Q2. The most characteristic biochemical change of this disease is: A. Hypokalemia. B. Hypoalbuminemia. C. Hyponatremia. D. Hypoglycemia. 6. A 10-year old boy is brought to the emergency department after being stuck by a car while riding his bicycle. Physical examination immediately upon arrival reveals that there’s no eye opening to both verbal and painful stimuli, he’s responsive to painful stimuli by non-specific sounds and decerebrate posture. Q1. His Glasgow coma score will be: A. 5. B. 6. C. 8. D. 9. Q2. The expected mortality rate in this patient will be: A. 90%. B. 70%. C. 50%. D. 10%. 7. 3 months old infant presented since birth with excessive regurgitation especially following feedings, with attacks of irritability, aching, chocking with failure to thrive. The patient was seized by one apneic attack relieved by stimulation. The patient is breast-fed. Q1. The diagnosis of this condition is: A. Milk allergy. B. Urinary tract infection.

118


C. Epilepsy. D. Gastro-esophageal reflux disease. Q2. To confirm your diagnosis, choose the most appropriate diagnostic tool: A. D-Xylose. B. EEG. C. Urine analysis. D. pH monitoring. 8. 12 months previously well-boy presents to the emergency department in shock with hypothermia. The parents gave a history of severe vomiting. with severe paroxysmal colicky pain that recurs at frequent intervals during the last 6 hrs. Palpation of the upper quadrant of the abdomen. On rectal examination, the baby passed red current jelly stools. Q1. Concerning this disease, all of the following are TRUE EXCEPT: A. The most common cause of intestinal obstruction between 3 months and 6 years of age. B. Barium enema is diagnostic. C. More common in males. D. D. In this patient, try hydrostatic reduction before attempting surgery. Q2. This emergency could complicate which of the following conditions: A. Inverted appendicular stump. B. Meckel’s diverticulum. C. Henoch-Schoenleir purpura. D. All of the above. 9. A 5-week-old boy presents to the clinic with vomiting for the last two weeks. The mother states that vomiting is projectile, non-bilious. After vomiting the patient is hungry and wants to feed again. This condition is progressive. He’s not gaining weight properly. Examination revealed an olive-like mass felt above and to the right of umbilicus. Q1. Regarding this condition, all of the following are TRUE EXCEPT: A. Surgery is the usual treatment. B. Infants often have a metabolic acidosis. C. There's a strong familial predisposition. D. Abdominal US examination is diagnostic. Q2. Other causes of vomiting in this age include all of the following EXCEPT: A. Gastroenteritis. B. Gastroesophageal reflux disease. C. Duodenal atresia. D. Hypothroidism. 10. A 4-week-old breast fed female baby has worsening jaundice, noted by parents 15 days ago with clay-colored stools and dark urine. On examination, she is well appearing with good sucking and reflex activities, and it’s noted to have a liver edge of 3 cm below costal margin. Total bilirubin is 12 mg/dL and direct bilirubin is 9 mg/dL. AST was 44 u/L, ALT was 47 u/L, serum alkaline phosphatase was 952 Bodanesky U/dL with elevated serum bile acids. Q1. All of the following are TRUE regarding this disease EXCEPT: A. Jaundice appears at birth, but may be delayed until 3-4 weeks of life. B. May be complicated by cirrhosis and portal hypertension. C. It’s a progressive disease. D. Kasai operation should be done at 6 months of age. Q2. Diagnostic investigations of this disease include all of the followings EXCEPT: A. Abdominal ultrasound. B. Operative cholangiogram. C. Liver biopsy. D. Non of the above.

119


Examination for 5th year in Pediatrics Pediatrics Department October 18, 2009

Tanta University Faculty of Medicine Time allowed: 3 hours

Q1) Long Assay: 1. Discuss genotypes, complications and counseling of Down syndrome. Q2) Short Assay: Give a short account on: 1. Sex-linked inheritance. 2. Pseudo seizures. 3. Treatment of hypertension. 4. Tracheomalaecia. 5. Critical pulmonary stenosis. Q3) Short answer: 1. Define mutagen. 2. Mention causes of chronic renal failure. 3. Enumerate the causes of pneumothorax in children. 4. Enumerate cardiac causes of syncope. 5. Define migraine. Q4) Problem solving: 1. A 7-year-old child is seen for evaluation of motor delay. The examination reveals limited voluntary movements, as well as slow writing continuous movements of the arms and legs when the patient initiates motor movements. At rest these movements disappears. A- What’s the best classificaltion of the child motor deficit? B- Mention causes of such problem. 2. A 6-year-old girl is brought to the clinic for a routine health maintenance visit. Her growth was normal until 2 years of age, when her height started to fall off of growth curve, steadily decreasing to below the fifth percentile. On examination, she has a webbed, a shield chest and scoliosis. A- Mention the phenotype and the most common genotype of this girl. B- What’s the most common cardiac disease that associated with this disorder? 3. A 8 years old girl, 28 kg weight, came to the pediatric ER with drowsiness, rapid deep breathing, irritability, her BP was 145/100, her mother gave mother a history of sore throat since 2 weeks, smoky urine since 2 days and scanty urine in the last 24 hrs. A- What’s the most acceptable diagnosis? B- Give 3 essential laboratory investigations for this girl. 4. A term male is born to a 35-year-old women after an uncomplicated pregnancy. Immediately after birth, the child is noted to have tachypnea, nasal flaring, grunting, retractions

120


and cyanosis. Apgar scores were 4 and 4 at 1 and 5 minutes respectively. Auscultation of the chest reveals right sided heart sounds and the neonate is noted to have scaphoid abdomen. A- What’s the most likely diagnosis? B- What’s the best management for this condition? 5. A 9 months old infant was admitted in ICU with heart rate of 240/minute and respiratory rate of 50/minute. There was no history of fever, diarrhea, vomiting or any illness. Then, he became very irritable with hepatomegaly, gallop rhythm, hypotension and faint peripheral pulse. Chest X-ray was free and ECG showed narrow QRS complexes. A- What’s the most likely diagnosis? And what’s the treatment? B- Is ECG important after treatment of the infant? Why?

121


Examination for 5th year in Pediatrics Pediatrics Department October 24, 2009

Tanta University Faculty of Medicine Time allowed: 3 hours Q1) Long Assay: 1. Up to date management of status epilepticus. Q2) Short Assay: Give an account on: 1. Gynecomastia. 2. Digitalis toxicity. 3. Extra hepatic portal hypertension. 4. Total parental nutrition. 5. Hyperleucocytosis.

Q3) Short answer: 1. Enumerate causes of hypocalcaemia. 2. Enumerate risk factors of cerebral edema complicating DKA management. 3. Mention causes of hepatomegaly with eosinophilia. 4. Enumerate causes of malabsorption syndrome. 5. Mention causes of congenital aplastic anemia. Q4) Problem solving: 1. A one day old full term neonate, born by CS for non-relative parents, presented by ambiguous genitalia. The infant has complete labial fusion and a phallus that resembles a small penis with hypospadias. No gonads were palpable. Blood pressure, pulse and respiration were normal. Serum electrolyte were: Na+ 133 mmol/L and K+ 3.65 mmol/L and random blood Sugar was 3.2 mmo/L. A- What’s your diagnosis? And what are the other types? B- What’s the suitable management? 2. Five years old girl, presented with acute onset of fever, right-sided clonic facial seizures. Level of consciousness was fluctuating with GCS ranging from 9-12 for 8 hrs, then the patient developed coma with GS 7. CT scan revealed neither hemorrhage nor edema and MRI revealed hyperdense focal lesion in the left temporal lobe. SF showed negative gram stain with negative latex agglutination tests for conventional bacteria. Chemistry revealed: protein (70 mg/dL. Glucose (60 mg/dL), concomitant blood glucose 100 mg/dL, white blood cell count 250 mm3 with 85% lymphocytes and RBCs count 135/mm3. A- What’s the diagnosis of this case? B- Mentions the lines of treatment. 3. A female baby was born by spontaneous vaginal delivery, the baby weight was 2.8 kg and was thought to be at 36 weeks’ gestation. Apgar score was 3 at birth and 10 at three minutes. Initial examination was normal and then breast feeding started. At 36 hrs she had bee

122


with feeds and had vomited at each feed. On examination: slight jaundice and liver was 2 cm below the costal margin in right mid-clavicular line. Spleen was not palpable. Serum bilirubin was 10. mg/dL. 12 hrs later the baby was seen again because vomiting and diarrhea. On examination: bilateral cataract was detected. Serum bilirubin was 15.6 mg/dL. A- What’s the most likely diagnosis? And what are the differential diagnosis? B- Mention 2 diagnostic investigations essential for the diagnosis. 4. An 18-month-old boy was presented with history of irritability, bowel disturbance and loss of appetite. On examination, his vital records and anthropometric measurements were accepted. On examination, there was an evidence of generalized tenderness. Hypertrophy of the gums was present. There was prominent costo-chondral junction. There was tender swelling in the end of the femur. Pallor, cervical lymphadenopathy, petechial hemorrhage in skin and mucous membrane were present. A- What’s the most acceptable diagnosis? And what’s the differential diagnosis of this presentation? B- Mention 2 diagnostic tests for this infant. 5. A child aged 6 years presented with pallor, jaundice and hepatosplenomegaly. A complete blood count with differential revealed Hb: 7 mg/dL, MCV: 95 fL, MCH: 30 picogram, MCH: 36%, osmotic fragility revealed hemolysis started at 0.6% NaCl, the reticulocyte count was 6%. The parents stated that several relatives have also suffered from a similar illness. A- Give the diagnosis and mention the main defect in this condition. B- Give 3 main lines of treatment of this child.

123


Examination for 5th year in Pediatrics Neonatology and healthcare October 26, 2009

Tanta University Faculty of Medicine Time allowed: 3 hours

(1) Neonatology: Q1) Long Assay: 1. Discuss pain in neonatal period. Q2) Short Assay: Give an account on: 1. Neonatal shock. 2. Primitive reflexes. Q3) Short answer: 1. Mention causes of neonatal cyanosis. 2. Define hypoglycemia in IDM. Q4) Problem solving: 1. A term 4200 gm female infant is delivered by cesarean section because of cephalo-pelvic disproportion. The amniotic fluid was clear and the infant cried immediately after birth. Within the first 15 minutes of life, her respiratory rate increased to 80 breathes/minute and she began to have intermittent granting. The infant oxygen saturation was 94%. The chest radiograph showed a fluid in the fissure, over aeration and prominent pulmonary vascular markings: A- What’s the most likely diagnosis? B- What’s the treatment? 2. Both the mother and the baby have O +ve blood group. The baby’s direct serum bilirubin was 0.2 mg/dL, with a repeat total serum bilirubin of 11.8 mg/dL. Urine bilirubin is positive. The hemoglobin is 17 gm/dL, and the platelet count is 278,000/uL. Reticulocyte count is 1.5%. The peripheral smear doesn’t show fragments or abnormal cell shapes. Liver enzymes and US were normal. G6pD and osmotic fragility testing were normal. A- What’s the most likely diagnosis? B- What’s the treatment? (2) Health care: Q1) Long Assay: 1. Discuss one of the national post-neonatal screening programs. Q2) Short Assay: Give an account on: 1. Mile stone in the first year of life. 2. Ruta virus vaccine.

124


Q3) Short answer: 1. Enumerate advantages of breast milk. 2. Plan for pre- and post- exposure prophylaxis of hepatitis B virus. Q4) Problem solving: 1. An 8-year-old boy suffering from all under initial therapy. He was brought to the pediatrics OPD complaining from acute chest pain on the right side. After 2 days, skin lesions appeared at the same side of the chest pain. The skin lesions include small vesicles in infra-scapular and inflammatory regions. Systemic examinations were normal. A- Mention the most likely diagnosis. B- Planning for management. 2. A 6-day-old boy was brought to the hospital with a history of excessive cry and not taking feeds. The child was born at home at full term by normal delivery. On examination, the mouth of the child was kept open. There was cyanosis, tachypnea and generalized rigidity. Umbilical stump was infected. A- What’s the most acceptable diagnosis for this presentation? B- What’re the prevention and treatment of this disease?

125


Examination for 5th year in Pediatrics August 28, 2008

Tanta University Faculty of Medicine Time allowed: 3 hours

Q1) Long Assay: 1. - Enumerate causes of generalized edema and give one example of each group. - Discuss the differential diagnosis of a child presenting with anasarca. 2. - Classify the causes of hemolytic anemia. - Manage a neonate suspected to have hemolytic anemia. 3. Diagnose an infant presenting with acute diarrhea complicated with early hypovolemic shock. 4. Plan the prevention of primary pulmonary tuberculosis. Q2) Problem solving: 1. The parents of a 5-day-old term infant noticed that he is jaundiced. Your physical examination is remarkable only for scleral icterus. The infant’s total bilirubin level is 18 mg/dL, with direct component of 0.4 mg/dL. C-reactive protein is 5 mg/dL. A- What’s the most likely diagnosis? B- Mention the suitable treatment for this baby. 2. Soon after birth, a term newborn infant presents with increased oral secretions and mild respiratory distress. He vomits after every feeding. The parental history is notable for polyhydramnions. Physical examination is significant for scaphoid abdomen and paucity of bowel sounds. A- What’s the most likely diagnosis? B- Mention the characteristic radiological finding of this disease. 3. A 9-year-old girl with bronchial asthma. On average she uses albuterol inhaler 3 times per week, but for the past 10 days, she has been wheezing both day and night and is using inhaler 3-4 times per day. On examination, you note working alae nasi. O2 saturation is 90%. A- Classify this patient according to severity of paroxysm. B- Mention 2 mandatory investigations for this girl. 4. A 7-month-old infant is pink. S1 and S2 are normal. There is a machinery continuous murmur which is best heard in pulmonary area. A mid-diastolic murmur is heard at the apex. B- What kesiob is typified by basal murmur. C- What does the apical murmur signifies. 5. A 7-year-old boy with complain of migratory arthralgia. 2 weeks ago, he had a sore throat. On examination, nonpuretic trunchal rash and tender swelling of the right knee and left ankle are evident. ESR is 50mm/hr, Anti-streptolysin O titers is 800u/ml, and ECG shows prolonged PR interval. A- Mention 2 major criteria for this disease in this patient.

126


B- Mention 2 minor criteria in this patient. 6. A 6-year-old male infant is admitted to the pediatric ward with non-bilious vomiting and dehydration. The parents note that their child has been vomiting for the last 3 weeks with increasing severity and frequency. A- Mention the most likely diagnosis. B- Mention the most likely acid base status of this infant. 7. A 5-year-old boy complaining of pallor, mild jaundice, and spelnomegaly. There was a past history of painful bone crisis. He had recently history of viral infection. His hemoglobin is 5.2 g/dL and reticulocyte is 0.4%. A- Mention the most likely diagnosis. B- Explain the low level of reticulocyte. 8. A 3-year-old girl has a 3-week history of bruising, pallor and fever. Her white blood cell count is 55,000 cells/mm3. Bone marrow contains 35% lymphoblast cells. A- Mention the most likely diagnosis. B- Mention 2 favorable prognostic factors in this patient. 9. A 9-month-old male infant has a 2-day history of fever, irritability and emesis. A urinalysis demonstrates 50-75 white blood cells per high power field and leukocyte cast. Urine culture grows > 100,000 organisms/ml. A- What’s the most likely diagnosis? B- Mention the most common organism that may cause this presentation.

127


Examination for 5th year in Pediatrics September 2, 2007

Tanta University Faculty of Medicine Time allowed: 3 hours

Q1) Long Assay: 1. Discuss the diagnosis of obstructive shock. 2. Discuss the differential diagnosis of gross hematuria in late childhood. 3. Discuss the management of acute rheumatic heart failure in infancy. 4. Discuss the prevention of Kernicterus (Bilirubin Encephalopathy). Q2) Problem solving: 1. A 10-year-old girl with rheumatic heart disease. She complains of a 10-day history of low grade fever. Physical examination reveals temperature is 38 oC, pulse is 100 b/m. A new cardiac murmur is heard. There’s no signs of congestive heart failure. ESR is 20 mm/hour. ASOT is 300 TU. A- What’s the most acceptable diagnosis of this new presentation? B- Mention 2 important diagnostic tests must be done for this girl. 2. A male infant was born at 32 weeks gestation via cesarean section. Soon after birth, he developed respiratory distress. Chest X-Ray shows a diffuse ground glass pattern with air bronchograms. A- Mention the laboratory investigation must be done for this infant. B- What’s the chronic respiratory complication that may occur in this boy? 3. A 4-year-old boy with new onset type I diabetes mellitus who is in diabetic ketoacidosis. Blood glucose= 500 mg/dL and serum K+= 2.5 mEq/L. A- Mention the type and dose of fluid therapy in the 1st hour. B- Mention the normal value for serum K+. 4. A 15-year-old boy has a 3-month history of fever, weight loss and night sweat. Physical examination reveals only cervical lymphadenopathy. Lymph node biopsy shows Reed-Strenberg cells. A- What’s the most acceptable diagnosis of this presentation? B- What’s the stage of the disease? 5. A 12-month-old girl presents with fever, vomiting and irritability. The disease has a subacute onset and slowly progressive course. examination reveals neck rigidity and bulging anterior fontanels. CSF leukocyte is 1500 with 80%n neutrophils. A- What’s the acceptable diagnosis of this disease. B- Mention 2 microorganisms that may produce this presentation. 6. A 5-month-old male infant is evaluated for generalized tonic-clonic convulsion. No family history of epilepsy. Temperature is 40.4 oC. Examination reveals pharyngitis. Neurological

128


examination is normal. A- What’s the most acceptable cause of this convulsion? B- Mention the risk factor for development of epilepsy in this boy. 7. A 9-month-old boy presents with profuse watery diarrhea. Physical examination reveals muscle wasting, loss of subcutaneous fat, severe loss of weight to less than 60% with generalized edema. Skin pinch returns very slowly. A- Classify this boy according to Wellcome classification. B- Mention the dose of plasma transfusion for this boy. 8. An infant aged 3 years, resented with chronic pallor, jaundice and splenomegaly. His investigations revealed: Hb content= 6 g%, Fetal Hb= 40% and reticulocyte count= 10%. A- What’s the most acceptable diagnosis of this presentation? B- Mention the most important line of prevention of this disease. 9. Soon after birth, a newborn infant presents with central cyanosis. Echocardiogram reveals: Tetralogy of Fallot. A- What’s the most acceptable cause of this early presentation? B- Mention the drug and its dosage for treatment of this infant. 10. A 12-year-old girl presents with a 2-week history of fever, anorexia, cough and chronic fatigue. On examination, you find mild jaundice, hepatosplenomegaly and ascites. Chest examination reveals generalized wheezes with coarse crepitations and positive D’Espine sign. Investigations reveal: ESR is 100mm/hr, lymphocytosis without blast cells and mild raise transaminases and -ve antinuclear antibodies. A- What’s the most acceptable diagnosis for this presentation? B- What’s the most acceptable cause of positive D’Espine sign in this girl?

129


Examination for 5th year in Pediatrics September 2, 2006

Tanta University Faculty of Medicine Time allowed: 3 hours

Q1) Long Assay: 1. Diagnosis of bronchopneumonia. 2. Discuss differential diagnosis of apical mid-diastolic murmur in infants. 3. Discuss clinical presentations of glucose 6-phosphate dehydrogenase deficiency. 4. Discuss management of an infant with acute diarrhea associated with severe hypertonic dehydration. 5. Discuss prevention of congenital hyperthyroidism. Q2) Problem solving: 1. A preterm infant developed respiratory distress after birth. Chest X-Ray shows a diffuse ground glass pattern with air bronchograms. A- What’s the most likely cause of this condition? B- Mention the definition of prematurity. 2. A previously well 6-month-old infant has had a runny nose for 2 days. Four hours ago, he has cough, and moderate respiratory distress. On examination, he has hyperexpansion of the chest and audible wheezing without rales. A- What’s the most likely diagnosis? B- What’s the most likely organism that causes this disease? 3. A 12-year boy presents with severe arthritis of the hips and sacroiliac joints. Laboratory studies reveal that the child is HLA-B27 +ve. A- What’s the most likely diagnosis? B- What’s the complication of this disease? 4. An 8-year-old girl has brief staring spells throughout the day. EEg shows a generalized 3-Hz spikes and wave discharge pattern arising from both hemispheres. A- What’s the most likely diagnosis? B- What’s the drug of choice for treatment of this disorder? 5. A 3-year-old girl has Acute lymphocytic leukemia. WBC count is 25,000 cells/cm3. Leukemic cells have pre-B-cell phenotype and are L3 according to FAB classification and demonstrate chromosomal translocation: t(12,21). A- What’s the unfavorable prognostic indicator in this child? B- Mention the indication of bone marrow transplantation in this disease. 6. An infant aged 11 months was presented with delayed walking and recurrent attacks of convulsions. His investigations revealed: Serum calcium= 6 mg/dL, serum phosphorus= 3.5 mg/ dL and serum alkaline phosphatase= 100 BU.

130


A- What’s the most acceptable cause of this convulsion? B- Mention one situation in which this complication may occur even with normal total serum calcium. 7. A 3-year-old girl has a 5-day history of dark-colored urine. physical examination reveals hypertension and mild edema. Urine analysis reveals hematuria, red blood cells cast and mild proteinuria. ASOT is 800 TU. A- What’s the most acceptable diagnosis? B- Mention 3 complications that may occur in this disease. 8. A 3-week-old female infant was presented jaundice and acholic stool. On examination, his liver is 5 cm below the costal margin. A total serum bilirubin of 15.6 mg/dL with direct component of 6.9 mg/dL. A- What are the most common 2 causes of this presentation? B- Mention 2 non-invasive diagnostic tools aiding in diagnosis of this condition. 9. A 3-week-old female infant was presented to Tanta University hospital for evaluation of purposeless involuntary movements. ESR is 40 mm/hr, ASOT is 600 TU. A- Mention one mandatory investigation must be done for this girl. B- Mention 4 types of involuntary movements. 10. A 4-day-old male infant has gross hematuria. Prenatal history of maternal gestational diabetes. Physical examination reveals a flank mass. Laboratory finding reveals thrombocytopenia. A- What’s the most acceptable diagnosis for this presentation? B- Mention the most common neonatal tumor.

131


Examination for 5th year in Pediatrics August 30, 2005

Tanta University Faculty of Medicine Time allowed: 3 hours

Q1) Enumerate differential diagnosis and investigations of: 1. Chronic cough in child aged 4 years. 2. Neonatal respiratory distress. Q2) Enumerate the biochemical and laboratory findings in: 1. Acute leukemia in children. 2. Acute renal failure. 3. Acute meningitis. 4. Diabetic coma. Q3) Discuss the difference between: 1. Down syndrome and cretinism. 2. Marasmus and kwashiorkor. Q4) Enumerate clinical picture, complications and treatment of: 1. Tetanic. 2. Sickle cell anemia. Q5) Enumerate causes, types and clinical presentations of: 1. Acute hepatitis. 2. Pericarditis.

132


Examination for 5th year in Pediatrics August 29, 2004

Tanta University Faculty of Medicine Time allowed: 3 hours Q1) Enumerate types, indications and problems of: 1. Artificial milk formula. 2. Blood transfusion. Q2) Enumerate the investigations for diagnosis of: 1. Liver diseases in children. 2. Nephrotic syndrome. 3. Acute meningitis. 4. Diabetic coma.

Q3) Give an account on resuscitation in children in infants and children. Q4) What’re the normal daily requirement and nutritional importance protein, iron and vitamin A & D. Q5) Discuss the mental development in the first year of life and enumerate causes and grades of mental evaluation. Q6) Give an account on congestive heart failure in children. Q7) Give an account on clinical picture, diagnosis, complications and treatment of typhoid fever in children. Q8) Give an account on acute bronchiolitis in children. Q9) Give an account on surgical emergencies in the newborn. Q10) What’re the types, causes and general management of GIT bleeding in infants and children.

133


Examination for 5th year in Pediatrics June 20, 2004

Tanta University Faculty of Medicine Time allowed: 2 hours

Q1) Enumerate the special characters of pulse and the clinical significance of each type. Q2) The investigational findings of: 1. CSF in septic meningitis. 2. Active infantile rickets. Q3) Discuss the chest signs of: 1. Consolidation. 2. Pneumothorax. Q4) Differential diagnosis of apical systolic murmur. Q5) Enumerate causes and Diagnosis of abdominal distention. Q6) Male aged 9 mo, We 7kg with frequent vomiting and diarrhea. On examination, the following signs observed: drowsiness, dry tongue, depressed anterior fontanel, sunken eyes, rapid deep respiration. Serum N+= 155mEq/L, serum K+= 4.5mEq/L and blood pH= 7.2. Bicarb= 20mEq/L. - What’s the diagnosis and what’re the main lines of treatment? Q7) GBoy aged 6 yrs presented with bluffness, edema, BP 15/100 mmHg, dark turbid tea color urine, After 2 days, he developed headache, drowsiness, vomiting and convulsions. - What’s the possible diagnosis and what’re the laboratory investigations? Q8) Child aged 5 yrs presented with pallor, fever, skin petechial rashes and bone ache. - Give two possible diagnosis and investigations for each.

134


Examination for 5th year in Pediatrics September 28, 2003

Tanta University Faculty of Medicine Time allowed: 2 hours Q1) Give an account on: 1. Urinary tract infections. 2. Neonatal respiratory distress syndrome. Q2) Enumerate the causes and diagnostic points of: 1. Acute pallor in infants and children. 2. Infectious lymphadenopathy. Q3) Clinical manifestations and treatment of: 1. Congenital hyperthyroidism. 2. Vitamin deficiencies in PEM.

Q4) Give a short account on: 1. Pulmonary TB. 2. Causes of a cyanotic congenital heart diseases and discuss one of them. Examination for 5th year in Pediatrics August 2, 2003

Tanta University Faculty of Medicine Time allowed: 1 1/2 hours

Enumerate the following: 1. The complications of: A. Post streptococcal glomerulonephritis. B. Pneumonia. 2. Differential diagnosis of: A. Pallor with splenomegaly. B. Arthritis. 3. Investigations of: A. A case of Active rickets. B. A case of dehydration. 4. Clinical manifestations of: A. Congenital heart failure. B. Down syndrome.

135


Examination for 5th year in Pediatrics September 19, 2002

Tanta University Faculty of Medicine Time allowed: 2 hours Q1) Give an account on: 1. Advantages and difficulties of breast feeding. 2. Hemorrhagic disease of the newborn. Q2) Enumerate the causes and management of: 1. Stridor. 2. Shock.

Q3) Enumerate causes, investigations and treatment of: 1. Acute renal failure. 2. Pericarditis. Q4) Enumerate causes and diagnostic points of: 1. Hepatomegaly with jaundice in infants and children. 2. Hematological causes of splenomegaly. Examination for 5th year in Pediatrics July 22, 2002

Tanta University Faculty of Medicine Time allowed: 1 1/2 hours

Enumerate the following: 1. Causes of delayed walking. 2. Causes of failure to thrive. 3. Laboratory investigations done in case with generalized edema. 4. Causes of hypochromic microcytic anemia. 5. Causes of enlarged head. 6. Clinical types of cerebral palsy. 7. Clinical signs of pulmonary hypertension. 8. Physical and mental milestones during the first year of life. 9. Complications of pneumonia. 10. Complications of gastroenteritis.

136



Growth and development 1) Give short notes on Stages of human growth and development. (Oct 2015) 2) Give short notes on Deciduous and permanent teeth. (Nov 2015) 3) Give short account on: the most important ossification centers in children up to the age of 10 years. (Dec 2012) 4) Enumerate the ossification centers present in full term neonate delivery. (Dec 2011, Aug 2010) 5) Enumerate mile stones in the 1st year of life. ( Jul 2002,Oct 2009) 6) Mention the deciduous teeth & the age of their eruption. (Dec 2012)

Developmental, behavioral disorders & preventive pediatrics

1) 2) 3) 4) 5) 6)

Discuss in details the common behavioral disorders. (Oct 2015) Mention factors affecting children behavior. (Oct 2011) Short account on: Enuresis in children. (Oct 2011) Short account on: Attention deficit & hyperactivity disorder. (Oct 2012) Mention indications of autism spectrum disorders in children. (Oct 2011 , Dec 2012) Short account on: Counseling of parents of child with behavioral disorders. (Nov 2015 ,Dec 2012) 7) Give short note on Indications of hepatitis B vaccine. (Oct 2015) 8) Give short note on Preparations available for passive immunity. (Nov 2015) 9) Give short note on Special vaccinations. (Nov 2015) 10) Give short account on DPT vaccine. (Dec 2012) 11) Give short account on BCG vaccine. (Oct 2011) 12) Short account on Rota virus vaccine. (Oct 2009( 13) Plan for pre & post exposure prophylaxis of hepatitis A virus. (Oct 2009) 14) Active prevention of hepatitis B. (Aug 2010, Oct 2012)

138


Nutrition 1) Enumerate advantages of breast milk. (Oct 2009) 2) Daily vitamin requirements for infants and children. (Nov 2015) 3) Give short note on Non vitamin D deficiency rickets. (Oct 2015) 4) Give short note on dried powdered milk. (Nov 2015) 5) Give short account on Non nutritional marasmus. (Nov 2015) 6) Enumerate 4 anti-infective properties of breast milk. (Aug 2010) 7) Mention contraindications of breastfeeding. (Oct 2015, Oct 2011) 8) Short account on: Colostrum. (Oct 2011, Aug 2009, Oct 2001) 9) Mention main problem in animal milk when given for feeding infants. (Dec 2010) 10) Give short account on Advantage & disadvantage of artificial feeding. (Dec 2012) 11) General rules of weaning. (Oct 2012) 12) Discuss KWS (def, clinical manifestations. Laboratory findings, DD, prognosis). (Dec 2011) 13) List common findings of KWS. (Aug 2009) 14) Clinical manifestation of marasmus. (Aug 2001) 15) Clinical manifestation & complications of infantile malnutrition. (Sep 2000) 16) Biochemical manifestation of vitamin D deficiency Rickets. (Aug 2001) 17) Short account on Tetany. (Oct 2012) 18) Mention features of excess intake of vitamin A. (Oct 2011) 19) Vitamin A deficiency & excess. (Aug 2010)

Genetics & teratology 1) 2) 3) 4) 5)

Give short account about Recurrence risk of Down syndrome. (Oct 2015) Give short account on Teratogenesis. (Nov 2015) Give an account on chromosomal abnormalities. (Dec 2011) Give short account on sex linked inheritance. (Oct 2009) Discuss trisomy 21 (incidence, geontypes, investigations, genetic counseling, complications & recurrence risk). (Oct 2012, Oct 2009) 6) Imaging studies in Trisomy 21. (Aug 2010) 7) Define mutagen. (Oct 2009)

139


Infectious diseases 1) Give short note on Etiological classification of fever. (Oct 2015) 2) Treatment of amoebiasis. (Oct 2015) 3) Give short notes on Etiology of encephalitis. (Nov 2015) 4) List non-infectious causes of fever of unknown origin. (Aug 2009) 5) Discuss chicken pox (etiology, C/P, complications, TTT & prognosis). (Dec 2012) 6) Diagnosis & complications of measles. (Aug 2001) 7) Clinical manifestations & complications of Rubella. (Dec 2011) 8) Management of bacterial meningitis in infants & children. (Aug 2010) 9) Prevention if pertussis. (Sep 2000) 10) Clinical manifestations of typhoid fever in infants & children. (Oct 2012) 11) Mention drugs used for TTT of Giardiasis & proper doses. (Oct 2012)

Pediatric Emergencies 1) Discuss in details diagnostic approach, treatment and prognosis of the comatose child. (Nov 2015) 2) Give short note on Prognosis of comatosed child. (Oct 2015) 3) Mention gastrointesinal decontamination in a poisoned child. (Oct 2012) 4) Etiology & clinical manifestations of respiratory failure. (Oct 2011) 5) Enumerate causes & management of shock. (Sep 2002) 6) Manifestations of multiple organ failure. (Aug 2009, Oct 2012) 7) Enumerate 8 monitoring data in pediatric shock. (Aug 2010) 8) Metabolic coma. (Aug 2010) 9) Mention factors increasing ICT in comatosed child. (Oct 2011) 10) Give short note on prevention & urgent TTT of increased ICT. (Dec 2012)

140


Endocrinology 1) 2) 3) 4) 5) 6)

Give short answer on Fluid therapy in diabetic ketoacidosis. (Oct 2015) Give short answer on Acquired hypothyroidism. (Nov 2015) Etiology of 1ry hypothyroidism. (Dec 2011) Enumerate neonatal signs of congenital hypothyroidism. (Aug 2010) Prevention of congenital hypothyroidism. (Sep 2006) Enumerate the investigations required for acquired hypothyroidism. (Oct 2012, Dec 2012) 7) Clinical Picture of type 1 DM. (Oct 2012) 8) Mention features of type 2 DM. (Dec 2011, Dec 2012) 9) Discuss diabetic ketoacidosis in children (def, pathophysiology, clinical features & management). (Oct 2011) 10) List causes of short stature. (Aug 2009)

Neonatology 1) Give short note on Clinical types and treatment of brachial palsy. (Oct 2015) 2) Give short answer about Ethical issues in neonatal resuscitation. (Oct 2015) 3) Discuss in details etiology, types, diagnosis, prognosis & TTT of neonatal seizures. (Nov 2015) 4) Give short note on patterns, etiology and prognosis of neonatal seizures. (Oct 2014) 5) Give short answer about Indications of immediate exchange transfusion in hemolytic disease of the newborn. (Oct 2014) 6) Discuss in details neonatal birth injuries. (Dec 2014) 7) Give short account on Idiopathic neonatal hepatitis. (Dec 2014) 8) Give short account on Treatment of neonatal sepsis or meningitis. (Oct 2013) 9) Discuss pain in neonatal period. (Oct 2009) 10) Mention causes of neonatal cyanosis. (Oct 2009) 11) Discuss the prevention of Kernicterus (Bilirubin Encephalopathy). (Sep 2007) 12) Enumerate DD & investigations of Neonatal respiratory distress. (Aug 2005, Sep 2003) 13) Give an account on Hemorrhagic disease of the newborn. (Sep 2002)

141


Gastrointestinal diseases 1) Give short notes on Pathophysiological mechanisms and treatment of persistant diarrhea. (Oct 2015) 2) Gibe short answer of TTT of constipation. (Oct 2015) 3) Give short answer about aphthous ulcer. (Nov 2015) 4) Give short account on stomatitis. (Oct 2014) 5) Short account on: DD of oral ulcers. (Sep 2012) 6) Mention differential diagnosis of congenital hypertrophic pyloric stenosis. (Oct 2013) 7) Give short note on Persistant diarrhea (definition, clinical importance and pahtophysiology). (Oct 2013) 8) Short answer of: diagnostic criteria of pathological gastro-esophageal reflux. (Nov 2012 , Dec 2011) 9) Give short account on Clinical manifestations and diagnosis of generalized malabsorption. (Dec 2014) 10) Enumerate causes of malabsorption syndrome. (Oct 2009) 11) Non-infectious diarrhea. (Dec 2011,Nov 2012) 12) Enumerate etiology & complications of diarrhea. (Aug2001) 13) Mention mechanisms of diarrhea. (Oct 2010) 14) Enumerate warning signs of dehydration. (Aug 2010) 15) Discuss management of an infant presenting with acute diarrhea with severe hypertonic dehydration. (Sep 2006) 16) Diagnose an infant presenting with acute diarrhea complicated with early hypovolemic shock. (Aug 2008) 17) Complications of gastroenteritis. (Jul 2002) 18) Short answer of: modified ORS. (Nov 2015 ,Sep 2012)

Hepatology 1) 2) 3) 4) 5) 6) 7) 8) 9)

Give short notes on Causes of cholestasis. (Oct 2015) Give short account on Etiology and epidemiology of hepatitis B. (Nov 2015) Give short notes on Etiology and epidemiology of hepatitis E. (Oct 2014) Complications of acute viral hepatitis A. (Oct 2011) Mention clinical features of portal hypertension. (Dec 2011) Enumerate 4 diagnostic tools of portal hypertension in pediatrics. (Oct 2011) Extra hepatic portal hypertension. (Oct 2009) Enumerate causes of cholestasis in infants & children. (Oct 2011) (Sep 2012) Short account on: DD of ascites.

142


Respiratory system diseases 1) Give short account on Manifestations and etiology of laryngeal obstruction. (Oct 2015, Oct 2013) 2) Give short account on Mid-night croup. (Oct 2014) 3) Give short answer about Criteria of severity of asthma paroxysm. (Dec 2014) 4) Give short account on pneumothorax. (Oct 2014) 5) Enumerate causes & management of stridor. (Sep 2002) 6) Enumerate causes of paroxysmal cough in pediatrics. (Aug 2010) 7) List common causes of cough in infancy. (Aug 2009) 8) Causes of infectious croup. (Aug 2009) 9) Short answer of the meaning of laryngismus stradiulosa. (Sep 2012) 10)Discuss in details: Def, Etiology, pathogenesis, Classification & TTT of bronchial asthma in children. (Nov 2012) 11)Diagnosis of bronchial asthma. (Aug 2001) 12)TTT of acute attack of bronchial asthma. (Aug 2001) 13)Prevention of bronchial asthma. (Aug 2010) 14)Causes of wheezy chest. (Aug 2000) 15)Discuss causes, clinical picture, complications & TTT of lobar pneumonia in children. (Oct 2011,Sep 2000) 16)Diagnosis & TTT of lobar pneumonia. (Aug 2001) 17)Discuss diagnosis of bronchopneumonia. (Sep 2006) 18)Complications of pneumonia. (Jul 2002) 19)Enumerate investigations of pneumonia in infants & children. (Aug 2010) 20)Causes & characteristics of serofibrnous pleurisy. (Dec 2014, Dec 2011) 21)Enumerate causes of pneumothorax in children. (Oct 2009) 22)Enumerate indicators of adding corticosteroids to anti-tuberculosis drugs. (Dec 2011) 23)Plan the prevention of 1ry pulmonary T.B. (Nov 2015, Aug 2008, Sep 2012)

143


Cardiovascular diseases 1) 2) 3) 4)

Complications of congenital cyanotic heart diseases. (Aug 2009) Give short account on Acquired complete atrio-ventricular block. (Nov 2015) Classification of congestive heart failure. (Nov 2015) Doses, route of administration and duration of anti-hypertensive drugs. (Oct 2015, Dec 2014) 5) Etiology and classification of pulmonary valve stenosis. (Dec 2014) 6) Mention the clinical picture of supraventricular tachycardia. (Oct 2013) 7) Give short account on Sydenham’s syndrome. (Oct 2013) 8) Complications of tetralogy of fallot. (Oct 2011) 9) Critical pulmonary stenosis. (Oct 2009) 10) Short account on restrictive VSD. (Nov 2012) 11) Enumerate atypical presentation of acute rheumatic fever. (Nov 2012) 12) Enumerate investigations & TTT of acute rheumatic carditis. (Sep 2000) 13) Minor manifestations of rheumatic fever. (Aug 2001) 14) Characterestic manifestations of rheumatic chorea. (Aug 2001) 15) Discuss clinical picture & management of congestive heart failure. (Aug 2010, Aug 2001,Aug 2000) 16) TTT of congestive heart failure. (Dec 2011) 17) Discuss in details Def, etiology, clinical picture, laboratory investigations, prevention &TTT of Infective Endocarditis. (Ot 2014 ,Sep 2012) 18) Enumerate Types of tachyarrhythmia. (Dec 2011) 19) Enumerate causes, investigations & TTT of pericarditis. (Sep 2002) 20) Enumerate endocrinal causes of hypertension. (Oct 2011) 21) TTT of hypertension. (Oct 2009)

144


Hematologic and Oncologic disorders 1) Short answer of CNS manifestations of iron deficiency anemia. (Nov 2012) 2) Discuss in details etiology clinical picture, laboratory diagnosis, differential diagnosis & TTT of iron deficiency anemia. (Oct 2015) 3) Give short answer of Indications of plasma exchange. (Oct 2015) 4) Give short answer for Tests for evaluation of hemostatic mechanisms. (Nov 2015) 5) Give short account on Transfusion modalities.(Dec 2014) 6) Hematological causes of splenomegaly. (Sep 2002) 7) Clinical manifestations of sickle cell anemia. (Nov2012) 8) Causes of hypochromic microcytic anemia. (Jul2002) 9) Classify causes of hemolytic anemia. (Aug 2008) 10) Causes & Clinical manifestations of hemolytic anemia. (Sep 2000) 11) Discuss clinical picture of G6PD deficiency anemia. (Sep 2006) 12) Laboratory & radiological findings of beta thalassemia major. (Aug 2001) 13) Mention causes of congenital aplastic anemia. (Oct 2009) 14) Mention the etiology of secondary aplastic anemia. (Oct 2013) 15) Enumerate lines of management of aplastic anemia in pediatrics. (Aug2010) 16) Causes of thrombocytopenic purpura. (Aug2000) 17) Causes of non-thrombocytopenic purpura. (Aug2009) 18) Clinical manifestations of idiopathic thrombocytopenic purpura. (Aug 2001) 19) Mention diagnostic criteria of Henoch-Schoenlein purpura. (Oct 2011) 20) Clinical manifestations & Laboratory findings of acute lymphoblastic leukemia. (Aug 2001) 21) Laboratory of acute lymphoblastic leukemia. (Aug 2010) 22) List complication of leukemia. (Aug 2009) 23) Hyperleukocytosis. (Oct2009) 24) Prognostic factors of acute lymphoblastic leukemia. (Sep 2012) 25) Complications of blood transfusion. (Oct 2011)

145


Nephrology and Urology 1) Give short note on Etiology and pathogenesis of acute renal failure. (Oct 2015) 2) Give short notes on Clinical presentations and treatment of acute renal failure. (Dec 2014) 3) Properties of normal urine. (Dec 2011) 4) Causes of red urine in a 4 years child. (Aug 2000) 5) Pathogenesis of minimal change nephrotic syndrome. (Nov 2012) 6) Complications of nephrotic syndrome in pediatrics. (Aug 2010) 7) Investigation of nephrotic syndrome. (Aug 2000) 8) Enumerate indications of renal biopsy in a child with nephrotic syndrome. (Oct 2010) 9) Characteristic manifestations of APSGN. (Aug 2009) 10) Complications of APSGN. (Aug 2009) 11) TTT of APSGN. (Oct 2011) 12) Indications of dialysis in acute renal failure. (Nov 2012) 13) Causes & TTT of acute renal failure. (Sep 2002 ,Aug 2009) 14) Enumerate lines of TTT of hyperkalemia.'mention doses' (Aug 2010) 15) Short answer of biochemical disturbance of acute renal failure. (Sep 2012) 16) Discuss the etiology, predisposing factors, clinical presentations, laboratory investigations & TTT of UTIs in infants and children. (Oct 2013) 17) Enumerate predisposing factors for UTIs in pediatrics. (Nov 2015, Aug 2010) 18) Investigations & TTT of UTIs. (Oct 2014, Aug 2001, Sep 2000)

146


Neurologic and Neuromuscular disorders 1) Enumerate clinical signs which help in detection of cerebral palsy in neonatal period. (Aug 2010) 2) Give short answer of Indications of anti-convulsant drug monitoring. (Oct 2015, Oct 2014) 3) Give short note on Status epilepticus. (Nov 2015) 4) Give short note on intractable epilepsy. (Dec 2014) 5) Give short answer about Criteria for diagnosis of flobby infant syndrome. (Dec 2014) 6) Give short note on infantile spasms. (Oct 2013) 7) Describe the features of ataxic cerebral palsy. (Oct 2013) 8) Etiology & clinical types of C P. (Jul 2002, Aug 2001) 9) Prevention of C P. (Sep 2000) 10) Primitive reflexes. (Oct 2009) 11) Short answer: diagnostic criteria of C P. (Sep2012) 12) Discuss in details Diagnostic criteria, assessment, grades, etiology, investigations, prevention & TTT of mental retardation. (Oct 2014) 13) Grades of mental retardation. (Oct 2011) 14) Enumerate causes of preventable mental retardation. (Aug 2010) 15) DD of hypotonic infant. (Dec 2011) 16) Mention recurrence risk factors of epilepsy. (Nov 2012) 17) Short account on febrile convulsion. (Nov 2012) 18) Mention risk factors for development of epilepsy as a complication of febrile seizures. (Oct 2011) 19) TTT of epilepsy in pediatrics. (Aug 2010) 20) Common anti-epileptic drugs. (Aug 2009) 21) Def of status epilepticus. (Dec 2011) 22) Up to date management of status epilepticus in PICU. (Oct2009) 23) Short account on: causes of acute convulsion. (Sep 2012)

147


Facebology

Copyright Š 2015/2016 Facebology. All Rights Reserved.


Turn static files into dynamic content formats.

Create a flipbook
Issuu converts static files into: digital portfolios, online yearbooks, online catalogs, digital photo albums and more. Sign up and create your flipbook.